Está en la página 1de 174

Introducción al

cálculo para ingeniería

Guía de apuntes
y problemas

Gonzalo Píngaro

2022

1
Índice

1. Introducción 5

2. Conjuntos, números y nociones de geometría 13


2.1. Símbolos y notaciones . . . . . . . . . . . . . . . . . . . . . . . . . . . . . . 13
2.2. Los números naturales, los enteros y los racionales . . . . . . . . . . . . . . 15
2.3. Los números reales . . . . . . . . . . . . . . . . . . . . . . . . . . . . . . . . 22
2.4. Los logaritmos . . . . . . . . . . . . . . . . . . . . . . . . . . . . . . . . . . 26
2.5. Polinomios y expresiones algebraicas . . . . . . . . . . . . . . . . . . . . . . 27
2.6. Los polinomios . . . . . . . . . . . . . . . . . . . . . . . . . . . . . . . . . . 29
2.7. Nociones de geometría y trigonometría . . . . . . . . . . . . . . . . . . . . . 35
2.8. Polígonos . . . . . . . . . . . . . . . . . . . . . . . . . . . . . . . . . . . . . 38
2.9. Circunferencia y círculo . . . . . . . . . . . . . . . . . . . . . . . . . . . . . 38
2.10. Cuerpos . . . . . . . . . . . . . . . . . . . . . . . . . . . . . . . . . . . . . . 38
2.11. Razones trigonométricas . . . . . . . . . . . . . . . . . . . . . . . . . . . . . 39
2.12. Funciones trigonométricas . . . . . . . . . . . . . . . . . . . . . . . . . . . . 45
2.13. Lógica proposicional . . . . . . . . . . . . . . . . . . . . . . . . . . . . . . . 48
2.14. Los conjuntos . . . . . . . . . . . . . . . . . . . . . . . . . . . . . . . . . . . 50
2.15. Cotas y extremos de los conjuntos . . . . . . . . . . . . . . . . . . . . . . . 53

3. Relaciones y funciones 63
3.1. Dominio . . . . . . . . . . . . . . . . . . . . . . . . . . . . . . . . . . . . . . 63
3.2. Funciones polinómicas. . . . . . . . . . . . . . . . . . . . . . . . . . . . . . . 66
3.3. Funciones homográficas . . . . . . . . . . . . . . . . . . . . . . . . . . . . . 76
3.4. Funciones exponenciales y logarítmicas . . . . . . . . . . . . . . . . . . . . . 79
3.5. Composición de funciones . . . . . . . . . . . . . . . . . . . . . . . . . . . . 80
3.6. Función inversa . . . . . . . . . . . . . . . . . . . . . . . . . . . . . . . . . . 81

4. Límites y continuidad 93
4.1. Álgebra de límites . . . . . . . . . . . . . . . . . . . . . . . . . . . . . . . . 93
4.2. Límites de funciones definidas a tramos . . . . . . . . . . . . . . . . . . . . 97
4.3. ¿Cómo evaluamos la continuidad de una función? . . . . . . . . . . . . . . . 100
4.4. Asíntotas . . . . . . . . . . . . . . . . . . . . . . . . . . . . . . . . . . . . . 103
4.5. Estudio de las funciones a tramos . . . . . . . . . . . . . . . . . . . . . . . . 106

2
5. Derivadas 109
5.1. Definición de la derivada . . . . . . . . . . . . . . . . . . . . . . . . . . . . . 109
5.2. Tabla de derivadas . . . . . . . . . . . . . . . . . . . . . . . . . . . . . . . . 112
5.3. Aplicaciones de las derivadas . . . . . . . . . . . . . . . . . . . . . . . . . . 120
5.4. Máximos y mínimos . . . . . . . . . . . . . . . . . . . . . . . . . . . . . . . 130
5.5. Optimización . . . . . . . . . . . . . . . . . . . . . . . . . . . . . . . . . . . 137
5.6. Funciones implícitas y sus derivadas . . . . . . . . . . . . . . . . . . . . . . 142

6. Integrales 147
6.1. ¿Cómo calculamos el área determinada por curvas? . . . . . . . . . . . . . . 154
6.2. Longitud de un arco de curva . . . . . . . . . . . . . . . . . . . . . . . . . . 159
6.3. Aplicaciones físicas de las integrales . . . . . . . . . . . . . . . . . . . . . . . 163
6.4. Métodos de integración . . . . . . . . . . . . . . . . . . . . . . . . . . . . . . 164
6.5. Tabla de integrales . . . . . . . . . . . . . . . . . . . . . . . . . . . . . . . . 173

7. Bibliografía y links multimedia 174

3
Registro

Quedan todos los derechos reservados. Está prohibida la duplicación total o parcial, la
reproducción por cualquier medio, y la comercialización sin la autorización del autor.

ISBN: 978-987-86-3807-2

Colaboraron con los problemas y las respuestas:

Diego Bagú Juan José Madsen

Alejandro Giordano Daniela Natero

Samira Abdel Masih Fabio Romani

Santiago Martin Fiore Sebastián Perdicaro

María Florencia Tavarone Alexis Muñoz Estefano

Compilación de los problemas y apuntes

Prof. Gonzalo Píngaro & Lic. Prof. Santiago M. Fiore.

4
1. Introducción

Estimados estudiantes de las carreras de Ingeniería sean ustedes bienvenidos a Mate-


mática I, de la Universidad Nacional Arturo Jauretche.

Es de vital importancia que lean atentamente las primeras páginas de este documento,
porque encontrarán los propósitos, objetivos, contenidos, la metodología que adoptamos
en la materia, el cronograma, los criterios de evaluación, la temporización de los contenidos
que vamos a desarrollar clase a clase y algunos comentarios acerca del examen diagnóstico
y sus implicaciones.

Esta guía de estudio les proporcionará un marco teórico -básico- que será complemen-
tado con la bibliografía pertinente.

Sugerencias

A continuación listamos algunas sugerencias para la cursada exitosa.

Destinale el tiempo y espacio que esta materia requiere. Además de las nueve horas
semanales de la cursada vas a necesitar algunas horas de estudio, trabajo y práctica en
tu casa. Intentá prever qué momento de la semana le podrás asignar a ello. Organizá
tu lugar físico de estudio, que será de vital importancia durante toda tu carrera
universitaria.

Utilizá con responsabilidad todas las instalaciones y servicios que provee la univer-
sidad. Aprovechá los espacios (aula, biblioteca, salones de estudio) y los recursos
humanos (docentes) al máximo. Cuidalos y respetalos. Sé puntual en los horarios, y
dejá las instalaciones en condiciones de uso.

Organizá el material de trabajo. Asegurate de contar con todo lo necesario en cada


una de las clases. Prepará distintos recursos para optimizar las tareas (por ejemplo,
elegí un color para marcar los problemas que no pudiste resolver, otro para los que
resolviste y querés verificar, etc.).

Utilizá de manera eficiente el tiempo de trabajo en clase. Dedicale el 100 % de la


atención; evitá el uso del celular u otros dispositivos que te distraigan. También
considerá que pueden distraer a tus compañeros y al docente. Trabajá en grupos, y
recurrí al docente cada vez que lo consideres necesario. Preguntá todas las dudas,
consultá por los problemas que no entendiste o no pudiste resolver.

Sé responsable con la asistencia. Tené en cuenta que, cada vez que faltás, es una clase
que te perdés. Intentá consultar con tus compañeros los temas vistos y los ejercicios
resueltos durante las clases en las que no estuviste. El hecho de haber faltado no es
argumento válido para no cumplir con las tareas propuestas.

Prepará los parciales con tiempo. Organizate con las actividades previas y con las
fechas. Si es necesario, pedí los días de trabajo que te corresponden, prevé llegar al
examen con anticipación, y enlistá el material que vas a necesitar.

Conservá el material de trabajo. Todos los exámenes, trabajos prácticos, etc., son
documentos que respaldan las notas subidas a las plataformas.

5
Contenidos preliminares a Matemática I

1. Conjuntos, números y geometría.

a) Los números naturales, los números enteros, los racionales y los números reales.
Propiedades. Operaciones en los conjuntos numéricos.
b) Polinomios y expresiones racionales algebraicas.
c) Geometría y trigonometría.
d ) Lógica proposicional. Tablas de verdad. Razonamientos lógicos.
e) Conjuntos. Propiedades y operaciones con los conjuntos.

Contenidos de la materia

2. Funciones

a) Dominio, raíces, ordenada, intervalos de crecimiento y decrecimiento. Paridad


de funciones. Funciones partidas. Composición de funciones. Función inversa.
b) Función lineal. Función cuadrática. Elementos. Otras funciones polinómicas.
c) Funciones racionales, trascendentes y trigonométricas.

3. Límite y continuidad.

a) Noción de límite. Álgebra de límites. Límites laterales. Existencia del límite.


b) Indeterminaciones: cociente de infinitésimos. Cociente de infinitos. Otras inde-
terminaciones.
c) Límites de funciones a tramos.
d ) Continuidad en un punto. Definición. Propiedades. Clasificación.
e) Asíntotas.
f ) Estudio de funciones a tramos.

4. Derivadas.

a) Concepto y definición de derivada. Derivabilidad.


b) Propiedades de las derivadas.
c) Aplicaciones a las derivadas: pendiente, recta tangente, puntos críticos, extremos
locales y absolutos. Puntos de inflexión.
d ) Polinomio de Taylor.
e) Optimización.
f ) Funciones implícitas y sus derivadas.
g) Teoremas de L’Hôpital, y del valor medio.

5. Integrales.

a) Integración de funciones.
b) Primitiva. Integrales indefinidas.
c) Integrales definidas e impropias. Suma de Riemann. Área bajo una curva.
d ) Aplicaciones de la integral definida: área bajo una curva, volumen de revolución
y longitud de arco.
e) Longitud de un arco de curva.
f ) Técnicas de integración. Método de sustitución, de partes y de fracciones sim-
ples.

6
Clase a clase

Sem. Clase Fecha Tema Problemas

1 1 5/4 Números naturales y reales.


2 8/4 Polinomios.
2 3 12/4 Funciones I.
4 15/4 Feriado Nacional
3 5 19/4 Funciones II.
6 22/4 Composición e inversa.
4 7 26/4 Funciones a tramos
8 29/4 Límites.
5 9 3/5 Continuidad
10 6/5 Asíntotas
6 11 10/5 Estudio de funciones I.
12 13/5 Consultas.
7 13 17/5 PRIMER PARCIAL.
14 20/5 Derivadas I
8 15 24/5 Derivadas II
16 27/5 Regla de la cadena.
9 17 31/5 Recta tangente
18 3/6 Extremos
10 19 7/6 Estudio de funciones II
20 10/6 Teoremas de L’Hôpital y Taylor.
11 21 14/6 Optimización
22 17/6 Feriado nacional.
12 23 21/6 Derivadas implícitas.
24 28/6 Diferencial y primitivas.
13 25 28/6 Integrales.
26 1/7 Áreas de superficies.
14 27 5/7 Longitud y volumen.
28 8/7 Método de sustitución
15 29 12/7 Método de partes
30 15/7 Método de fracciones simples
16 31 19/7 Otras aplicaciones
22/7 SEGUNDO PARCIAL
26/7 RECUPERATORIOS

7
El diagnóstico

El día 12 de abril se tomará un diagnóstico de una hora de duración. Los contenidos a


evaluar en dicho diagnóstico son:

Números racionales y reales. Ecuaciones logarítmicas y exponenciales.

Ecuaciones e inecuaciones. Trigonometría y ecuaciones


Sistemas de ecuaciones lineales. trigonométricas.

Ecuaciones cuadráticas. Función lineal y cuadrática.

Los parciales

Los parciales y los recuperatorios son escritos, y presenciales Los estudiantes podrán
contar con apuntes propios. No está permitido el uso de software, o de celular. La
duración de los parciales es de 3 (tres) horas.
Las fecha de los parciales son:

Primer parcial: 17 de mayo.

Segundo parcial: 22 de julio

En la fecha de recuperatorio los estudiantes podrán recuperar un parcial o los dos.

El material

Es importante que en las clases de matemática cuentes con:

La carpeta o cuaderno habitual (que es el material de consulta más importante).

La guía de trabajos prácticos, con el programa y los apuntes teóricos.

La calculadora (que es la herramienta más importante). Hablamos del instrumento


calculadora, no de una aplicación de celular que sirva como calculadora. Tengan en
cuenta que no está permitido el uso del celular en los exámenes. Antes de
empezar cualquier actividad es muy importante que leas el manual de tu
calculadora1 .

Hojas cuadriculadas o milimetradas para las gráficas de funciones.

Regla, escuadra, compás.

Material de escritura (lápices, lapicera, marcadores,...)

El GeoGebra instalado o bien la versión On line disponible.

Los apuntes deben ser de confección propia; no está permitido concurrir al examen
con libros o fotocopias.

Es aconsejable confeccionar los apuntes durante toda la cursada. No resulta


productivo esperar hasta último momento para armarlos.
1
Que lo podés descargar de internet, por ejemplo, en https://sites.google.com/view/
matematicaunaj/links/calculadoras.

8
Sugerimos socializar los apuntes con los compañeros de clase, de modo de verificar
que no falten cosas o que no haya cosas mal copiadas.

Agregá en los apuntes todas las fórmulas que usás en la resolución de los problemas,
porque es muy probable que sean necesarias para resolver los problemas de los
parciales.

Condiciones para aprobar la cursada

Para aprobar la materia es necesario tener una nota mayor o igual a 4 (cuatro) en cada
uno de los parciales (o en sus recuperatorios). Una vez cumplido este requisito estás en
condiciones de presentarte a rendir el examen final.

Condiciones para promocionar la materia

Para promocionar la materia es necesario tener una nota mayor o igual a 7 (siete) en cada
uno de los parciales. La nota de promoción es el promedio de las notas de los parciales.

La asistencia

Para aprobar la cursada tenés que tener el 66 % (o más) de la asistencia a las clases
presenciales; para promocioar la materia tenés que tener el 75 % (o más) de la asistencia
a las clases presenciales.
Es de vital importancia concurrir personalmente a las devoluciones de los exámenes,
dado que estas instancias son también momentos de aprendizaje. En la última clase,
además, se realiza el cierre del ciclo, en el que el docente hace la devolución de notas. Es
imprescindible que cada alumno concurra a estas fechas, dado que -entre otras cosas- el
docente vuelca las notas de los exámenes al sistema y el alumno debe corroborar que
éstas coincidan con las que se les transmitió.

Los trabajos prácticos y las respuestas breves

La presentación de los trabajos prácticos (TP) consignados por el docente es condición


obligatoria para promocionar la materia. En caso de no haber presentado uno o ambos en
tiempo y forma, pueden presentar un trabajo extra después de la fecha del segundo
parcial. Asimismo, la participación en las respuestas breves (RB) les permitirá
mantenerse al día con los temas tratados en cada clase.

Las autoevaluaciones

El propósito de las autoevaluaciones es que las empleen para evaluar su situación previa a
los parciales. Con éstas, podrán determinar cuánto más necesitan practicar, estudiar y
consultar, y qué contenidos necesitan más dedicación.

9
Conocimientos previos

Es imprescindible que, antes de comenzar a cursar Matemática I, tengas en claro algunos


conceptos que utilizaremos como herramientas en la materia, y que por cuestiones de
tiempo e incumbencia no se dictan en Matemática I. Entre ellos, destacamos:

Operaciones con números enteros. Resolución de ecuaciones lineales y


cuadráticas.
Operaciones con fracciones.
Relación entre las fracciones y su Cuadrado de un binomio.
expresión decimal.
Definición y propiedades de las Perímetro y área de triángulos y
potencias y raíces. cuadriláteros.

Estos conceptos mencionados, sus propiedades, y algunos problemas de aplicación los


podés encontrar en la presente guía, y repasar en el sitio
https://sites.google.com/view/matematicaunaj/material-extra.
Esperamos que puedas aprovechar todos estos recursos.

Contactos

La página de “Matemática I”:


https://sites.google.com/view/matematicaunaj/p%C3%A1gina-principal.

El sitio del SIU: https://guarani.unaj.edu.ar.

El grupo de Telegram: https://t.me/+ksoVwCaGFH4wMjUx

El correo electrónico del docente: propingaro@gmail.com

Objetivos

Son los objetivos de Matemática I, que el alumno pueda:

Revisar el concepto de función, y profundizar algunas características. Analizar


funciones polinómicas, homográficas, trigonométricas, exponenciales y funciones
definidas a tramos.

Calcular límites, resolver indeterminaciones, aplicarlos a la resolución de problemas


y al estudio de las funciones.

Comprender la definición de la derivada. Aplicar el cálculo diferencial a la


resolución de problemas y al estudio de funciones.

Relacionar la integral con la derivada y hallar funciones primitivas de ciertas


funciones dadas.

Aplicar las integrales al cálculo de área, de longitudes de curva y a la resolución de


problemas.

10
Propósitos

Los propósitos de los docentes de Matemática I son:

Brindarle a los estudiantes las nociones básicas de función y de sus elementos.

Mostrar variadas aplicaciones de las funciones como herramientas de resolución de


problemas de modelizaciones.

Introducir el cálculo diferencial e integral como una herramienta de resolución de


problemas.

Les proponemos en el presente material algunas situaciones referidas a la matemática en


sí, y variadas aplicaciones a la ingeniería, la ciencia, la física y la tecnología.
Nuestra propuesta está basada en la resolución de problemas, ya que consideramos este
abordaje como muy productivo, para que construyan su conocimiento matemático. Y
esperamos que, a través de la lectura de este material, puedan conocer y trabajar los
conceptos que atañen a la materia y elaboren las soluciones a los problemas planteados.
Les ofrecemos algunas recomendaciones de cómo pueden organizarse para aprovechar la
materia:

Cuando comiencen a resolver un ejercicio o un problema, lean toda la información


que te brindan y presten atención a lo que se solicita.

No duden en releer las consignas las veces que lo necesiten. La relectura es muy
importante para que puedan resolver cada situación, de manera adecuada.

Hagan lo posible por resolver todas las actividades. Si alguna les resulta difícil, les
sugerimos que pasen a la siguiente y la retomen más tarde para volver a pensarla.

Pueden usar una hoja como borrador para hacer los cálculos que necesites.

En algunas actividades aparecen figuras y dibujos que son para ayudarlos a pensar
y analizar la situación. No siempre tienen las medidas que indica el enunciado. Por
eso, en esos casos, se incluye la aclaración que dice: "La figura no esta a escala. Es
una figura de análisis para ayudarte a pensar".

Intenten realizar alguna anticipación sobre un posible resultado y su razonabilidad


y luego verifiquen con la calculadora o algún desarrollo que valide o rechace tu
conjetura.

Cierre de actas

El cierre de las notas, promociones y registro de las mismas se realiza a través del SIU
Guaraní. Una vez cerrado el período lectivo, las notas no se pueden modificar, por ello es
muy importante que cada alumno verifique que las notas subidas coincidan
con las notas que le transmitió su docente. Les pedimos por ello que estén atentos
en las fechas de cierre de notas y que, ante cualquier duda, consulten por mail. No
esperen a las fechas de finales dado que en esas instancias es muy complicado realizar
rectificación de notas.

11
*

Conjuntos, números y nociones de


geometría
2. Conjuntos, números y nociones de geometría

2.1. Símbolos y notaciones

Símbolo Significado Comentario ó ejemplo.


< Menor a < b se lee: a es menor que b.
> Mayor
≤ Menor o igual
≥ Mayor o igual
ȧ Múltiplo de a
= Igual

= Aproximadamente igual
6= Distinto
L Límite
L+ Límite por derecha
L− Límite por izquierda
I Integral
π Es el número pi Es un número irracional.
Su valor aproximado es 3,14159.
e Es la constante de Euler Es un número irracional.
Su valor aproximado es 2,7182818.
logb (a) Logaritmo en base b del número a
log (a) Logaritmo en base 10 del número a
ln (a) Logaritmo en base e del número a
f (x) Función que depende de la variable x
sen2 (x) Equivale a (sen x)2 Se aplica para muchas otras funciones,
en particular, para las
funciones trigonométricas.
f 0 (x) Derivada (primera) de la función f
f 00 (x) Segunda derivada de la función f Es la derivada de la derivada
R de la función f .
f (x) · dx Integral de la función f
Rb
a f (x) · dx Integral de la función f definida entre x = a y x = b
A(S) Área de la superficie S
α̂ Es el ángulo “alfa” o bien su medida o amplitud
lı́m f (x) Se lee “límite cuando x tiende a a de la función f
x→a
f (x) ba Es el resultado de calcular f (b) − f (a)
vo Velocidad inicial
vf Velocidad final
vm Velocidad media
xo Posición inicial o valor particular de la variable x
xf Posición final
Gr(P ) Es el grado del polinomio P (x)
N Conjunto de los números naturales No consideramos el cero.
N0 Conjunto de los números enteros no negativos N0 = N ∪ {0}
Z Conjunto de los números enteros Z = {0; 1; −1; 2; −2; · · · }
Z+ Conjunto de los números enteros positivos Z+ = N
Q Conjunto de los números racionales Es el conjunto de todos los números
que se pueden escribir en forma de fracción.
I Conjunto de los números irracionales
R Conjunto de los números reales
R+ Conjunto de los números reales positivos
R+0 Conjunto de los números reales no negativos

13
(a; b) Es el par ordenado en el que la primera componente es a y la segunda es b.
En otro contexto es un intervalo abierto; es el conjunto de todos los x tales que a < x < b
[a; b] Intervalo cerrado. Es el conjunto de todos los x tales que a ≤ x ≤ b
(a; b] Intervalo semiabierto. Es el conjunto de todos los x tales que a < x ≤ b
[a; b) Intervalo semiabierto. Es el conjunto de todos los x tales que a ≤ x < b
[a; +∞) Intervalo cerrado. Es el conjunto de todos los x tales que a ≤ x
(a; +∞) Intervalo abierto. Es el conjunto de todos los x tales que a < x
(−∞; b] Intervalo cerrado. Es el conjunto de todos los x tales que x ≤ b
(−∞; b) Intervalo abierto. Es el conjunto de todos los x tales que x < b
(−∞; +∞) Es el intervalo de todos los números reales.
Ec;R Entorno de centro c y radio R. Es el conjunto de todos los x tales que c − R < x < c + R
0
Ec;R Entorno reducido de centro c y radio R.
Es el conjunto de todos los x 6= c tales que c − R < x < c + R
∈ Pertenece o perteneciente 7∈N

/ No pertenece
∪ Unión entre conjuntos A ∪ B = {x : x ∈ A ∨ x ∈ B}
∩ Intersección entre conjuntos A ∩ B = {x : x ∈ A ∧ x ∈ B}
⊂ Incluido estrictamente A ⊂ B ⇔ (x ∈ A → x ∈ B ∧ A 6= B)
⊆ Incluido A ⊆ B ⇔ (x ∈ A → x ∈ B)
A Complemento de A A = {x : x 6∈ A}
∨ Es la disyunción o disjunción de proposiciones.
∧ Es la conjunción de proposiciones.
¬ Es la negación de una proposición.
⇒ Es la implicación.
⇔ Es la doble implicación.
v(p) = V La proposición p es verdadera.
v(p) = F La proposición p es falsa.
Domf ó Df Es el dominio de la función f .
Imf Es el conjunto imagen de la función f .
Codf Es el codominio de la función f .
f ◦g Es la composición de las funciones f y g. Lo podemos simbolizar f (g(x)).
f −1 Es la función inversa de f .
≡ Indica el nombre de un punto. p ≡ (−2; 3) indica que las coordenadas
del punto p son (−2; 3)
n! Factorial del número n. Es igual a 1 · 2 · 3 · · · n.
|x| Valor absoluto de x
f (n) Derivada n−ésima de la función f .
∀ Para todo
∃ Existe
6∃ No existe
#A Es el cardinal del conjunto A
∆ Antecediendo a una variable o función, indica variación
Es el discriminante de una ecuación cuadrática ∆ = b2 − 4ac
En la teoría de conjuntos indica la diferencia simétrica.
dy
P Es el diferencial de la función y. Se puede aplicar a cualquier función o variable.
Q Sumatoria
Productoria
± x = a ± b indica que x toma dos valores: x = a + b ∨ x = a − b
∓ x = a ∓ b indica que x toma dos valores: x = a − b ∨ x = a + b
∅ Conjunto vacío
sen−1 (x) Indica arcsen(x)
In Es el intervalo natural inicial. In = {x : x ∈ N ∧ x ≤ n}

14
2.2. Los números naturales, los enteros y los racionales

Los conjuntos numéricos son muy importantes en matemática, ya que si sabemos con cuál
de ellos estamos trabajando, podemos decidir qué resultados son válidos. Además existen
propiedades de las operaciones que usamos o no, según sea el conjunto numérico en cuestión.

Definiciones

Los números naturales: El conjunto de los números naturales es el conjunto de los


cardinales2 de los conjuntos no vacíos. Lo simbolizamos con la letra N. Entonces:

N = {1; 2; 3; · · · }

La unión entre el conjunto de los números naturales y el conjunto unitario {0} es el


conjunto de los números enteros no negativos y lo simbolizamos N0 . Entonces:

N0 = {0; 1; 2; 3; · · · }

Números opuestos: Dos números a y b son opuestos si a + b = 0. Notamos que el opuesto


al cero es el cero, ya que 0 + 0 = 0.
Números positivos y negativos: Decimos que un número es negativo si es menor que el
número cero, y es positivo si es mayor que cero. El cero no es positivo ni negativo.
Números enteros: Denominamos conjunto de los números enteros al conjunto formado por
todos los números naturales, sus opuestos y el cero. Lo simbolizamos con la letra Z.

Z = {0; 1; −1; 2; −2; 3; −3; · · · }

Problema 1. Resuelvan mentalmente las siguientes cuentas.

a) 12 − 5 + 4 = e) 2 + 3 · 5 =
b) 12 − (5 + 4) = f ) (2 + 3) · 5 =
c) 5 − 6 − 7 = g) (−2 − 3) · 5 =
d ) 16 − 4 + 1 = h) 2 + (3 · 5) =

Problema 2. Hallen el valor de x que verifique las igualdades. Comprueben el resultado


obtenido.

a) 3x + 6 = 27 e) 2 · (x + 1) = 3 · (x − 7)
b) 8 + 5x = −2
f ) −3(x − 5) + 2 = 2(x − 5) + 15
c) 29x − 12 = 17x
d) 16x + 1 = 13x − 5 g) 5 − 4(3 − x) = 6x + 3

Problema 3. Sean a y b números enteros distintos de cero. Determinen la validez de las


siguientes expresiones3 .

a) 2a + 2b = 2 · (a + b) a+b a b a 5a
c) = + e) 5 · =
5 5 5 b 5b
7 7 7
b) 2 · a · b = 2a · 2b d) = + f ) 3a · b = a · 3b
a+b a b
2
Esta definición es la definición formal de los números naturales. En la práctica, podemos decir que los
números naturales son aquellos que utilizamos para contar objetos.
3
Dos expresiones son iguales si son iguales para cualquier asignación de valores.

15
Problema 4. Relean las expresiones del problema anterior. Pongan un contraejemplo en
cada una de las expresiones que consideraron que no son válidas.

Problema 5. Calculen mentalmente las siguientes cuentas.

a) (2 + 1)3 = f ) 2 · (4 − 1)2 =
b) (3 − 1)2 = g) 2 + 32 =
c) 32 − 12 = h) −32 =
d ) 23 + 3 3 = i) (−3)2 =
e) (2 + 3)3 = j ) 5 + 4 · 32 =

Problema 6. Determinen en qué casos resulta P = Q, cualesquiera sean los números a


y b. Propongan un contraejemplo si P 6= Q.

a) P = (3a)2 Q = 32 · a2
b) P = (a + 3)2 Q = a2 + 3 2
c) P = ab Q = ba
d ) P = a2 · a3 Q = a2+3
e) P = a2 Q = −a2
f ) P = a2 · a3 Q = a2·3
g) P = a · a Q = 2a
h) P = b0 Q=1

Problema 7. Resuelvan las siguientes cuentas mentalmente.


√ √
a) 16 + 9 = c) 16 + 9 =
√ √ √3
b) 16 + 9 = d ) 16 + 11 =

Problema 8. Cada una de las siguientes ecuaciones tiene dos soluciones. Hallen ambas
soluciones y verifiquen los resultados obtenidos.

a) x2 = 16 c) 2(x − 5)2 = 50x − 296


b) (x + 1)2 = 144 d ) (x − 1)2 = 1

Problema 9. Armen un resumen con las propiedades, reglas, ejemplos y contraejemplos


que utilizaron para resolver los problemas anteriores.
Números primos: Un número entero es primo si tiene exactamente cuatro divisores
enteros distintos: él mismo y su opuesto, el 1 y el -1. Por ejemplo, 7 es primo, ya que sus
únicos divisores son 7, −7, 1, −1. En cambio, 9 no es primo porque es divisible por 3.
Tampoco es primo el número 1 ya que sólo tiene dos divisores.
Máximo común divisor (mcd): El máximo común divisor de dos o más números
enteros es el mayor divisor de los números dados4 . Para calcularlo contamos con varios
algoritmos.

Factorizamos los números (en factores primos), y calculamos el producto de todos


los factores comunes elevados a los menores exponentes.
Calculamos el resto en la división entre a y b. Según la propiedad vista
anteriormente, si dicho resto es igual a r entonces mcd(a; b) = mcd(a; r)
4
Si el máximo común divisor entre dos números a y b es 1, decimos que los números a y b son coprimos.

16
Problema 10. Calculen el máximo común divisor entre 60 y 72.

Propiedad. mcd(a; b) = mcd(a; a ± b)

Problema 11. Calculen el máximo común divisor entre 2550 y 726.


Mínimo común múltiplo (mcm): El mínimo común múltiplo entre dos números
enteros no nulos es el menor número entero que es múltiplo de ambos. Para calcularlo
conocemos al menos dos algoritmos.

Factorizamos los números (en factores primos) y calculamos el producto entre todos
los factores (comunes y no comunes) elevados a los mayores exponentes.
Si conocemos el mcd entre los números dados, y según la propiedad enunciada
a·b
anteriormente, resulta mcm(a; b) = .
mcd(a; b)

Problema 12. Calculen el mínimo común múltiplo entre 60 y 72.

Propiedad. Dados dos números naturales a y b; si r = mcd(a; b) y s = mcm(a; b)


entonces a · b = r · s.
Problema 13. Dos abejas vuelan desde el panal hasta una flor. La primera tarda 45
segundos en ir y volver. La segunda tarda 60 segundos. ¿Cuántas veces salen en el mismo
momento durante una hora?

Problema 14. Ana y Beto son dos hermanos que visitan periódicamente a su abuela.
Ambos fueron a visitarla el 1 de enero de este año. Ana vuelve a lo de su tía cada 12 días,
y Beto cada 15. ¿Cuántas veces en un año van juntos a visitar a su abuela?

Problema 15. Encuentren el error en el siguiente razonamiento.


Consideramos dos números a y b tales que

a=b

Entonces
a2 = a · b
Restamos b2 en ambos miembros.

a2 − b2 = a · b − b2

Factorizamos las expresiones5

(a + b) · (a − b) = b(a − b)

Cancelamos las expresiones idénticas

a+b=b

Por hipótesis a = b entonces


2b = b
Dividimos por b ambos miembros
2=1
5
Si bien dedicaremos una sección a la factorización de expresiones algebraicas, el hecho de que este paso
es correcto lo pueden verificar aplicando la propiedad distributiva de la multiplicación con respecto a la
suma y a la resta.

17
Los números racionales

Definición. Un número racional es aquel que puede expresarse en forma de fracción6 ; es


decir que es igual al cociente entre dos números enteros a y b con b 6= 0. En símbolos:
n ao
Q = x : ∃a ∈ Z; b ∈ Z; b 6= 0 : x =
b

Propiedad. Todo número racional es un número periódico en su expresión decimal.

Definición de la división

Dados dos números a y b (b 6= 0), decimos que a : b es igual a c si se verifica que


b · c = a.
Notamos que si b = 0 y a 6= 0, no existe c tal que 0 · c = a.
En el caso en que b = 0 y a = 0, cualquier c verifica que 0 · c = 0.
Por lo visto en los dos ítems anteriores no está definida la división por cero.
a
Existe una doble modalidad en cuanto a su escritura: a : b = b
La división sólo es distributiva con respecto a la suma y a la resta cuando está a la
derecha. Es decir: (a ± b) : n = a : n ± b : n y (n =
6 0).

Propiedades de las razones

En las operaciones y propiedades que siguen, consideramos todos los denominadores


distintos a cero.

Suma y resta.
a c ad ± bc
± =
b d bd
Multiplicación y división.
a c ac
· =
b d bd
a c ad
: =
b d bc

Propiedades de las potencias y las raíces

a0 = 1 a−n = a1n
p √
a1 = a a q = q ap
am · an = am+n (a · b)n = an · bn
am a n n
= abn

m−n
an = a b

n √ √
n
(am )n = am·n Si a ≥ 0; b ≥ 0 : a·b= n
a· b

na
n a
a−1 = 1
p
a
Si a ≥ 0; b > 0; b = √
n
b

Aclaraciones:

La radicación está definida para los índices naturales.


La raíz de los números negativos sólo está definida para los índices impares, es decir

que si n es par y a es negativo, n a.

18
Problema 16. Calculen las siguientes sumas y restas sin usar calculadora.
2 9 2 2 7
a) + = f) −1= k) − =
3 7 13 3 13
3 1 2 3
b) + = g) + = 4 1
5 4 5 10 l) − =
2 4 1 1 9 3
c) + = h) + =
15 21 6 3 1
7 75 3 2 m) 4 − =
d) − = i) + = 7
20 24 7 5
1 1 1 1 4 3
e) + − = j) + = n) +6=
2 3 4 9 15 4

Problema 17. Verifiquen los resultados obtenidos utilizando una calculadora científica.

Problema 18. Calculen las siguientes multiplicaciones y divisiones sin usar calculadora.
2 9 5 15 3 1 1 1
a) · = d) · = g) : = j) : =
3 7 4 2 5 4 2 3
3 1 6 5 2 4 3
b) · = e) · = h) : = k) 5 · =
5 4 5 3 15 21 25
2 21 2 9 7 5
c) · = f) : = i) : =
15 4 3 7 20 24

Problema 19. Comparen los resultados obtenidos en los ítems c y h. Extraigan


conclusiones y justifiquen su respuesta.

Problema 20. Verifiquen los resultados obtenidos utilizando una calculadora científica.

Problema 21. Calculen el resultado de las siguientes cuentas sin usar calculadora.
3
 
2 9 3
+ · −8 
1

3 7 2 b) 7 · −1 =
a) = 
1 2

4
1 3 5 · +
− 4 5
2 5

Problema 22. Calculen el resultado de las siguientes cuentas sin usar la calculadora.
2 0 3 17 3 −15 7 −1
   
a) 3 = c) 5 · 5 = e) 2 =
2 12 2 9 1 2 14
−16
d) = · 72
  
b) 7 : 7 = 7 f) 7 =
2

Problema 23. Comparen los resultados obtenidos en los ítems d y e. Extraigan


conclusiones y justifiquen sus respuesta.

Problema 24. Consideren las expresiones:

x = (a + b)2 ; y = a2 + b2

¿Es cierto que x = y? Extraigan conclusiones y propongan por lo menos seis ejemplos en
los cuales se demuestre dicha conclusión.

Problema 25. ¿En qué casos sucede que (a + b)n = an + bn ?

19
Problema 26. Completen la siguiente tabla con los resultados que se obtienen de
reemplazar los valores de a, b y c. Comparen las columnas y extraigan conclusiones.

a b c c · (a + b) (a + b)2 a2 + b2 (a + b)3 a3 + b3
2 3 5
0 −2 5
−1 0 4
4 −2 −3
−9 −1 7
−5 −7 −3
0 0 4
2
Problema 27. Prueben que 27− 3 = 19 . Escriban qué propiedades aplicaron.
Problema 28. Resuelvan la cuenta del problema anterior con la calculadora científica.
¿Llegaron al mismo resultado? Si la respuesta es afirmativa, continúen con el problema
siguiente. Si obtuvieron un resultado distinto, entonces busquen el error cometido y
expliquen por qué es un error.
Problema 29. Prueben que si a 6= 0, b 6= 0, a 6= b entonces
1
a− 1b 1
=−
a−b ab

Problema 30. Prueben que si a 6= 0, b 6= 0 entonces


1 b
a =
b a

Problema 31. Dada la proposición: Para cualquier número natural n, la expresión

m = 10n + 3(3n + 5) + 23

resulta un número múltiplo de 19. Indiquen verdadero o falso y justifiquen su respuesta.


Problema 32. ¿Es cierto que la expresión p = n2 − n + 17 genera (para todos los n
naturales) todos los números primos? Si la respuesta es afirmativa, expliquen porqué.
Sino, hallen el primer valor de n natural para el cual n2 − n + 17 no es primo.
Problema 33. Resuelvan las siguientes ecuaciones.
Problema 34. Resuelvan las siguientes ecuaciones.
4
a) 7x − 5x + 15 = x + 8 g) −7 = 3 − x
b) 2y + 3(y − 4) = 2(y − 3) h) 5 = 4
+ 7
x x
c) 17 = 10 2
t − t
d ) 4t = −8 i) −3 = − x5 + 8
x
9
b
b
e) x + 2a = 2a j ) 2[−(x − 1) + 4] = 5 + [−(6x − 7) + 9x]
f ) 17 = −2 + x6 k) 2
r = 1
p − 1
x (Despejen x).

Problema 35. Cierto día, estando el ascensor descompuesto, me vi obligado a bajar por
la escalera. Ya había descendido 7 escalones cuando vi al profesor Zizoloziz en la planta
baja, aprestándose a subir. Sin detenerme seguí el descenso con mi habitual paso regular,
saludo a Zizoloziz al cruzarme con él y, finalmente, comprobé con asombro que cuando
aún me faltaban 4 escalones por bajar el profesor acababa de alcanzar el rellano desde
donde yo había empezado el descenso. "Parece el gato con botas - pensé- cuando yo bajo
un peldaño él sube dos de una zancada". ¿Cuántos escalones tiene la escalera?

20
Problema 36. Calculen las dimensiones de un rectángulo sabiendo que su base es el
triple de su altura, y el perímetro es igual a 96 cm.

Problema 37. En un triángulo isósceles, la base mide la mitad que uno de los lados
iguales. El perímetro del triángulo vale 55 cm. Calculen la longitud de los lados de la
figura.

Notación científica

En el universo en que vivimos podemos encontrarnos con algunos números inmensamente


grandes, y con otros increíblemente pequeños. Por ejemplo:

El volumen de la Tierra es de 1.080.759 miles de billones de m3 .


La distancia del Sol a la estrella más cercana (α-Centauro) es de 40.681.440.000.000
km.
La edad del universo es aproximadamente 15.000.000.000 años.
En cosmología, la época de Planck es el período de tiempo durante el cual las cuatro
fuerzas fundamentales (interacción nuclear fuerte, interacción nuclear débil,
interacción electromagnética e interacción gravitatoria) están unificadas y no existen
todavía las partículas elementales. Su duración es de 0,000 000 000 000 000 000 000
000 000 000 000 000 000 001 segundos.
Un cm3 de estrella neutrónica pesa 1.000.000.000.000.000 kg.
Una ameba pesa 0,000 005 gramos.
Una célula humana mide 0,000 000 05 m de radio.
La velocidad de la luz es de 10.800.000 km/h.

Para representar estas cantidades de una forma más sencilla, abreviada y entendible a
primera vista utilizamos la notación científica, que consiste en escribir cada número como
el producto entre dos factores: un número real entre 1 y 10, y una potencia de 10.

Problema 38. Escriban los números a = 23600000000000 y b = 0, 000000000017 en


notación científica.

Problema 39. Calculen x = a · b y expresen el resultado en notación científica.


a
Problema 40. Calculen y = b y expresen el resultado en notación científica.

Problema 41. Discutan en pequeños grupos cómo expresa la calculadora de cada uno de
ustedes a los números en notación científica. Para ello prueben hacer cuentas con números
muy grandes o muy pequeños. Pueden utilizar los ejemplos que mencionamos al comienzo
de la sección. Experimenten el funcionamiento del modo científico de las calculadoras.

Problema 42. ¿Es posible escribir en notación científica en la calculadora? Sugerencia:


investiguen el uso de la tecla EXP de sus calculadoras.

Problema 43. Escriban a los números mencionados al comienzo de la sección en forma


de notación científica. Aproxímenlos con dos cifras decimales, por redondeo.

Problema 44. La masa de la estrella α-Centauro es 18 veces la masa solar. Averigüen la


masa del Sol y calculen la masa de la estrella.

Problema 45. ¿Cuántos segundos hay en un año?

21
Problema 46. Según la ecuación de Albert Einstein, la energía que produce un cuerpo es
igual al producto entre su masa y el cuadrado de la velocidad de la luz. Es decir que
E = m · c2 . ¿Qué energía produce una estrella cuya masa es de 3 · 1013 kg?

Si querés leer más acerca de notación científica y resolver otros problemas, podés
consultar en https://campus.unaj.edu.ar/mod/book/view.php?id=21097

Orden de magnitudes.

El orden de magnitud7 de un número es la potencia decimal del valor relativo de su cifra


significativa. Por tanto, se dice, por ejemplo, que dos números difieren 2 órdenes de
magnitud si uno es 100 veces más grande que el otro.
El uso más extendido de describir los órdenes de magnitud es mediante la notación
científica o las potencias de diez. Por ejemplo, el orden de magnitud de 1500 es 3, ya que
1500 puede escribirse como 1, 5 × 103 .

Valor absoluto

El valor absoluto (o módulo) de un número es la distancia que hay entre dicho número y
el cero. Al valor absoluto del número a lo simbolizamos |a| y lo definimos:
(
a si a ≥ 0
|a| =
−a si a < 0

Notamos que el valor absoluto de un número es siempre mayor ó igual a cero.


Si necesitan más actividades adicionales de valor absoluto, pueden consultar el siguiente
link: http://www.mat.uson.mx/~jldiaz/Documents/Desigualdades/
valor_absolutov1.pdf.

Propiedades. El valor absoluto cumple con las siguientes propiedades, para todos los
números a, b y k.

a) |a| = | − a| g) |a| = |b| ⇐⇒ a = b ∨ a = −b


b) |a · b| = |a| · |b| h) −|a| ≤ a ≤ |a|
c) |a + b| ≤ |a| + |b|
i) |a| ≥ 0
d) |a − b| ≤ |a| + |b|
j ) ab = |a|

e) |a − b| ≥ ||a| − |b|| |b| (si b 6= 0)

f) |a| = 0 ⇐⇒ a = 0 k ) |a| = k ⇔ a = k ∨ a = −k

l ) |x| ≤ k ⇐⇒ −k ≤ x ≤ k si k > 0 y x ∈ R
m) |x| ≥ k ⇐⇒ x ≥ k ∨ x ≤ −k si k > 0 y x ∈ R

2.3. Los números reales

Hasta ahora hemos trabajado con los números racionales (es decir, con aquellos que
pueden expresarse como el cociente entre dos números enteros). Pero existen números que
no pueden escribirse en forma de fracción. El primer matemático en registrar que

existen números con esta cualidad fue Hípaso de Metaponto8 , que demostró que 2 no es
un número racional.
7
Extraido de https://es.wikipedia.org/wiki/Orden_de_magnitud
8
https://es.wikipedia.org/wiki/H%C3%ADpaso_de_Metaponto.

22

Problema 47. Prueben que 2 no es racional.

Definición. Decimos que un número es irracional si no es racional. Al conjunto de


todos los números irracionales lo simbolizamos con la letra I. Entonces:

I = {x : x ∈
/ Q}

En general, durante el curso de Matemática I trabajaremos con los elementos del


conjunto de los números reales (que es la unión entre el conjunto de los números
racionales y el conjunto de los números irracionales). Entonces:

N ⊂ N0 ⊂ Z ⊂ Q ⊂ R
N ∪ {0} = N0
Z− ∪ {0} ∪ N = Z
Q∪I=R

Propiedades de los números reales

Propiedad Suma Multiplicación


Uniforme Si a = b entonces a + c = b + c Si a = b entonces a · c = b · c
Asociativa (a + b) + c = a + (b + c) (a · b) · c = a · (b · c)
Conmutativa a+b=b+a a·b=b·a
Elemento neutro a+0=0+a a·1=1·a
Elemento opuesto a + (−a) = −a + a = 0 a · ( a1 ) = ( a1 ) · a = 1;a 6= 0
Distributiva a(b + c) = a · b + a · c
Elemento absorbente a·0=0·a=0
Ley de anulación del producto a·b=0⇒a=0∨b=0

Ecuación cuadrática

Denominamos ecuación cuadrática a una ecuación de la forma9 ax2 + bx + c = 0 siendo


a, b y c números reales cualesquiera, con a 6= 0. Uno de los primeros matemáticos (si no el
primero) en hallar una fórmula general para resolver estas ecuaciones fue Bhaskara10 .
Una expresión equivalente a la ecuación dada es:

−b ± b2 − 4ac
x=
2a

A partir de esta expresión notamos que:

Si ∆ = b2 − 4ac > 0 entonces la ecuación tiene dos soluciones reales distintas.


Si ∆ = b2 − 4ac = 0 entonces la ecuación tiene una solución real (equivalentemente,
dos soluciones iguales, también denominada solución o raíz doble).
Si ∆ = b2 − 4ac < 0 entonces la ecuación no tiene soluciones reales.

En el caso de ∆ ≥ 0 (es decir, si la ecuación tiene soluciones reales) denominamos x1 y x2


a las soluciones de la ecuación. Las soluciones halladas verifican que:

b c
r1 + r2 = − r1 · r2 = ax2 + bx + c = a(x + x1 )(x − x2 )
a a

9
O reducible a la forma...
10
Breve biografía en https://es.wikipedia.org/wiki/Bhaskara_I.

23
Problema 48. Resuelvan las siguientes ecuaciones.

a) 7 = 4 − 12 t2 g) 20x + 75
2 = − 52 x2
b) −t2 + 5t = 0 h) −2x2 − 12 = −14 x
c) 4t2 − 7t = 0 i) −3x2 + 3x + 18 = 0
d ) 3x2 − 3x + 18 = 0 j ) − 12 x2 + 20x + 75
2 =0
e) 3x2 − 3x − 18 = 0 k) −2x2 − 16x − 32 = 0
f ) 5x2 − 20x − 25 = 0 l ) x2 − x − 2 = 0

Problema 49. Escriban cada ecuación cuadrática del problema anterior como el
producto de tres factores. Recuerden que a x2 + b x + c = 0 ⇔ a(x − x1 )(x − x2 ) = 0.

Problema 50. Resuelvan las siguientes ecuaciones.

a) (x + 3)(x − 9) = 0 f ) (w − 1)(3w + 2) = 4w

b) (7x − 3)(6x + 4) = 0 g) x − x + 7 = 5
√ √
c) x2 + 9x = 14 h) 2x − 5 − x − 3 = 1
√ √
d ) 15r2 = 2 − 7r i) x − x − 5 = 1
√ √
e) (x + 4)(x − 6) = −16 j ) 3x + 1 = 1 + x + 4

Sistemas de ecuaciones

En los problemas que siguen, expresen cada enunciado como una o varias ecuaciones y
resuélvanlas. Indiquen la o las soluciones de los problemas.

Problema 51. El largo de un rectángulo es 3cm más que el doble de su ancho. Su


perímetro es 108 cm. Hallen las dimensiones del rectángulo (es decir, el largo y el ancho).

Problema 52. María trabaja en una compañía educativa que provee materiales a las
escuelas de una región. Ella alquiló los servicios de una combi para hacer la entrega de
estos materiales y le cobran $200 diarios más $10 por kilómetro recorrido. María alquiló
la combi por tres días y pagó $1450. ¿Cuántos kilómetros viajó durante esos tres días?

Problema 53. El bono navideño de José fue calculado a base de un 23 % de su sueldo


anual. Su bono fue de $2477. ¿Cuál es su sueldo anual?

Problema 54. Carmen y Sonia vendieron 43 cajas de galletitas el mes pasado. ¿Cuántas
cajas de galletitas venderán cada una el próximo mes si Carmen duplica su venta y Sonia
triplica la suya para vender 108 cajas en total?

Problema 55. Andrés viajó por la autopista a una velocidad fija durante 3 horas. Luego
bajó la velocidad 15 km/h para utilizar una ruta (no la autopista). El viaje completo le
llevó 5 horas y recorrió una distancia de 245km. ¿Cuál fue su velocidad en la autopista?

Problema 56. En un examen de 20 preguntas, la calificación de un estudiante ha sido un


8. Si cada acierto vale un punto, y cada error resta dos puntos, ¿cuántas preguntas
respondió correctamente?

Problema 57. Marco fue al quiosco de la esquina, compró 5 gaseosas y 3 chocolates y


pagó $190. Polo fue al mismo quiosco, compró 6 gaseosas y 2 chocolates, y pagó $180. ¿A
qué precio se vende cada producto?

Problema 58. Resuelvan los siguientes sistemas de ecuaciones y realicen luego las
correspondientes verificaciones.

24
( (
y = 2x − 4, 0 = 25 5 2
2 + 10t − 2 t
a) 5
g)
x=y+2 vf = v0 + a t
(3
2 = −3x + 7y (
b) Vf = 10 − 5t
−8 = 5x − 8y h)
( 0 = 25 5 2
2 + 10t − 2 t
x + 2y = 6
c) 2x+y
(
=y−6 0 = 5 + 2t − 12 9, 8t2
( 3 i)
2(x − 32 y) = 5 vf = 2 − 9, 8t
d) 2 5
3x − y = 6
(
( 0 = 2t − 21 9, 8t2
x = 5 + 3t − 12 4t2 j)
e) vf = 2 − 9, 8t
0 = 3 − 4t
( (
9 = 7 + 3t + 12 a t2 12 = 14t − 2t2
f) k)
5=3+a t Vf = −7 + 5t

Problema 59. En una granja hay gallinas y ovejas. En total hay 30 animales. Si
podemos contar 80 patas, ¿cuántas gallinas y cuántas ovejas hay?

Problema 60. En un rectángulo la base mide 17cm más que la altura. Sabemos que el
área es 1260cm2 . ¿Cuánto mide el perímetro del rectángulo?

Problema 61. Una pinturería dispone de 650 litros de pintura azul y 200 litros de
pintura blanca, para hacer dos tipos de mezcla: Adriática, que lleva 3 partes de azul por
cada parte de blanca, y Tirrena, que lleva 4 partes de azul por cada parte de blanca.
¿Cuántos litros de cada mezcla debe hacer para agotar el stock de pintura blanca y azul?

Problema 62. Marisol irá al festival de cine. Hay entradas de $30, $40 y $50. Marisol
gastó $360 y vio 10 películas en total. ¿Cuántas entradas de cada precio compró? Den
todas las posibilidades.

Problema 63. Una compañía petrolera posee dos refinerías que producen derivados de
petróleo. Por cada barril de crudo, las dos refinerías producen las siguientes cantidades:

La refinería I produce 30 litros de diesel y 70 litros de nafta.


La refinería II produce 65 litros de diesel y 55 litros de nafta.

La demanda de diesel es de 8 millones de litros y la demanda de nafta es de 15 millones


de litros.

a) ¿Cuáles son las incógnitas en este problema?


b) Planteen una ecuación que represente la producción de cada combustible.
c) Planteen y resuelvan un sistema de ecuaciones que represente la situación.
d ) ¿Cuántos barriles de crudo necesitará producir cada refinería para satisfacer la
demanda?

Problema 64. En una industria hay varias máquinas que producen tubófonos
parafínico-cromáticos. El número de máquinas es tres unidades menor al número de
tubófonos que produce cada máquina por día. Si se producen 868 tubófonos diariamente,
¿cuántas máquinas hay? ¿Cuántos tubófonos produce cada una?

Problema 65. En un bolsillo tengo cierta cantidad de dinero; en el otro bolsillo tengo el
doble. En total tengo $390. ¿Cuánto dinero tengo en cada bolsillo?

25
Problema 66. La medida de los tres lados de un triángulo son tres números enteros
consecutivos. Si el perímetro mide 78 centímetros, ¿cuánto miden los lados? ¿Es posible
averiguar el área del triángulo con esos datos?

Problema 67. El perímetro de un terreno rectangular mide de 480 metros. Si el largo es


5 veces el ancho, ¿cuánto miden sus lados?

Problema 68. Calculen las longitudes de los lados de un rectángulo sabiendo que su
perímetro mide 30 metros y uno de los lados mide 3 metros menos que el otro.

Problema 69. Calculen las longitudes de los lados de un triángulo rectángulo sabiendo
que son tres números que están en progresión aritmética de razón 7. Aclaración: Si dos
números a y b están en progresión aritmética de razón r entonces a = b ± r.

Inecuaciones

Habitualmente resolvemos ecuaciones, ya que muchas veces necesitamos calcular algún


valor o valores de la una o varias incógnitas. Pero también existen problemas en los que
tenemos que calcular la incógnita cuando es mayor que determinado valor.
Denominamos inecuación a la expresión que presenta alguno de los símbolos: ≤, ≥, < o >
entre dos miembros.
A continuación presentamos tres ejemplos para abordar el proceso de resolución y cómo
se escribe su conjunto solución.

Problema 70. Resuelvan las siguientes inecuaciones. Además escriban el conjunto


solución correspondiente:

a) 3x + 1 < 2 b) −2x + 7 < 5 c) −x + 1 ≥ 2

2.4. Los logaritmos

Definición del logaritmo

El logaritmo en base a de argumento b es un número real c. Este número c es el


exponente al cual tenemos que elevar la base a, para obtener (el argumento) b.

loga (b) = c ⇐⇒ b = ac con a > 0, a 6= 1, b>0


Comentario: el logaritmo es la herramienta por excelencia para resolver ecuaciones
exponenciales.
Ejemplos:

a) log(1) = 0 ⇐⇒ 1 = 100 g) ln(1) = 0 ⇐⇒ 1 = e0


b) log(10) = 1 ⇐⇒ 10 = 101 h) ln(e) = 1 ⇐⇒ e = e1
c) log(100) = 2 ⇐⇒ 100 = 102 i) log2 (2) = 1 ⇐⇒ 2 = 21
d ) log(1000) = 3 ⇐⇒ 1000 = 103 j ) log3 (729) = 4 ⇐⇒ 729 = 34
e) log2 (8) = 3 ⇐⇒ 8 = 23 k ) log25 (5) = 0, 5 ⇐⇒ 5 = 250,5
f ) log5 (25) = 2 ⇐⇒ 25 = 52 l ) log(0, 1) = −1 ⇐⇒ 0, 1 = 10−1

26
Base 10, base e y otras bases

Las calculadoras11 disponen de dos teclas para calcular logaritmos y estas son:

Las teclas log o log calculan logaritmos en base 10 (por convención sabemos que
la base es 10 y no se escribe).
Las teclas ln o ln calculan logaritmos en base e (por convención sabemos que la
base es e y no se escribe; a estos logaritmos los denominamos neperianos en honor al
matemático John Napier). El número e es un número irracional12 y su valor
aproximado es 2, 7183.
Si el logaritmo tiene una base distinta de las dos anteriores, es necesario escribir
dicha base.

Si necesitamos calcular un logaritmo que esté expresado en una base que no sea 10 o e
usamos la propiedad del cambio de base, o la tecla log  dependiendo del modelo de
calculadora del que se trate.

Algunas propiedades de los logaritmos

a) loga (x · y) = loga (x) + loga (y)


 
b) loga xy = loga (x) − loga (y)
c) loga (x)n = n · loga (x)
logk (x)
d ) loga (x) = logk (a) (Propiedad del cambio de base).

2.5. Polinomios y expresiones algebraicas

Las expresiones algebraicas son necesarias para expresar en lenguaje simbólico, problemas
que se presentan en lenguaje coloquial, generar fórmulas que luego desembocan en
aplicaciones para las distintas ramas de la ingeniería o a veces son el punto de inicio para
abordar problemas.
Las expresiones algebraicas tienen un subconjunto muy particular que es el de los
polinomios y en este texto sólo nos ocupamos de los polinomios con coeficientes e
indeterminadas13 reales.
Les recomendamos a los estudiantes que busquen algún formalismo matemático o
definiciones rigurosas, leer el siguiente documento en Internet: http://www3.uah.es/
jmmartinezmediano/ACMat/ACM%20Tema%2003%20Polinomios%20y.pdf.
Por último consideramos importantísimo recordar que la división por cero no está
definida en matemática. Es decir que nunca los denominadores son iguales a cero. Por lo
tanto y para no complicar la notación que usamos, sobreentendemos que todos los
denominadores que proponemos siempre son distintos de cero.

11
Recomendamos que leas el manual de tu calculadora.
12
De hecho, es un número trascendente, es decir que no es la raíz de ningún polinomio de coeficientes
enteros. Podés conocer algo más de la base de los logaritmos neperianos mirando alguno de los videos
siguientes: https://www.youtube.com/watch?v=Z5czpA-fyMU; https://www.youtube.com/watch?v=MKgjf
-1XcNM
13
La parte literal.

27
Ejemplos de expresiones algebraicas

a+b x2 − 1 3x 5 + x4 − x7 + 31
2 x5
ax + x2 − x 5x+7 10 + x4 − x7 + 31
ab+mx−z
a5 + x+1 6x3 − 5mt − 10
4
2x5 + x− 7 − x7 + 31
√ x4
3x−1 + 2 −7x0,3 + x − 19 −7x5 − −x7 +5t4 −11

Aclaración: A los números de cada término -que están multiplicando la parte literal- los
denominamos coeficientes. Y al conjunto de las letras con sus correspondientes
exponentes lo denominamos parte literal. Por ejemplo:
En a + b hay dos términos, 1 · a y 1 · b , cada uno tiene coeficiente 1, pero la parte literal
en el primero es a y en el segundo es b.
En ax + x2 hay dos términos, 1 · ax y 1 · x2 , cada uno tiene coeficiente 1, pero la parte
literal en el primero es ax y en el segundo es x2 .
En 6 · x3 − 5 · mt − 10 hay tres términos, 6x3 , −5mt y −10. En el primer término el
coeficiente es 6 y la parte literal x3 . En el segundo término el coeficiente es −5 y la parte
literal es mt. En el tercer término el coeficiente es −10 y no tiene parte literal.

Términos semejantes en las expresiones algebraicas

Son aquellos términos que tienen la misma parte literal. Es decir que las indeterminadas
que aparecen son las mismas y están elevada al mismo exponente. En otras palabras para
que dos términos sean semejantes sólo pueden tener diferentes coeficientes. Mostramos
algunos ejemplos:
x2
2a es semejante a 3a ; x2 es semejante a ; 8mx es semejante a − 3mx
5

Suma y resta de expresiones algebraicas

Únicamente podemos sumar o restar los términos semejantes. Ejemplos:

1a + 5b + 7z + 15a + 2b =
= 1a + 15a + 5b + 2b + 7z =
= 16a + 7b + 7z

Multiplicación y división

Para multiplicar expresiones algebraicas aplicamos la propiedad distributiva de la


multiplicación en la suma (o en la resta) y las propiedades de potencias de igual base.
Luego por una cuestión de practicidad sumamos los términos semejantes (cuando sea
posible). Ejemplo:

(ma + x2 )(−5ma + 15ma3 ) = −5m2 a2 + 15m2 a4 − 5max2 + 15ma3 x2


Para dividir expresiones algebraicas aplicamos la propiedad distributiva de la división en
la suma (o en la resta) y las propiedades de potencias de igual base. Luego por una
cuestión de practicidad sumamos los términos semejantes (cuando sea posible). Ejemplos:

m5 a6 + x2 m5 a6 x2 a x2
= + = − + −
−6m7 a5 −6m7 a5 −6m7 a5 6m2 6m7 a5

a3 m 8 x2 1 m 6 x2
(a3 + m8 x2 ) : (m2 a5 ) = + = +
m2 a5 m2 a5 m2 a2 a5

28
2.6. Los polinomios

Definición. Un polinomio es una expresión algebraica en donde todas las potencias de


las indeterminadas son números enteros no negativos.

En esta sección trabajaremos con polinomios de una indeterminada (que habitualmente


denotaremos con la x). Entonces un polinomio en la indeterminada x lo podemos definir
como
n
X
P (x) = ak · xk = a0 + a1 x + a2 x2 + · · · + an xn con n ∈ N0 y an 6= 0
k=0

Al número n lo denominamos Grado del polinomio y lo simbolizamos gr(P ); al número


an lo llamamos Coeficiente principal del polinomio.

Ejemplos de polinomios:

P (x) = 3x + 2 C(x) = 6x2 − 6x − 10
A(x) = x2 − 1 D(x) = −7x3 + x − 19
B(x) = −x2 + 7x E(x) = 2x5 + x4 − x7 + 31

Ejemplos de expresiones algebraicas que NO son polinomios


5 √
F (x) = 2x−1 + 1 I(x) = x3 + 8 − 7x 8 − 9x2 + − 5x4
1
G(x) = −x5 − 3x0,5 − 25 J(x) = − 49 x 2 + 5x3
2

3
H(x) = 5x 3 − x + 8x6 K(x) = 13x 2 − 2x + 2

Problema 71. Discutan en grupo, argumenten y justifiquen porqué las últimas seis
expresiones no son polinomios.

Características de un polinomio

La indeterminada en cada término está elevada a un exponente que pertenece al


conjunto de los enteros no negativos, es decir a N0 .
El polinomio tiene un coeficiente principal que lo denominamos an (siempre an 6= 0;
este es el coeficiente que multiplica a la indeterminada de mayor exponente).
El término en el que sólo aparece un número lo denominamos término independiente.
(En particular si no lo observamos en el polinomio, entonces el término
independiente es cero, ya que todos los polinomios tienen término independiente).
Los coeficientes del polinomio son números reales. En particular, si todos los
coeficientes son iguales a cero, al polinomio lo denominamos polinomio nulo.
Cualquier número distinto de cero es un polinomio y su grado es cero.
El polinomio nulo, P (x) = 0, no tiene grado.
Si an = 1, el polinomio se llama mónico, normalizado o reducido.

Polinomio ordenado y completo

A cada término del polinomio lo denominamos monomio. A los monomios de un


polinomio los podemos ordenar en forma creciente o decreciente según el grado. Optamos
por ordenarlos siempre en forma decreciente.

29
Un polinomio está completo si en cada término la indeterminada aparece elevada a
los distintos exponentes desde n hasta 0.
Cuando faltan algunos de los términos, podemos completar el polinomio agregamos
los términos faltantes con coeficiente igual a 0.

Problema 72. Completen la siguiente tabla para los polinomios

A(x) = −x8 + x9 − 17 D(x) = 11


B(x) = 5x2 + 12 x − 9 E(x) = 4
3 − 7x5 + x3 + x10
x4
C(x) = 3 + x2 + x N (x) = 0

Polinomio Coef. principal Mónico Grado Tér. ind. Ordenado Completo


A(x) =

B(x) =

C(x) =

D(x) =

E(x) =

N (x) =

Especialización, especificación o valor numérico de un polinomio

La especialización o valor numérico de un polinomio es el valor que obtenemos tras


reemplazar la indeterminada por un número real cualquiera. Por ejemplo:

Si P (x) = 3x + 8 y x = 2 entonces P (2) = 3 · 2 + 8 → P (2) = 14; y decimos que P


especificado en x = 2 es 14.
Si A(x) = −x2 + 3x + 1 y x = −1 entonces
A(−1) = −(−1)2 + 3(−1) + 1 → A(−1) = −3; y decimos que A especificado en
x = −1 es −3.

Raíz de un polinomio

La raíz de un polinomio es un número cuya especialización es cero.


Ejemplos:

Si P (x) = x + 1 y x = −1 entonces P (−1) = 0, por lo tanto x = −1 es raíz de P (x).


Si A(x) = x2 − 5x + 6 y x = 2 entonces A(2) = 0. Y si x = 3 entonces A(3) = 0. Por
lo tanto x = 2 y x = 3 son las raíces de A(x).
Si B(x) = x2 + 6 entonces B(x) no tiene raíces reales.

Aclaración: un polinomio de grado n tiene a lo sumo n raíces reales. Es decir: puede


suceder que no existan raíces reales, que exista sólo una o que existan dos o más. Pero
como máximo, un polinomio de grado n tiene n raíces reales.

30
Orden de multiplicidad de las raíces de un polinomio

Cuando nos referimos al orden de multiplicidad de las raíces de un polinomio, vemos si


esa raíz es única, se repite dos veces o más. Entonces si la raíz del polinomio:

Aparece sólo una vez, el orden de multiplicidad de la raíz es 1 o decimos que la raíz
es simple. Por ejemplo: P (x) = 2x + 1, sólo tiene una raíz que es x = − 21 .
Aparece dos veces (la misma raíz), el orden de multiplicidad de la raíz es 2 o
decimos que la raíz es doble. Por ejemplo: P (x) = x2 + 6x + 9 es equivalente a
P (x) = (x + 3)2 y x = −3 es raíz doble (o tiene orden de multiplicidad 2).

El lector puede verificar que P (x) = x4 + x3 − 3x2 − 5x − 2 tiene una raíz igual a (−1) y
su orden de multiplicidad es 3. La otra raíz es 2 y es de multiplicidad 1.

Operaciones entre polinomios

Suma y resta.
La suma de dos polinomios es otro polinomio que se obtiene realizando la suma de los
coeficientes de los monomios semejantes.

Problema 73. Dados: A(x) = 3x + 7 B(x) = 7x − 29 F (x) = x2 + 1 G(x) = 9x − 2


Calculen: A(x) + B(x) y F (x) + G(x).

Multiplicación de un monomio por un polinomio.


El producto de un monomio por un polinomio es otro polinomio y lo obtenemos apli-
cando la propiedad distributiva de la multiplicación con respecto a la suma. Luego
realizamos las multiplicaciones en cada uno de los términos.

Problema 74. Dados los polinomios A(x) = 3x ; B(x) = 7x − 29. Calculen:


A(x) · B(x)

Multiplicación entre dos polinomios.


El producto de dos polinomios es otro polinomio y lo obtenemos aplicando la propiedad
distributiva de la multiplicación con respecto a la suma. Luego operamos y sumamos
los términos semejantes.

Problema 75. Dados: C(x) = 5x2 + x − 17, D(x) = 7x2 + 12 x + 9


Calculen: C(x) · D(x).

División de un polinomio por un monomio.


Para dividir un polinomio A(x) por un monomio B(x), aplicamos la propiedad distri-
butiva de la división con respecto a la suma.

Problema 76. Dado el polinomios A(x) = 3x + 7, y B(x) = 7x


Calculen A(x) : B(x), con x 6= 0.

Regla de Ruffini14

La Regla de Ruffini es un método para realizar la división entre polinomios ordenados y


completos, tales que:

El polinomio dividendo sea de grado mayor o igual que 1.


El polinomio divisor sea de la forma15 (x − a).
14
Valentano, 1765 - Módena, 1822 Matemático y médico italiano.
15
Si el divisor es de la forma (x + b) entonces consideramos el número a = −b y obtenemos un divisor
de la forma (x − a) con a negativo.

31
Algoritmo para aplicar la Regla de Ruffini

Verificamos que el grado del polinomio dividendo sea mayor o igual que el grado del
polinomio divisor.
Verificamos que el polinomio divisor sea un polinomio mónico de grado 1, ordenado
y completo.
Verificamos que el polinomio dividendo esté ordenado y completo. Si esto no es así,
previamente tenemos que ordenar y completar el polinomio dividendo.
Construimos la disposición práctica con los coeficientes numéricos del polinomio
dividendo.
Colocamos la raíz del polinomio divisor a la izquierda de la línea vertical.
Ubicamos el coeficiente principal del polinomio dividendo, que no se modifica,
debajo de la línea horizontal.
Efectuamos las correspondientes multiplicaciones y sumas hasta agotar los
coeficientes numéricos.

Problema 77. Dados: F (x) = 13x3 + 4x − 1 y G(x) = x + 1, calculen F (x) : G(x).


Disposición práctica
13 0 4 −1
−1 −13 13 −17
13 −13 17 −18

El polinomio cociente es C(x) = 13x2 − 13x + 17.


El polinomio resto es R(x) = −18.
Sabemos que x = −1 no es raíz del polinomio F , ya que el polinomio resto es
distinto de cero.

Verificación de lo obtenido en la regla de Ruffini

De la misma manera que verificamos una división de números enteros, podemos verificar
la división de polinomios aplicando la definición correspondiente; es decir, viendo si se
cumple que:
P olinomio Dividendo = P olinomio divisor × P olinomio Cociente + P olinomio Resto.
En símbolos: D(x) = d(x) · C(x) + R(x).

Problema 78. Dados A(x) = x2 + x − 12 y B(x) = x − 3, calculen el cociente y el resto


en la división A(x) : B(x).

El Teorema del resto

El resto de la división entre un polinomio P (x) y otro polinomio de la forma (x − a) es


igual a P (a).

Problema 79. Verifiquen el resultado obtenido en los dos problemas anteriores.

Problema 80. Calculen el resto de la división P (x) : A(x) si P (x) = x2 + 5x − 14 y


A(x) = x + 7.

32
Factorización de polinomios

Q P (x) de grado n es hallar dos o más polinomios


Decimos que factorizar un polinomio
Ak (x) (k ∈ N) tales que P (x) = nk=1 an · Ak (x).
Los recursos más habituales que podemos usar para factorizar determinados polinomios
son:

Extraer factor común. Por ejemplo: 2ax3 + 12a2 x2 = 2ax2 (x + 6a)


Escribir un trinomio cuadrado perfecto (x2 ± 2xa + a2 , a ∈ R − {0}) como el
cuadrado de un binomio (x ± a)2 .
Ejemplo:
A(x) = x2 − 12x + 36
P (x) = (x − 6)2

Escribir un cuatrinomio cubo perfecto (x3 ± 3x2 a + 3xa2 ± a3 , a ∈ R − {0}) como el


cubo de un binomio (x ± a)3 .
Ejemplo:
B(x) = 8x3 + 12x2 + 6x + 1
B(x) = (2x + 1)3

Escribir una diferencia de cuadrados como el producto de dos binomios conjugados.


Ejemplo:
C(x) = x2 − 9
C(x) = (x − 3)(x + 3)

Escribir el polinomio de grado dos que está en forma polinómica


(ax2 + bx + c, con a 6= 0) en forma factorizada: a(x − x1 )(x − x2 ), aplicando la
fórmula resolvente de una ecuación cuadrática para calcular sus raíces reales.
Ejemplo:
D(x) = 3x2 + 12x − 15
D(x) = 3 · (x − 1) · (x + 5)
Ya que, luego de aplicar la fórmula resolvente de la ecuación cuadrática, obtenemos
que las raíces de D(x) son x = 1 ∧ x = −5.
Aplicar la regla de Ruffini en un polinomio de grado 3 para obtener un polinomio
cociente de grado 2 y luego seguir factorizándolo como en el ítem anterior.
Aplicar el Teorema de Gauss para polinomios, (Les sugerimos que vean
http://www.oma.org.ar/omanet/misc/00-06.htm)

Problema 81. Factoricen las siguientes expresiones.

a) 5m2 + 15m3 = k ) a2 − 2ab + b2 =


b) ab − bc = l ) a2 + 2ab + b2 =
c) x2 y + x2 z = m) x2 − 2x + 1 =
d ) 2a2 x + 6ax2 = n) y 4 + 1 + 2y 2 =
e) 8m2 − 12mn = ñ) a2 − 10a + 25 =
f ) a2 + ab + ax + bx = o) x2 − y 2 =
g) am − bm + an − bn = p) a2 − 4 =
h) ax − 2bx − 2ay + 4by = q) 1 − 4m4 =
i) a2 x2 − 3bx2 + a2 y 2 − 3by 2 = r ) a2 − 25 =
j ) 3m − 2n − 2nx4 + 3mx4 = s) 4a2 − 9 =

33
t) 4x2 − 24 = w ) x4 + 27x =
u) −60 − 2x + 12x2 + 2x3 =
v ) 2x2 + 6x − 12 = x ) x3 y − 5xy =

Problema 82. Hallen las raíces de los siguientes polinomios y escríbanlos en forma
factorizada.

a) x2 − 5x + 6 c) 2a2 − 18a + 28 e) 3x2 + 2x − 16


b) 3x2 − 15x + 18 d ) 5x2 + 7x − 12 f ) 3x2 − 12

Problema 83. Factoricen los siguientes polinomios, sabiendo que a es una de sus raíces16 .

a) x3 − 4x2 − 11x + 30 ; a = 2 d ) x5 + x4 + 4x3 + 4x2 + 4x + 4 ; a = −1


b) x3 − 5x2 − 49x + 245 ; a = 5 e) 3x3 + 9x2 + 9x + 6 ; a = −2
1 3
c) x3 + x2 − 8x − 12 ; a = 3 f) 2x + 2x2 − 10x − 24 ; a = 4

Problema 84. Discutan en pequeños grupos, cómo podemos verificar la veracidad de una
factorización. Utilicen el o los métodos propuestos para verificar los problemas anteriores.

Simplificación de expresiones algebraicas

En ocasiones, cuando la expresión algebraica es racional17 podemos factorizar el


numerador y el denominador y luego simplificar el o los factores que tienen en común el
numerador y el denominador (si existen).
Aclaración: Recuerden que si la expresión algebraica es racional y no la pueden factorizar
entonces no pueden realizar simplificaciones. (Descartamos el caso de que el numerador
sea igual al denominador, ya que entonces lo propuesto se simplifica y es igual a 1).

Problema 85. Simplifiquen las siguientes expresiones racionales.


4x − 4 2x2 − 9x − 5 x2 + 4x + 4
a) e) i)
6x − 6 10 + 3x − x2 x+2
a2 − b2 a2 + a − 2 x2 − 16
b) f) j)
a−b n − an − m + am (x + 4)(x − 4)
m 2 − n2 4x2 − 4xy + y 2 x2 x−1
c) g) k) − x+2
(m − n)2 5y − 10x x2 − 4
y x
x2 − x − 12 2x2 − 22x + 60 x − y
d) h) l) 1 1
16 − x2 75 − 3x2 y − x

Resolvemos los tres primeros ítems.

a) Para simplificar la expresión racional 4x−4


6x−6 factorizamos el numerador y el
denominador. En este ejemplo en usamos factor común en ambos. Luego cancelamos
los factores que estén en el numerador y el denominador. Entonces obtenemos:

4x − 4 4(x − 1) 2
= =
6x − 6 6(x − 1) 3
16
No olviden el coeficiente principal cuando escriben la expresión factorizada
17
Es decir, tiene numerador y un denominador.

34
−b 2 2
b) El numerador de la expresión aa−b lo factorizamos según una diferencia de
cuadrados. Luego realizamos la simplificación correspondiente. En símbolos:

a2 − b2 (a + b)(a − b)
= =a+b
a−b a−b
m −n 2 2
c) El numerador de la expresión (m−n) 2 lo factorizamos según una diferencia de

cuadrados. El denominador lo escribimos como la multiplicación de las base,


fundado esto en la definición de potenciación. En el paso siguiente simplificamos. En
símbolos:

m2 − n2 (m + n)(m − n) m+n
2
= =
(m − n) (m − n)(m − n) m−n

Problema 86. Discutan en pequeños grupos, cómo podemos verificar la veracidad de una
simplificación de expresiones algebraicas. Utilicen el o los métodos propuestos para
verificar los problemas anteriores.

Problema 87. Hallen los números reales a y b tales que:


1 a b
a) = +
x2
−4 x+2 x−2
2x − 7 a b
b) 2
= +
x−x x x−1
x+7 a b
c) 2 = +
x − 8x + 7 x−1 x−7
x2 c
Problema 88. Escriban la expresión como una suma de la forma ax + b +
x+5 x+5
en donde a, b y c son números reales.

2.7. Nociones de geometría y trigonometría

A continuación exponemos algunas definiciones, conceptos, fórmulas y propiedades que


atañen a los conceptos básicos de la geometría plana. Sugerimos tener a mano estos
conceptos en el momento de la resolución de los problemas que contienen algunos de estos
temas.
Definición. Denominamos polígono a la figura plana y cerrada, delimitada por seg-
mentos que son sus lados. Además un polígono es convexo si todos sus ángulos interio-
res tienen amplitud menor a 180o . Si alguno de sus ángulos interiores tiene amplitud
mayor a 180o entonces decimos que es cóncavo.

Triángulos

Algunas definiciones

Triángulo equilátero: es aquel que tiene sus tres lados iguales.


Triángulo isósceles: es aquel que tiene dos lados iguales. Nota: todos los triángulos
equiláteros son isósceles.
Triángulo escaleno: es aquel que tiene los tres lados distintos entre sí.
Triángulo acutángulo: es aquel que tiene todos sus ángulos interiores agudos (es
decir, menores a 90o ).
Triángulo rectángulo: es aquel que tiene un ángulo interior recto (es decir, igual a
90o ).

35
Triángulo obtusángulo: es aquel que tiene uno de sus ángulos interiores obtuso (es
decir, mayor a 90o ).
Altura de un triángulo: es el segmento con un extremo en un vértice, el otro extremo
en el lado opuesto al vértice, y que es perpendicular a dicho lado. Nota: las tres
alturas de un triángulo se intersecan en un punto llamado ortocentro.
En un triángulo rectángulo denominamos catetos a los lados que forman el ángulo
recto, e hipotenusa al lado opuesto al ángulo recto.
El baricentro es la intersección de las medianas de un triángulo. Si es interior al
triángulo, es el punto de equilibrio del mismo.

En el gráfico de la izquierda observamos una de las alturas de un triángulo; el ángulo


α es recto. En el gráfico de la derecha observamos las tres alturas de un triángulo y
el ortocentro.

Fórmulas y propiedades de los triángulos

Desigualdad triangular: en todo triángulo abc se verifica18 que ab + bc ≥ ac.


Perímetro de un triángulo: es la suma de la medida de sus tres lados.
Área de un triángulo: es la medida de la superficie de un triángulo. La podemos
calcular como la mitad del producto entre una base y la altura correspondiente,
aunque existen otras formas equivalentes.
b·h
Área(Triángulo)= 2

Si a y b son dos lados del triángulo, y α es la medida del ángulo comprendido entre
a y b entonces19 :
a·b·sen(α)
Área(Triángulo)= 2

Si a, b y c son las medidas de los tres lados del triángulo, y S = a+b+c


2 entonces20
p
Área(Triángulo)= S(S − a)(S − b)(S − c)
La suma de los ángulos interiores de un triángulo es igual a 180o .
Teorema de Pitágoras: En todo triángulo rectángulo, de catetos A y B, e hipotenusa
H, se verifica que A2 + B 2 = H 2 .

Los lados A y B son los catetos y el lado H es la hipotenusa.


18
Si ab + bc = ac decimos que el triángulo degenera en un segmento; una de sus alturas es igual a cero,
y por lo tanto su área es nula.
19
Más adelante explicamos el significado de sen(α).
20
Esta fórmula se conoce como Fórmula de Herón.

36
En todo triángulo isósceles, una altura interseca a la base en el punto medio, y es
bisectriz del ángulo opuesto (es decir que lo divide en dos ángulos iguales).

Si ab = bc entonces ab̂m = cb̂m y el pie de la altura m es además punto medio de ac.

Cuadriláteros

Un cuadrilátero es un polígono con cuatro lados.

Definiciones

Un trapecio es un cuadrilátero con un par de lados paralelos.


Un trapecio isósceles es un cuadrilátero con un par de lados paralelos y otro par de
lados congruentes.
Un paralelogramo es un cuadrilátero con dos pares de lados paralelos.
Un rombo es un cuadrilátero con los cuatro lados iguales.
Un rectángulo es un cuadrilátero con los cuatro ángulos rectos (Notamos que todo
rectángulo es también un paralelogramo).
Un cuadrado es un cuadrilátero con los cuatro lados iguales y los cuatro ángulos
rectos (es decir que es un rombo y es un rectángulo.)

Fórmulas y propiedades

El perímetro de un cuadrilátero es igual a la suma de las medidas de sus cuatro


lados.
El área de un trapecio es igual al semiproducto entre la suma de sus bases y su
altura, es decir:
(B+b)·h
Área(Trapecio)= 2

El área de un paralelogramo es igual al producto entre su base y su altura, es decir:


Área(Paralelogramo)= b · h
El área de un rombo es igual a la mitad del producto entre sus diagonales, es decir
1
Área(Rombo)= 2 ·d·D
Las diagona1es de un rombo son perpendiculares, se intersecan en su punto medio y
dividen a cada par de ángulos en dos ángulos iguales.

37
2.8. Polígonos

Definiciones y propiedades

Un polígono es regular si tiene todos sus lados de la misma longitud y todos sus
ángulos interiores de la misma amplitud.
La apotema de un polígono es el segmento con un extremo en el centro y el otro en
el punto medio de un lado.
El número de diagonales que pasan por un vértice de un polígono de n lados es n − 3.
El número de diagonales totales de un polígono de n lados es n(n−3)
2 .
La suma de los ángulos interiores de un polígono de n lados es 180o · (n − 2).
180o ·(n−2)
La amplitud de un ángulo interior de un polígono regular de n lados es n .
o
La amplitud del ángulo central de un polígono regular de n lados es α = 360
n .
El perímetro de un polígono regular de n lados de longitud L es n · L.
Perímetro×Apotema
El área de un polígono regular es 2 .

2.9. Circunferencia y círculo

Decimos que una circunferencia es el conjunto de todos los puntos del plano que
equidistan21 de un punto fijo llamado centro. A esa distancia la denominamos radio.
El círculo es el conjunto de todos los puntos que están a una distancia menor o igual al
radio, de un punto fijo llamado centro.

Definiciones y fórmulas

a) El diámetro es el segmento que tiene por extremos dos puntos de la circunferencia y


contiene al centro.
b) La longitud del diámetro es igual al doble de la longitud del radio.
c) La razón entre la longitud de la circunferencia y su diámetro es igual al número π,
cuyo valor aproximado es 3, 1415926536.
d ) La longitud de la circunferencia de radio r es igual a 2 · π · r.
e) El área de un círculo de radio r es igual a π · r2 .

Si querés profundizar las definiciones, propiedades y conceptos vistos, te sugerimos la


lectura de estos links.

https://es.wikipedia.org/wiki/Método_exhaustivo.
https://www.geoka.net/poligonos/poligonos_inscritos.html.
https://www.geoka.net/poligonos/poligonos_circunscritos.html.

2.10. Cuerpos

Mostraremos una pequeña parte de los cuerpos geométricos. Para un estudio más
exhaustivo, que comprenda otros cuerpos y propiedades pueden consultar
http://lya.fciencias.unam.mx/gfgf/ga20132/poliedros/arch8.pdf.
21
Están a igual distancia.

38
Prismas

Son cuerpos que tienen dos caras idénticas y paralelas llamadas bases y otras caras
perpendiculares a las bases, idénticas entre sí, llamadas caras laterales. A los lados de las
caras los llamamos aristas. A las aristas con extremos en las bases las llamamos alturas.
Entre los prismas podemos mencionar al paralelepípedo, que es el prisma de seis caras
rectangulares, al cilindro (cuyas bases son dos círculos, y cuya única cara lateral es un
rectángulo).
Volumen de un prisma: El volumen de los prismas es igual al producto entre el área
de la base y la medida de la altura. Es decir:

V = área base·h

Algunos casos particulares:

Volumen del paralelepípedo: a · b · h.


Volumen del cilindro: π · r2 · h.
Volumen del cubo: a3 .

Aclaración: denominamos a y b a las medidas de las aristas de la base, h a la altura y r


al radio.

Esfera

La esfera es el conjunto de todos los puntos del espacio que están a una distancia menor o
igual a un punto fijo llamado centro. A esa distancia la llamamos radio.

Volumen de la esfera: V = 34 πr3 .


Área de la superficie de la esfera: A = 4πr2 .

Pirámides

Son cuerpos que tienen una cara llamada base y otras caras, idénticas entre sí, llamadas
caras laterales. A los lados de las caras los llamamos aristas. A la distancia entre la base
y el punto de intersección entre todas las caras laterales la llamamos altura. Entre las
pirámides podemos mencionar al tetraedro, que es el prisma de cuatro caras triangulares
y al cono (cuya base es un círculo, y cuya única cara lateral es una porción de círculo).
Volumen de la pirámide. El volumen de una pirámide es la tercera parte del volumen
del prisma que tiene la misma base y altura; es decir

V = 31 · área base·h

2.11. Razones trigonométricas

En esta sección mostraremos las definiciones y las propiedades de las razones


trigonométricas en un triángulo rectángulo.

Definiciones

Definimos las razones trigonométricas en el triángulo rectángulo abc con b̂ = 90o :

39
cateto opuesto bc
sen(α) = =
hipotenusa ac

cateto adyacente ab
cos(α) = =
hipotenusa ac

cateto opuesto bc
tan(α) = =
cateto adyacente ab

Nota: en la notación sen(α) el ángulo α es el argumento de la función seno; no es una


multiplicación.

Algunos comentarios:

Cuando decimos cateto opuesto o cateto adyacente, en trigonometría, es con respecto


a un ángulo interior del triángulo rectángulo.
Las razones trigonométricas expresan relaciones entre los lados del triángulo
rectángulo.
Cuando calculamos sen(α), o cualquier otra razón trigonométrica, obtenemos por
resultado un número (carente de unidad), ya que las unidades del numerador y el
denominador de la razón se cancelan entre sí.

La circunferencia trigonométrica

La circunferencia trigonométrica es aquella cuyo centro es el origen de coordenadas y el


radio es igual a 1.

Notamos que (dado que para cualquier ángulo α la hipotenusa mide 1) la medida del
segmento bc es igual a sen(α) y la medida del segmento ab es igual a cos(α).
Cada una de las razones trigonométricas tiene un signo según el cuadrante en el que se
encuentre el lado del ángulo α.

Cuadrante I II III IV
sen(α) Positivo Positivo Negativo Negativo
cos(α) Positivo Negativo Negativo Positivo
tan(α) Positivo Negativo Positivo Negativo

40
Propiedades de los ángulos complementarios y suplementarios.

Decimos que dos ángulos son complementarios si su suma es igual a 90o y dos ángulos
son suplementarios si su suma es igual a 180o . Se verifican las siguientes identidades para
cualquier valor de α:

a) sen(90o − α) = cos(α) c) sen(180o − α) = sen(α)


b) cos(90o − α) = sen(α) d ) cos(180o − α) = −cos(α)

Otras razones

Existen otras tres razones (que son las razones inversas multiplicativas, también llamadas
recíprocas) que definimos:

hipotenusa ac 1
cosec(α) = = =
cateto opuesto bc sen(α)

hipotenusa ac 1
sec(α) = = =
cateto adyacente ab cos(α)
cateto adyacente ab 1 cos(α)
cotan(α) = = = =
cateto opuesto bc tan(α) sen(α)

Identidades trigonométricas

Existen algunas identidades (equivalencias) entre las razones trigonométricas que se


pueden demostrar a partir de las definiciones. Presentamos algunas que pueden resultar
de gran utilidad. En algunas de ellas usamos22 la notación sen2 α que equivale a escribir
(senα)2 . Si quieren contar con más identidades trigonométricas, pueden consultar en
https://www.vitutor.com/al/trigo/trigo_1.html o en
https://campus.unaj.edu.ar/mod/book/view.php?id=21208.
sen(α)
a) cos(α) = tan(α) e) sen(2α) = 2sen(α)cos(α)
cos(α) f ) cos(2α) = cos2 (α) − sen2 (α)
b) sen(α) = cotan(α)
1+cos(2α)
c) sen2 (α) + cos2 (α) =1 g) cos2 (α) = 2
1−cos(2α)
d ) sec2 (α) + 1 = tan2 (α) h) sen2 (α) = 2

En http://www.xente.mundo-r.com/ilarrosa/GeoGebra/index_trigonometria.html
podés encontrar una interesante aplicación del GeoGebra a las razones y funciones
trigonométricas.

Problema 89. Hallen la medida de todos los lados y los ángulos desconocidos en los
siguientes triángulos.
22
Equivalentemente para todas las razones.

41
Problema 90. Calculen la medida de la base de un rectángulo sabiendo que las
diagonales se cortan formando un ángulo de 74o y que el lado opuesto a dicho ángulo
mide 10 centímetros.
Problema 91. Debido a un accidente en una industria química se tuvieron que desalojar
todas las casas que estuvieran dentro de un radio de 402 m de la fábrica. Un hombre vive
a 200 m Este y 286 m Sur de la misma. ¿Tuvo que desalojar?
Problema 92. En una torre de 45m que está ubicada sobre un peñasco de 50m de alto
junto a una laguna se encuentra un observador; éste divisa un objeto que cae en la laguna
verticalmente con un ángulo de depresión de 18o 300 . ¿A qué distancia de la orilla del
peñasco debe empezar a buscar el objeto?
Problema 93. Calculen la medida de los lados y los ángulos interiores de un rombo
sabiendo que las diagonales miden 8 cm y 6 cm.
Problema 94. La entrada de una mina se encuentra en un risco vertical a la cual se
accede por medio de una escalera de 12, 13m de largo, y con un ángulo de elevación de
52o 400 con el piso. Se desea saber cuánta cuerda se necesita para subir y bajar canastas
de la entrada verticalmente.
Problema 95. Desde un punto ubicado a 10 cm del centro de una circunferencia se
trazan dos tangentes a la misma. Si el radio de ésta es 4 cm, ¿qué ángulo forman las
tangentes? (Ayuda: La tangente a una circunferencia es perpendicular al radio en el
punto de tangencia).
Problema 96. Calculen la altura de la pirámide eo a partir de los datos de la figura. Las
caras de la pirámide son triángulos isósceles. Calculen la medida del ángulo β.

42
Problema 97. Hemos colocado dos cables y un mástil como muestra la figura. Calculen
la medida del mástil y la longitud de los cables.

Problema 98. ¿A qué distancia se encuentra el horizonte para una persona que mide
1,75 metros? (Dato: El radio de la Tierra es aproximadamente 6300 km).

Problema 99. Supongamos que nos encontramos a cierta distancia de un rascacielos y


deseamos medir su altura. ¿De qué manera podríamos hacerlo usando funciones
trigonométricas a partir de la medición de ángulos y distancias?

Problema 100. Un árbol de 50m de alto proyecta una sombra de 60m de largo.
Encuentren el ángulo de elevación del Sol en ese preciso instante.

Problema 101. Desde un dirigible que está volando a 800m de altura, se distingue un
pueblo con un ángulo de depresión de 12o . ¿A qué distancia se halla del pueblo?

Problema 102. Un barco B pide socorro y dos estaciones de radio (A y C) reciben su


señal. La distancia entre las estaciones de radio es de 50km. Desde ellas se miden los
siguientes ángulos: BAC = 46o y BCA = 53o . ¿A qué distancia de cada estación se
encuentra el barco?

Problema 103. Calculen el área de una parcela triangular, sabiendo que dos de sus lados
miden 80m y 130m, y forman entre ellos un ángulo de 70o .

Definición. Denominamos pendiente de una recta a la tangente del ángulo que forma
ésta con la horizontal.
Problema 104. Una señal de peligro en una carretera nos advierte que la inclinación
«cuesta abajo» es de 12o . ¿Cuál es la pendiente en ese tramo de ruta? ¿Cuántos metros
hemos descendido luego de recorrer 7km por dicha ruta?

Teoremas del seno y del coseno

Teorema. Sean α, β y γ los ángulos opuestos a los lados A, B y C en un triángulo,


entonces se verifican las siguientes igualdades:
Teorema del seno
A B C
= =
sen(α) sen(β) sen(γ)
Teorema del coseno
A2 = B 2 + C 2 − 2BC · cos(α)

Problema 105. Calculen la suma de dos fuerzas concurrentes de magnitudes 4N y 6N


que forman un ángulo de 60o .

43
Resolución.
Planteamos un diagrama de análisis.

Trasladamos el vector v

Para calcular la fuerza resultante, calculamos la longitud del vector a. Aplicamos el


teorema del coseno:
1
|v + w|2 = |v|2 + |w|2 − 2|v||w|cos(120o ) = 16 + 36 + 2 · 4 · 6 · = 76
2
Luego √
|a| = |v + w| = 76
Comentario: Se puede también resolver utilizando el ángulo original de 60o . De esta
manera, tenemos |v + w|2 = |v|2 + |w|2 + 2|v||w|cos(60o ). Esta fórmula es habitual en el
ámbito de la física. La obtenemos a partir de la identidad cos(120o = −cos(60o ).
Para calcular el ángulo aplicamos el teorema del seno:

|a| |v|
o
=
sen(60 ) sen(α)

Despejamos el ángulo α y obtenemos α = 36o 350 .

Problema 106. Hallen la fuerza resultante de la suma de dos fuerzas concurrentes de


igual magnitud, con un ángulo de 120o .

44
Problema 107. Un avión vuela con una velocidad de 250 m s en sentido al Sur. En un
momento se desata una tormenta de viento con sentido Noreste, y una velocidad de 30 m s
que, además de asustar a muchos de los pasajeros, desvía el recorrido del avión. ¿Cuál es
la velocidad resultante del avión?23
Problema 108. Consideren el avión del problema anterior. ¿Qué distancia recorrió
después de una hora de vuelo en las circunstancias dadas? ¿A qué distancia se encuentra
después de una hora, con respecto al trayecto planificado24 ?
Problema 109. En una ruta de montaña, una señal indica una altitud de 785m. Tres
kilómetros más adelante, la altitud es de 1.265m. Hallen la pendiente media de esa ruta y
el ángulo que forma con la horizontal.
Problema 110. Sea ABCDE un pentágono regular de lado 15 cm, calculen:
La amplitud de cada ángulo interior de los triángulos ABE, BEC y EDC.
La medida de las diagonales BE, CE.
La altura de los triángulos ABC y CED.
Pueden encontrar más contenido de trigonometría en
https://campus.unaj.edu.ar/mod/book/view.php?id=21098.

2.12. Funciones trigonométricas

Las funciones trigonométricas son aquellas que para cada valor de x, el valor de f (x)
queda determinado según alguna relación trigonométrica (o composición entre ellas).

Sistema radial

Para poder representar gráficamente una función trigonométrica en el sistema de ejes


cartesiano, la variable independiente x debe ser una medida lineal, y no un ángulo.
Entonces utilizamos la relación en la cual, para cada ángulo, x es igual a la longitud del
arco que éste abarca en la circunferencia trigonométrica. La unidad de medida del ángulo
expresado en función de la longitud del arco es el radián25 . Al sistema de medición lo
denominamos radial26 .

23
Recuerden que la velocidad es una magnitud vectorial, a diferencia de la rapidez, que es una magnitud
unidimensional.
24
Es decir, si no hubiera habido viento.
25
Con tilde y n final.
26
Sin tilde y con l final.

45
Es sencillo notar que un ángulo de 360o abarca toda la circunferencia (cuya longitud es
igual a 2πr en donde r es el radio. Como asumimos que r = 1 entonces es válida la
relación
360o −→ 2π
a partir de la cual (aplicando conceptos de proporcionalidad) podemos obtener la medida
de cualquier ángulo en radianes. Por ejemplo:

Problema 111. Hallen, en radianes, la medida del ángulo recto.

Es importante estar atento al sistema que estamos usando, en el momento de usar la


calculadora, ya que debemos configurarla según queramos trabajar en sistema radial
R o en sistema sexagesimal D .

Nota: Cuando escribimos un ángulo en el sistema radial no escribimos la unidad; por


ejemplo 30o = π6 . Asimismo, si escribimos un número y no explicitamos la unidad,
entonces nos referimos a radianes.

Gráficas de algunas funciones

A continuación presentamos las gráficas de algunas funciones para que puedas analizar
sus raíces, su ordenada, sus extremos y los máximos locales.

Funciones inversas de las funciones trigonométricas

Muchas veces necesitamos conocer la amplitud de un ángulo conociendo únicamente las


medidas de los lados del triángulo rectángulo o nos presentan ecuaciones como
sen(x) = 21 , entonces para calcular la medida del ángulo x, desconocido aplicamos las
funciones inversas.
Función Función inversa Se lee...
sen arcsen Arco seno sen(x) = y → x = arcsen(y)
cos arccos Arco coseno cos(x) = y → x = arccos(y)
tan arctan Arco tangente tan(x) = y → x = arctan(y)

Resolvemos a modo de ejemplo la ecuación propuesta:


1
sen(x) =
2

46
 
1
x = arcsen
2
Una de las soluciones de la ecuación la hallamos con la calculadora científica. Entonces
obtenemos
x = 30o
Si necesitamos hallar el valor de una variable de una función lo expresamos en radianes;
entonces
π → 180o
x → 30o
Luego
π
x=
6
Observamos que ésta no es la única solución, ya que (por ejemplo) sen(150o ) = 12 . Para
hallar otras soluciones podemos aplicar las propiedades mencionadas anteriormente o
bien observar la gráfica de la función correspondiente.

Tabla de ángulos y funciones trigonométricas

En grados En radianes sen(α) cos(α) tan(α)


0o 0 0 √
1 √
0
π 1 3 3
30o 6 √2 √2 3
π 2 2
45o 4 √2 2 1
π 3 1

60o 3 2 2 3
π
90o 2 1 0 No existe

Problema 112. Grafiquen las funciones (pueden utilizar algún Software, por ejemplo, el
GeoGebra). Busquen características comunes y diferencias entre las funciones (máximos,
mínimos, períodos, amplitudes, raíces, ordenada). Organicen la información junto con las
gráficas. Esta tarea puede resultarles útil más adelante, por ejemplo, en el momento de
resolver ecuaciones trigonométricas, o problemas relacionados con ellas.

a) f (x) = sen(x) g) f (x) = 3sen(x)


b) f (x) = cos(x) h) f (x) = 4cos(x)
c) f (x) = −cos(x) i) g(t) = 2sen(2t)
d ) f (x) = −sen(x) j ) f (x) = 5sen(3x)
e) f (x) = tan(x) k ) f (x) = cos(4x)
f ) f (x) = 2sen(x) l ) f (θ) = 5 sin(2θ − π2 )

Sugerencia: Grafiquen las funciones dadas en el intervalo x ∈ [−4π; 4π] (en radianes).

Problema 113. Utilicen la calculadora para verificar con algunos ejemplos las siguientes
propiedades:

cos(α ± β) = cos(α)cos(β) ∓ sen(α)sen(β) sen(α ± β) = sen(α)cos(β) ± cos(α)sen(β)

Problema 114. Teniendo en cuenta las propiedades anteriores, calculen:


π

a) sen(α + 2π) c) cos α − 2
+ π2

b) cos(α + 2π) d ) sen α

47
Problema 115. Considerando las propiedades anteriores, demuestren que:
1−cos(2α)
a) sen(2α) = 2sen(α)cos(α) c) sen2 (α) = 2
1+cos(2α)
b) cos(2α) = 1 − 2sen2 (α) d ) cos2 (α) = 2

Problema 116. Resuelvan las siguientes ecuaciones. Determinen todas las soluciones para
x ∈ [−2π; 2π].

a) sen(x) = 0 f ) cos(x) = 0, 5
b) cos(x) = 0 g) cos(x) = −0, 5
c) sen(x) = 1 h) sen(x) = 0, 5
d ) cos(x) = 1 i) sen(x) = −0, 5
e) sen(x) = −1 j ) sen(x) − cos(x) = 0

2.13. Lógica proposicional

En esta sección mostramos algunas leyes lógicas. Sugerimos complementar el trabajo con
la lectura del siguiente paper :

https://www.cs.buap.mx/~fjrobles/LogPro.pdf.

Definición. Una proposición es una frase que es verdadera o falsa.


Toda proposición está asociada a un valor de verdad que es verdadero, o bien es
falso.
Problema 117. Determinen y justifiquen cuáles de las siguientes frases son proposiciones.

a) ¿Quién viene?
b) Mi nombre es Gonzalo.
c) ¡Deténgase!
d ) El calor dilata los cuerpos.
e) Cuatro es un número impar.
f) x+1 = 5
g) Existen más de 5 trillones de estrellas.

Las proposiciones genéricas las denotamos con las letras p, q, r, · · · . A partir de


proposiciones simples (irreducibles) podemos generar otras proposiciones, a través de los
conectivos lógicos. Algunos de ellos son:

La negación: La simbolizamos ¬. Entonces ¬p indica "...no es cierto que p...".


La conjunción: La simbolizamos ∧. Entonces p ∧ p indica "...p y q...".
La disyunción o disjunción: La simbolizamos ∨. Entonces p ∨ q indica "...o bien p o
bien q...".
La implicación: La simbolizamos ⇒. Entonces p ⇒ q indica "...Si p entonces q ...".
La doble implicación. La simbolizamos ⇔. Entonces p ⇔ q indica "...p si y sólo si
q...".

48
Las tablas de verdad

Problema 118. Dadas las siguientes proposiciones:

p: Mis maestros hacen que todas las lecciones sean aburridas.


q: No aceptan las respuestas que no figuran en los libros.
r: Imponen una gran cantidad de normas tontas.

Construyan las proposiciones:

a) p ∧ q b) ¬q ∨ r c) (p ∧ q) ⇒ r

Problema 119. Construyan las tablas de verdad de cada una de las proposiciones del
problema anterior.

Problema 120. Consideramos v(p) = V ; v(q) = F . Indiquen los valores de verdad de cada
una de las siguientes proposiciones.

a) (p ∧ q) ∨ q c) (p ⇒ q) ∨ ¬p
b) ¬(p ∨ q) ∧ p d ) ¬(p ⇔ q)

Definiciones.

Un razonamiento deductivo es una proposición de la forma


(A1 ∧ A2 ∧ · · · ∧ An ) ⇒ C en donde Aj son proposiciones (a las que llamamos
premisas) y C es una proposición a la que denominamos conclusión.
Un razonamiento deductivo es válido cuando la proposición (A1 ∧ A2 ∧ · · · ∧ An ) ⇒ C
es verdadera (es decir, cuando la implicación es una tautología). Si alguna de las
premisas es falsa, resulta inmediato notar que el razonamiento es válido.
Una contradicción es una proposición compuesta que resulta siempre falsa,
independientemente de los valores de verdad de las proposiciones simples.
Una contingencia es una proposición compuesta cuyo valor de verdad puede ser
verdadero o bien falso, dependiendo de los valores de verdad de las proposiciones
simples.
Una tautología es una proposición compuesta que resulta siempre verdadera,
independientemente de los valores de verdad de las proposiciones simples. Las
siguientes son algunas tautologías, conocidas como leyes lógicas:

a) Ley involutiva. ¬(¬p) ⇔ p


b) Leyes conmutativas. (p ∧ q) ⇔ (q ∧ p)
(p ∨ q) ⇔ (q ∨ p)
c) Leyes asociativas. [(p ∧ q) ∧ r] ⇔ [p ∧ (q ∧ r)]
[(p ∨ q) ∨ r] ⇔ [p ∨ (q ∨ r)]
d ) Leyes de idempotencia. (p ∧ p) ⇔ p
(p ∨ p) ⇔ p

49
e) Leyes de simplificación y ampliación. (p ∧ q) ⇒ p
p ⇒ (p ∨ q)
f ) Modus Ponens. [(p ⇒ q) ∧ p] ⇒ q
g) Modus Tollens. [(p ⇒ q) ∧ (¬q)] ⇒ (¬p)
h) Silogismo hipotético puro. [(p ⇒ q) ∧ (q ⇒ r)] ⇒ (p ⇒ r)
i) Ley de la contrarrecíproca. [(p ⇒ q) ⇔ [(¬q) ⇒ (¬p)]
j ) Leyes de absorción. [p ∧ (p ∨ q)] ⇔ p
[p ∨ (p ∧ q)] ⇔ p
k ) Leyes de De Morgan. [¬(p ∧ q)] ⇔ [(¬p) ∨ (¬q)]
[¬(p ∨ q)] ⇔ [(¬p) ∧ (¬q)]
l ) Principio de contradicción. [p ∧ (¬p)] ⇔ F
m) Principio del tercero excluido. [p ∨ (¬p)] ⇔ V

Problema 121. Propongan varios ejemplos de contradicciones.

Problema 122. Propongan varios ejemplos de contingencias.

2.14. Los conjuntos

Los conjuntos son objetos matemáticos de uso frecuente en diversos temas que
abordaremos durante el curso. Por lo tanto consideramos conveniente exponer
brevemente algunas ideas de la Teoría de Conjuntos (las definiciones, la notación
simbólica y algunos ejemplos).
Si bien el término Conjunto es un término primitivo en la Teoría de conjuntos (y por lo
tanto no se define) entendemos por conjunto a una colección o reunión de individuos, a
los que llamaremos elementos. Por ejemplo:

Los colores de la bandera argentina.


Los nombres de las provincias de la República Argentina.
El conjunto formado por los dígitos del número 658889859.
El conjunto de todas las estrellas del espacio.
El conjunto de los números naturales.

Nombramos a los conjuntos con letras mayúsculas. A los elementos (que los designamos
con letras minúsculas) los podemos mencionar de dos maneras: por enumeración y por
comprensión. Por ejemplo, si denominamos n al número 658889859 y A al conjunto de los
dígitos de n entonces el conjunto A enunciado por comprensión es:

A = {Los dígitos del número n}

Y enunciado por enumeración es:

A = {5; 6; 8; 9}

Representación gráfica de los conjuntos. Podemos representar uno o más conjuntos


mediante diagramas de Venn27 . Por ejemplo:
27
Venn, John: Matemático y Lógico británico (1824-1923).

50
Observamos en el ejemplo, que 5 pertenece al conjunto A. En símbolos: 5 ∈ A.
Notamos también que 6 no pertenece al conjunto A. En símbolos: 6 ∈/ A.

Inclusiones, igualdad y cardinales de los conjuntos.

El conjunto P está incluido en el conjunto Q si todos los elementos de P pertenecen


a Q. En símbolos: P ⊂ Q.
Además escribimos P ⊆ Q si P está incluido o es igual a Q.
En cualquiera de los casos, decimos que P es un subconjunto de Q.
Si P ⊆ Q y Q ⊆ P entonces P = Q.
Denominamos cardinal del conjunto A al número28 de elementos que tiene A. Al
cardinal del conjunto A lo simbolizamos #(A).
Dos conjuntos son coordinables si su cardinal es el mismo.
Conjunto finito. Es un conjunto tal que su cardinal es un número natural o cero.
Conjunto infinito. Es un conjunto que no es finito.
Conjunto vacío. Es el conjunto cuyo cardinal es cero (es decir, que no tiene
elementos). El conjunto vacío está incluido en todos los conjuntos.
Conjunto unitario. Es un conjunto cuyo cardinal es uno.
Conjunto de partes. El conjunto de las partes de un conjunto A es el conjunto
formado por todos los subconjuntos de A. Lo denotamos P (A). Por ejemplo:

Si A = {1; 2} −→ P (A) = {∅; {1}; {2}; {1; 2}}

Si el cardinal de A es n entonces el cardinal de P (A) es 2n .

Operaciones entre conjuntos

Unión: La unión entre dos conjuntos A y B es el conjunto C formado por todos los
elementos que pertenecen indistintamente a A o a B. En símbolos:

A ∪ B = C ⇔ C = {x : x ∈ A ∨ x ∈ B}

Intersección: La intersección entre dos conjuntos A y B es el conjunto C formado


por todos los elementos que pertenecen simultáneamente a A y a B. En símbolos:

A ∩ B = C ⇔ C = {x : x ∈ A ∧ x ∈ B}

Complemento: El complemento del conjunto A es el conjunto de todos los


elementos que no pertenecen a A. En símbolos:

A = {x : x 6∈ A}
28
En el caso de que A sea finito. Si A es infinito, el estudio de su cardinal requiere un análisis más
profundo, que corresponde a otra área de la matemática, denominada aritmética transfinita.

51
Diferencia: La diferencia entre dos conjuntos A y B es el conjunto C formado por
todos los elementos que pertenecen a A pero no a B. En símbolos:

A − B = C ⇔ C = {x : x ∈ A ∧ x 6∈ B}

Notamos que la diferencia de conjuntos A − B también la podemos expresar como


A ∩ B.
Diferencia simétrica: La diferencia simétrica entre dos conjuntos A y B es el
conjunto C formado por todos los elementos que pertenecen a A pero no a B y por
todos los elementos que pertenecen a B pero no a A. En símbolos:

A∆B = C ⇔ C = {x : (x ∈ A ∧ x 6∈ B) ∨ (x ∈ B ∧ x 6∈ A)}

Notamos que la diferencia simétrica la podemos expresar de varias formas. Por


ejemplo:
A∆B = (A ∪ B) − (A ∩ B) = (A − B) ∪ (B − A)

Producto cartesiano: El producto cartesiano entre dos conjuntos A y B es un


conjunto C tal que todos los elementos de C son pares ordenados, el primer
elemento de cada uno de los pares es un elemento de A y el segundo elemento de
cada par ordenado es un elemento de B. En símbolos:

A × B = C ⇔ C = {(a; b) ∧ a ∈ A ∧ b ∈ B}

Problema 123. Una consultora hace una encuesta a un grupo de alumnos. A cada uno se
le pregunta qué materias aprueba. En función de la información, determinen cuántas
personas respondieron las encuestas.

30 personas aprueban Matemática. 15 aprueban Matemática y Química.


35 personas aprueban Física.
20 aprueban Física y Química.
100 personas aprueban Química.
15 aprueban Matemática y Física. 5 aprueban las tres asignaturas.

Problema 124. En una universidad se anotaron 200 estudiantes, entre los cuales 98 son
mujeres. De los estudiantes, 60 se anotaron en economía y sabemos que 60 mujeres no se
anotaron en economía. ¿Cuántos hombres no estudian economía en esa universidad?

Problema 125. La misma consultora mencionada anteriormente realizó una encuesta a


135 personas para establecer preferencias de lectura de los libros A, B y C; obtuvieron los
siguientes resultados:

135 personas leen alguno de las 3 libros.


95 personas leen A.
15 personas leen A y B pero no C.
6 personas leen B y C pero no A.
10 personas leen sólo C.
La cantidad de personas que lee A y C (pero no B) es el doble de la cantidad de
personas que lee los 3 libros.
La cantidad de personas que lee sólo B es la misma cantidad de personas que lee A
y C (pero no B).

Hallen el número de personas que lee solamente A.

52
2.15. Cotas y extremos de los conjuntos

Sea A un conjunto con una relación de orden ≤ y sea a ∈ A. Decimos que a es elemento
máximo o último elemento de A, si x ≤ a para todo x ∈ A; y decimos que a es elemento
mínimo o primer elemento de A si a ≤ x para todo x ∈ A.

Teorema. Sea A un conjunto con una relación de orden ≤. El máximo (ó mínimo), si


existe, es único.

Sea A un conjunto con una relación de orden ≤ y sea B ⊂ A, decimos que a es una cota
superior de B, si x ≤ a para todo x ∈ B y decimos que a es una cota inferior de B, si
a ≤ x para todo x ∈ B.
Sea A un conjunto con una relación de orden ≤, y sea B ⊂ A. Si B posee alguna cota
superior (ó inferior), decimos que B está acotado superiormente (o inferiormente). Si B
está acotado superior e inferiormente, decimos que B está acotado.
Sea A un conjunto con una relación de orden ≤ y sea B ⊂ A. Decimos que e ∈ A es
extremo superior o supremo de B, si y sólo si:

e es cota superior de B.
Si a es cota superior de B, entonces e ≤ a. Es decir, el extremo superior de un
conjunto es la menor de las cotas superiores.

Análogamente, decimos que A es extremo inferior o ínfimo de B, si y sólo si:

e es cota inferior de B.
Si a es cota inferior de B, entonces a ≤ e. Es decir, el extremo inferior de un
conjunto es la mayor de las cotas inferiores.

Teorema: El supremo (o ínfimo) de un conjunto, si existe, es único.

Problema 126. (Optativo) Indiquen el valor de verdad de cada una de las siguientes
proposiciones.

a) k es una cota superior de un conjunto A sí y sólo si k es un número real que no es


superado por cualquier elemento de A.
b) Un conjunto no está acotado superiormente si tiene cota superior.
c) El supremo de un conjunto es la mayor de las cotas superiores.
d ) 0 es cota superior para R−
e) El extremo superior, es decir, el ínfimo es único.
f ) Si el supremo pertenece al conjunto A entonces es el máximo de A.
g) h es una cota inferior de un conjunto A de números reales sí y sólo si h es un
número real que no supera a cualquiera de los elementos del conjunto A.
h) t es cota inferior de A ⇔ ∀x ∈ A : x ≥ h
i) Un conjunto es acotada inferiormente si tiene por lo menos una cota inferior.
j ) El ínfimo es la mayor de las cotas inferiores.
k ) Si el ínfimo pertenece a un conjunto, entonces es elemento mínimo.

53
Problema 127. Consideren el conjunto A = {x ∈ R/x < 4}.

a) Escriban este conjunto como intervalo real.


b) Representen gráficamente este intervalo en un la recta real.
c) Indiquen el supremos de dicho conjunto.
d ) Justifiquen por qué no tiene elemento máximo.
e) Propongan por lo menos tres cotas superiores del conjunto.

Problema 128. Respondan las preguntas del problema anterior considerando el conjunto
A = {x ∈ R/x ≤ 4}.

Problema 129. Dado el conjunto B = {x ∈ R/x ≥ 31 }.

a) Escriban este conjunto como intervalo real.


b) Representen gráficamente este intervalo en un la recta real.
c) Indiquen el ínfimo de dicho conjunto.
d ) Justifiquen por qué tiene elemento mínimo.
e) Propongan por lo menos dos cotas inferiores del conjunto.

Definición. Llamamos entorno de centro c y radio r al intervalo abierto Ec;r = (c −


r; c + r). Equivalentemente: Ec;r = {x ∈ R : |x − c| < r}.
Un entorno reducido de centro c y radio r es el conjunto de todos los puntos del
0 = {x ∈ R : 0 < |x − c| < r}.
entorno, excepto el punto c. Equivalentemente: Ec;r

Definición. Si A es un conjunto de puntos de la recta real, un punto A es un punto de


acumulación de A si y sólo si a todo entorno reducido de centro a pertenece por lo
menos un elemento de A.

54
Respuestas a los problemas del capítulo I

1 - Resolución. Descomponemos los números dados:

72 = 23 · 32 ; 60 = 22 · 3 · 5

Entonces
mcd(60; 72) = 22 · 3 = 12
2 - Resolución. El resto de dividir 2550 por 726 es igual a 372. Entonces
mcd(2550; 726) = (726; 372). Para calcular el segundo mcd razonamos de la misma forma;
entonces calculamos el resto de dividir 726 por 372, que es igual a 354. Siguiendo con este
razonamiento:

mcd(2550; 726) = mcd(726; 372) = mcd(372; 354) = mcd(354; 18) = mcd(18; 6) = 6

Por lo tanto, mcd(2550; 726) = 6.


Este algoritmo (conocido como Algoritmo de Euclides) es particularmente práctico para
hallar el mcd entre números grandes.
3 - Resolución. Dado que

72 = 23 · 32 ; 60 = 22 · 3 · 5

entonces mcm(60; 72) = 23 · 32 · 5 = 360


5 - Respuestas.
2 9 7·2+3·9
a) 3 + 7 = 21 = 41
21 d) 7 75
20 − 24 =
6·7−5·75
120 = − 333
120 = − 111
40
3 1 4·3+5·1 17 1 1 1 6+4−3 7
b) 5 + 4 = 20 = 20 e) 2 + 3 − 4 = 12 = 12
2 4 7·2+5·4 34 2 2−13 11
c) 15 + 21 = 105 = 105 f) 13 − 1 = 13 = − 13

7 - Respuestas.
2 9 18 6 3 1 3 12
a) 3 · 7 = 21 = 7 g) 5 : 4 = 5 · 4= 5
3 1 3
b) 5 · 4 = 20
2 4 2 21 1 21 1 7 7
h) 15 : 21 = 15 · 4 = 15 · 2 = 5 · 2 = 10
2 21 1 21 1 7 7
c) 15 · 4 = 15 · 2 = 5 · 2 = 10 7 5 7 24 7 12 7 6 42
5 15 75 i) 20 : 24 = 20 · 5 = 10 · 5 = 5 · 5 = 25
d) 4 · 2 = 8
1 1 1 3
6 5 30 j) : = ·3=
e) 5 · 3 = 15 = 2
2 3 2 2
2 9 2 7 14 3 3 3
f) 3 : 7 = 3 · 9 = 27 k) 5 · 25 = 1· 5 = 5

10 - Respuestas.
( 32 + 97 )· 32 2 3
· + 79 · 32 1+ 27 41
41
a) 1
− 35
= 3 2
5−6 = 14
1
− 10
= 14
1
− 10
= 14 · (−10) = − 410 205
14 = − 7
2 10
3
−8 1 − 53 − 53
− 34 = − 34 = − 53 4
· ( − 34 ) = 53 1 159
  
b) 5(
7
1
+ 25 4 −1 = 7
5+8 13
7
· 7 · 13 7 · 13 ·3= 91
4 ) 5( 20 ) 4

11 - Respuestas.

a) ( 32 )0 = 1 d) 1
7 = ( 72 )−1
2

b) ( 27 )12 : ( 72 )9 = ( 27 )12−9 = ( 27 )3 e) ( 72 )−1 = 1


7
2

c) ( 35 )17 · ( 53 )−15 = ( 35 )17−15 = ( 35 )2 f ) ( 27 )14 · ( 72 )−16 = ( 72 )14−16 = ( 27 )−2

15 - Respuestas.

55
a b c c · (a + b) (a + b)2 a2 + b2 (a + b)3 a3 + b3
2 3 5 25 25 13 125 35
0 -2 5 -10 4 4 -8 -8
-1 0 4 -4 1 1 -1 -1
4 -2 3 6 4 20 8 56
-9 -1 7 -70 100 82 -1000 -730
-5 -7 -3 36 144 74 -1728 -468
0 0 4 0 0 0 0 0

16 - Las propiedades aplicadas quedan a cargo del estudiante. La expresión dada es:
2 1 1 1 1 1 1
27− 3 = 2 = √ = √ = √ √ = =
27 3
3
272
3
27 · 27 3 3
27 · 27 3·3 9

18 - Una posible solución es:


1
a− 1b 1 1 1 b−a 1 (a − b) 1 1
=( − )· = · =− · =−
a−b a b a−b ab a−b ab (a − b) ab

19 - Una posible solución es:

1 1 a a−1 b
a = = ( )−1 = −1 =
b ( ab )1 b b a

20 - Es verdadero: 10n + 3(3n + 5) + 23 = 19n + 38 = 19(n + 2).


21 - Es falso. Por ejemplo, si n = 17 entonces la fórmula genera el número 172 y ningún
cuadrado de un número natural es primo.√
22 - Resolución. Supongamos29 que 2 es un número racional, es decir que existen dos
números coprimos a y b (con b 6= 0) tales que
√ a
2=
b
Elevamos ambos miembros al cuadrado y multiplicamos por b2 .

a2
2=
b2
2b2 = a2 (∗)
Entonces a2 es un número par, y consecuentemente a es un número par. Por ello,
podemos escribir a = 2k con k ∈ Z. Reemplazamos esta expresión en (∗).

2b2 = (2k)2

Aplicamos la propiedad distributiva de la potenciación con respecto a la multiplicación.

2b2 = 22 · k 2 = 4k 2 −→ b2 = 2k 2

Es decir que b2 es par; por lo tanto b es par.


√ embargo, si a y b son pares, entonces no son coprimos. Es
Sin √ decir que, si suponemos que
2 es racional, llegamos a una contradicción. Por lo tanto, 2 no es un número racional.
23 - Resolución.Observamos que a = 236 · 100000000000. Y el primer factor tiene que
ser un número entre 1 y 10. Entonces

a = 2, 36 · 100 · 1011
29
El método que usamos en esta demostración se cononce como Método de reducción al absurdo. Está
basado en la Ley del contrarrecíproco. Consiste en suponer que la tesis es falsa y concluir una contradicción.

56
Entonces, según las propiedades de las potencias de igual base resulta

a = 2, 36 · 102 · 1011 = 2, 36 · 1013

Luego, a = 2, 36 · 1013 .
Análogamente notamos que
17 1
b= = 1, 7 ·
100000000000 10000000000
Entonces aplicamos las propiedades de las potencias de exponente negativo y resulta:

b = 1, 7 · 10−11

24 - Respuestas. a = 2, 36 · 1013 ; b = 1, 7 · 10−11 → a · b = 2, 36 · 1013 · 1,7 · 10−11 =


2,36 · 1,7 · 1013−11 = 2,36 · 1,7 · 102 = 4,01 · 102 .
25 - Respuestas.
2,36·1013 2,36 13−(11) = 2,36 · 1024 = 1, 39 · 1024 .
a = 2, 36 · 1013 ; b = 1, 7 · 10−11 → ab = 1,7·10 −11 = 1,7 · 10 1,7
28 - Respuestas.

Magnitud Valor
Volumen de la tierra, en m3 1, 08 · 1021
Distancia a la estrella más cercana, en km 4, 07 · 1013
Edad del universo, en años 15 ∗ 109
Duración de la época de Planck, en segundos 10−42
Peso de 1cm3 de una estrella neutrónica, en kg 1015
Peso de una ameba, en gramos 5 · 10−6
Radio de una célula humana, en m 5 · 10−8
Velocidad de la luz, en km/h 10,8 · 106

29 - Respuestas. M asa del sol = ms = 1, 989 · 1030 kg


M asa de α Centauro = 18 · ms = 18 · 1, 989 · 1030 kg = 35,8 · 1030 kg = 3, 58 · 1031 kg.
30 - Respuestas. 365 dia · 24 hora min s 7
dias · 60 hora · 60 min = 31536000 s = 3, 15 · 10 s.
31 - Respuestas.
E = mc2 ⇒ E = 3 · 1013 kg · (3 · 108 m 2 13 16 m2 = 27 · 1029 kg · m2 ⇒
s ) = 3 · 10 kg · 9 · 10 s2 s2
E = 2,7 · 1030 J
32 - El largo es 37 centímetros y el ancho es 17 centímetros.
33 - 85 kilómetros.
34 - 10769,57 pesos.
35 - Carmen vendío 42 cajas y Sonia 66 cajas.
36 - v = 55 km
h .
37 - 16 preguntas.
38 - El dominio es Df = (0, 17; 122, 83). La máxima tensión es 18806,25 y se alcanza a los
61,5 segundos.
39 - La solución es el par (20; 30). La gráfica la pueden verificar con el GeoGebra.
40 - Respuestas.
52000
a) $ 9300. d ) A cargo del alumno. g) f (x)200x + x
b) $ 7200 e) C(x) = 200x + 130z h) x = 16, 12 m;
c) $ 6600. f ) A cargo del alumno. z = 24, 81. m

41 - Respuestas.

57
a) x · y = 891 e) (27; 33), (33; 27)
b) 2x + 2y = 120 √
f ) D = 3 202
c) y = 891
x ; y = 60 − x
d) A cargo del alumno. g) α = 50o 430 ; β = 39o 170

42 - Respuestas.
q
a) x = −7 e) x=0 i) x = −1 l ) v = ± 2k
m
6
b) y=2 f) x = 19
8 j ) − 25 m) x1 = 0; x2 = 3
c) t = 17 g) x = 25
r·p
d) t = − 29 h) x = 11
5 k) x = r−2p

43 - La única solución es el par de números x = 125 750


7 ;y = 7 .
5
44 - Es el número − 2 .
45 - Tiene 22 escalones.
46 - Es el número 16.
47 - Se trata del número 10.
48 - La base mide 36 centímetros y la altura mide 12 centímetros.
49 - Respuestas. b = 11 cm; L1 = L2 = 22 cm.
50 - Tiene 14 años.
51 - Respuestas.

a) x1 = −3; x2 = 9 d) x1 = 51 ; x2 = − 23 h) x1 = 7; x2 = 3
e) x1 = 4; x2 = −2
b) x1 = − 73 ; x2 = 2
3 i) x = 9

f) x1 = 2; x2 = − 13
−9± 137
c) x = 2 g) x=9 j) x = 5

52 - Respuestas.

a) No tiene solución en los reales.


b) t1 = 0; t2 = 5; −t(t − 5) = 0
c) t1 = 0; t2 = 74 ; 4t t − 74 = 0


d ) No tiene solución en los reales.


e) x1 = 3; x2 = −2; 3(x − 3)(x + 2) = 0
f ) x1 = 5; x2 = −1; 5(x − 5)(x + 1) = 0
g) x1 = −3; x2 = −5; 25 (x + 3)(x + 5) = 0
h) x1 = 6; x2 = 1; −2(x − 6)(x − 1) = 0
i) x1 = 3; x2 = −2; −3(x − 3)(x + 2) = 0
√ √ √
j ) x = 20 ± 5 19; − 21 (x − 20 + 5 19)(x − 20 − 5 19) = 0
k ) x1 = x2 = −4; −2(x + 4)2 = 0
l ) x1 = 2; x2 = −1; (x − 2)(x + 1) = 0

53 - Respuestas.
1
a) x = 6; y = 8 f ) a = 4; t = 2
40
b) x = − 11 ; y = − 14
11 vf −v0
g) t = 5; a = 2
c) x = −4; y = 5
h) t = 5; vf = −15
d) Indeterminado. √
10+5 102

e) t = 43 ; x = 49 8 i) t = 49 ; vf = − 102

58
20

j) 49 ; −2 , (0; 2) k ) (6; 23), (1; −2)

54 - Veinte gallinas y diez ovejas.


55 - P = 146cm.
57 - Las soluciones enteras al sistema de ecuaciones son las ternas
(4; 6; 0), (7; 0; 3), (6; 2; 2) y (5; 4; 1).
59 - Hay 28 máquinas que producen 31 tubófonos cada una.
60 - En un bolsillo hay 130 pesos y en el otro hay 260 pesos.
61 - Miden 25 cm, 26 cm y 27 cm. Sí, es posible hallar el área (por ejemplo, utilizando la
fórmula de Herón). El área aproximada es 291.85 centímetros cuadrados.
62 - 200 metros de largo y 40 metros de ancho.
63 - Los lados miden 9 m y 6 m.
64 - Las medidas son: C1 = 21; C2 = 28; H = 35
65 - El primer gráfico corresponde al sistema c y el tercero al sistema b.
66 - Resolución.

a) Trabajamos de la misma manera que en una ecuación para despejar la incógnita.


1
3x + 1 < 2 =⇒ 3x < 2 − 1 =⇒ x <
3
Entonces la incógnita x tiene que ser menor a un tercio. El conjunto solución de esta
inecuación es un intervalo real abierto. En símbolos:
 
1
x ∈ −∞;
3

b) Antes de comenzar la resolución de la inecuación es importante notar que:

Si a = b =⇒ k · a = k · b

Para todo k ∈ R.
Esta propiedad también se verifica en las desigualdades si k ≥ 0. Sin embargo la
multiplicación por un número negativo no es homogénea en las desigualdades.
Notamos (a modo de ejemplo) que:

4<7

Pero observemos que


(−2) · 4 6< (−2) · 7
Dado que
−8 > −14
En general, resulta que si k es negativo entonces

a < b −→ k · a > k · b

Análogamente, si
a > b −→ k · a < k · b
Entonces, si multiplicamos (o dividimos) ambos términos de una inecuación por un
número negativo, debemos invertir la desigualdad. Lo mostramos en el ejemplo.

−2x + 7 < 5 −→ −2x < 5 − 7 −→ −2x < −2

Aplicamos la propiedad mencionada anteriormente (Notemos que hemos invertido la


desigualdad).
−2
x>
−2

59
x>1
El conjunto solución es el intervalo real

x ∈ (1; +∞)

La resolución del problema c) queda a cargo de los estudiantes.


68 - Respuestas.

Polinomio Coef. principal Mónico Grado Tér. ind. Ordenado Completo


A(x) = −x8 + x9 − 17 1 No 9 -17 No No

B(x) = 5x2 + 21 x − 9 5 No 2 -9 Sí Sí

x4 1
C(x) = 3 + x2 + x 3 No 4 0 Sí No

D(x) = 11 11 No 0 11 Sí Sí

4 4
E(x) = 3 − 7x5 + x3 + x10 1 Sí 10 3 No No

N (x) = 0 0 No No tiene 0 Sí Sí

69 - Resolución.

A(x) + B(x) = (3x + 7) + (7x − 29)


A(x) + B(x) = 3x + 7x + 7 − 29
A(x) + B(x) = 10x − 22
Calculamos
F (x) + G(x) = x2 + 1 + 9x − 2
F (x) + G(x) = x2 + 9x − 1
Comentario: en la suma de polinomios el polinomio resultado tiene grado menor o igual
que el mayor de los grados de los polinomios que tenemos que sumar.
70 -
A(x) · B(x) = 3x · (7x − 29)
A(x) · B(x) = 21x2 − 87x

71 - Resolución
 
2 1 2
C(x) · D(x) = (5x + x − 17) · 7x + x + 9
2
19 3 147 2 x
C(x) · D(x) = 35x4 + x − x + − 153
2 2 2
Comentario: En la multiplicación de polinomios el polinomio resultado tiene grado igual
a la suma de los grados de los polinomios que se multiplican.
72 - Resolución.

3x + 7 3x 7 3 1
= + = +
7x 7x 7x 7 x
73 - Resolución. Aplicamos la Regla de Ruffini, ya que los polinomios dividendo y
divisor cumplen con las condiciones para poder aplicar la regla.

60
Disposición práctica:
1 1 −12
3 3 12
1 4 0

El polinomio cociente es C(x) = x + 4 ya que es un polinomio con un grado menos


que el polinomio dividendo. Y los coeficientes del polinomio cociente son todos los
números que están debajo de la línea horizontal excepto el último.
El resto es el último número debajo de la línea horizontal. En este caso es 0, por lo
tanto el polinomio resto es R(x) = 0.
Si el polinomio resto es cero, entonces el número que está a la izquierda de la línea
vertical (en este ejemplo, es el número 3), es raíz del polinomio dividendo.
Si el polinomio resto es distinto de cero, entonces el número que está a la izquierda
de la línea vertical (en este ejemplo, es el número 3), no es raíz del polinomio
dividendo.

74 - Resolución.Debemos verificar si:

x2 + x − 12 = (x + 4) · (x − 3) + 0

x2 + x − 12 = x2 − 3x + 4x − 12
x2 + x − 12 = x2 + x − 12

75 - Resolución. El resto de la división de P (x) : A(x) es cero, ya que:

P (−7) = (−7)2 + 5(−7) − 14

P (−7) = 0

76 - Resolución. Especificamos el polinomio A en x = 3. El resultado de la


especificación debe coincidir con el resto de la división. Entonces:

A(3) = 32 + 3 − 12

A(3) = 0

107 - Resolución.
360o −→ 2π
90o −→ x
Entonces
90o · 2π π
= x =⇒ x =
360o 2
o π
Es decir que un ángulo de 90 en sistema sexagesimal, equivale a un ángulo de 2 radianes
en sistema radial.
112 - Respuestas.

a) S = {−2π; −π; 0; π; 2π} f ) S = {− 53 π; − 13 π; 13 π; 53 π}


b) S = {− 23 π; − π2 ; π2 ; 23 π} g) S = {− 43 π; − 23 π; 23 π; 43 π}
c) S = {− 32 π; 21 π} h) S = {− 11 7 1 5
6 π; − 6 π; 6 π; 6 π}
d ) S = {−2π; 0; 2π} i) S = {− 56 π; − 16 π; 76 π; 11
6 π}
e) S = {− 12 π; 23 π} j ) S = {− 74 π; − 34 π; 14 π; 54 π}

114 - Respuestas. a) sen(α) b) cos(α) c) − sen(α) d) cos(α)

61
**

Relaciones y
funciones
3. Relaciones y funciones

Definición. Una relación es un conjunto de pares ordenados. Al conjunto de las primeras


componentes lo denominamos Dominio y al conjunto de las segundas componentes lo
denominamos Conjunto imagen; a cada segunda componente la denominamos imagen
de la primera componente.

Definición. Una función es una relación que verifica que:

Para todo elemento del dominio existe una imagen en el codominio30 .

La imagen de cada elemento del dominio es única31 .

Podemos interpretar a una función como una máquina que transforma a la primera
componente en la segunda. Por ejemplo, la función y = 2x transforma a cada valor de x
(que es la primera componente) en su duplo.

3.1. Dominio

Las funciones son la base del trabajo que desarrollamos en Matemática I, ya que ellas nos
dan la posibilidad de modelizar problemas. De esta forma estudiamos posibles soluciones
a partir de los parámetros y variables de los problemas.
Las funciones que estudiamos en Matemática I son las que tienen una única variable real
independiente. De donde surge la necesidad de calcular y definir el conjunto dominio de
la función.

Definición de dominio de una función

En lenguaje coloquial: es el conjunto de valores de la variable independiente para


los cuales está definida o existe la función.

En lenguaje formal:
Df = {x ∈ X|∃ y ∈ Y ∧ f (x) = y}

Reglas para determinar el dominio de una función

Para determinar el dominio de una función de variable real tenemos que prestar en
general atención a las siguientes reglas y luego realizar los planteos y cálculos
correspondientes.

Para cualquier función tenemos que excluir de su dominio aquellos valores de la


variable independiente que anulan el denominador, es decir que hacen que el
denominador sea igual a cero. Ya que la división por cero no está definida en
Matemática. En símbolos:
A
f (x) = ⇒ B 6= 0
B

Si la función presenta raíces de índice par, entonces el radicando tiene que ser
mayor o igual que cero. Es decir:

2k
f (x) = A⇒A≥0

63
Si existen logaritmos entonces su argumento tiene que ser mayor que cero. Es decir:

f (x) = logb a ⇒ a > 0

Si la función modeliza un problema es muy importante evaluar qué valores puede


tomar la variable, en referencia al problema. Por ejemplo, si la variable x es la
longitud de un lado de un triángulo, entonces no puede ser negativo.

Aclaración: Las reglas que enunciamos se pueden combinar de distintas maneras en una
función. Por ejemplo, se puede presentar una función que tenga denominador y en él una
raíz cuadrada, por lo tanto será necesario considerar las dos primeras reglas y determinar
los valores que tenemos excluir para determinar el dominio de la función.
Las funciones polinómicas no tienen restricciones, es decir que su dominio es el conjunto
de todos los números reales. Existen otras funciones con otro tipo de restricciones que no
trataremos en Matemática I.

Problema 201. Determinen el dominio de cada una de las funciones de fórmula:


q
a) f (x) = 3x + 1 l ) f (x) = −7x + 37
b) g(x) = −x2 + 1 √3
2
m) y = x
c) h(x) = x + x √
√ n) y = x3
4

d ) y = 5x3 + x − 7
e) f (x) = 1 ñ) y = log(x)
x
f ) g(x) = x3 o) y = ln(x)
x+2
g) h(x) = x p) y = ln(4x − 1)
x2 −1

h) f (x) = sen(x) q) y = log2 (−x + 3)


5
2
x− 71 √ 
1 r ) y = ln − 32 x + 11
i) y = sen(x)
√ 1
s) y = ln(x)
j) y = x
√ √
−5x+0,8·log3 (38x)
k ) y = 2x + 9 t) y = −x2 +1

Problema 202. Determinen el dominio de cada una de las funciones en el intervalo real
indicado.
x
a) y = −5x en [2; 8] d ) h(x) = x+2 en [−3; 10]
b) f (x) = cos(x) en [0; π] 1
e) y = x3 −x
en [−9; +∞)
x
c) g(x) = x+2 en [−1; 2] f ) f (x) = ln(−x + 8) en (−∞; 7]

Problema 203. Determinen el dominio de las siguientes funciones a tramos.


(
a) y = |x| x
si x > 2
d ) y = x−3
−x2 +9
b) f (x) = |4x − 1| x+3 si x < 2
( (
2x si x > 1 log(x) si x < 0
c) g(x) = 2
e) g(x) =
x − 7x si x < 1 x2 − x si x ≥ 0

Problema 204. Determinen el dominio de las siguientes funciones.

a) Consideramos todos los cuadrados de lado x < 5 metros y denominamos f (x) al


perímetro del cuadrado.

64
b) Disponemos de una cuerda de 100 centímetros de longitud y queremos formar con
ella un rectángulo. Si uno de los lados del rectángulo mide x, entonces f (x) es la
longitud del lado adyacente al lado de longitud x.
c) Consideramos todos los rectángulos de 100 cm2 de área. Si uno de los lados mide x
entonces f (x) es la longitud del lado adyacente al lado de longitud x.
d ) El triángulo abc verifica que: ab = x + 8; bc = x + 11; ca = x + 14. Entonces f (x) es
el perímetro del triángulo abc.

Definición. Una función f es par si, para todo a perteneciente al dominio de f se


verifica que f (a) = f (−a).

Análogamente, decimos que una función f es impar si, para todo a perteneciente al
dominio de f resulta f (a) = −f (−a).

Nota: hay funciones que no son pares ni impares.

Problema 205. Sean f y g dos funciones pares. Estudien la paridad de f + g y de f · g.

Problema 206. Determinen si existen funciones que sean pares e impares


simultáneamente. Justifiquen la respuesta.

Una función f es inyectiva si:

∀x1 ; x2 ∈ Df : f (x1 ) = f (x2 ) ⇒ x1 = x2

Una función f es sobreyectiva si:

∀b ∈ Codf : ∃a ∈ Df : f (a) = b

Una función f es biyectiva si es inyectiva y sobreyectiva.

Problema 207. Respondan verdadero o falso y justifiquen la respuesta.

a) Una función es una relación cualquiera entre dos variables.


b) Una función inyectiva le asigna a cada elemento del dominio una imagen distinta.
c) Una función biyectiva puede NO ser sobreyectiva
d ) Toda recta que pasa por el origen de coordenadas es una función impar
e) Todas las rectas son funciones impares.
f ) La función y = mx + b es impar sólo para b = 0.

Si necesitan más problemas resueltos, pueden consultar en el siguiente link:


http://fernandorevilla.es/blog/2014/02/13/
aplicaciones-inyectivas-sobreyectivas-y-biyectivas/

Problema 208. Determinen la paridad de cada una de las siguientes funciones.

a) f (x) = x d ) f (x) = (2 − x) · (3 − x) + (x + 2) · (x + 3)
b) f (x) = 1/x e) f (x) = (1 + x) · (1 − x)
c) f (x) = x2 +x f ) f (x) = 0

Definción: Denominamos conjunto de positividad de la función f al conjunto de todos


los valores de la variable x tales que f (x) > 0; lo simbolizamos C + . Asimismo, el
conjunto de negatividad de f es el conjunto de todos los valores de la variable x tales
que f (x) < 0; lo simbolizamos C − . Notemos que C + ∪ C − ∪ C 0 = Domf .

65
Definición: Decimos que una función es creciente en el intervalo (a; b) si, para todo
x1 , x2 ∈ (a; b) se verifica que, si x1 < x2 ⇒ f (x1 ) < f (x2 ). Análogamente, una función
es decreciente en el intervalo (a; b) si, para todo x1 , x2 ∈ (a; b) se verifica que, si
x1 < x2 ⇒ f (x1 ) > f (x2 ).

Problema 209. Para cada una de las siguientes funciones:

a) Hallen las raíces, los intervalos de positividad y de negatividad.


b) Determinen su paridad.
c) Grafiquen en forma aproximada, considerando lo hallado anteriormente.
d ) Hallen los intervalos en donde la función crece, decrece o es constante.

1) y = 34 x + 2 5) y = (x − 3) · (x + 1)
1
2) y = − 21 x 6) y = x+1 −3
(
3) y = x2 + 1 x2 si x < 0
1 7) y =
4) y = 3 (x − 1)2 − 3 x si x > 0

3.2. Funciones polinómicas.

Las funciones polinómicas son aquellas en donde la fórmula que las definen es un
polinomio. Nos proponemos estudiarlas según el grado del polinomio.

Función lineal

Decimos que una función es lineal si es de la forma32 f (x) = mx + b y algunas de sus


características son:

Su representación gráfica es una recta (que no es vertical, ya que las rectas verticales
no representan funciones).
Depende de dos parámetros: m (que es la pendiente), y b (que es la ordenada al
origen).
La ordenada al origen b indica el valor en el cual la recta interseca al eje Y . Es decir
que la intersección entre la recta y el eje Y es siempre en el punto (0; b).
Dos rectas paralelas tienen la misma pendiente.
Si el producto entre las pendientes de dos rectas es igual a −1, entonces las rectas
son perpendiculares. Vale también la implicación recíproca (es decir, si dos rectas
son perpendiculares el producto de las pendientes es igual a menos uno)33 .
La raíz de una función es el valor en el cual su gráfica interseca al eje X.
b
En el caso de una función lineal, la raíz es x = − m siempre que m 6= 0.
Si m = 0 y b 6= 0 la función no tiene raíz.
Para el caso particular y = 0, la función es constante, coincide con el eje X y tiene
infinitas raíces.
La pendiente m es igual a la tangente del ángulo que forma la recta con el eje X.
∆y y2 −y1
Equivalentemente: m = ∆x = x2 −x1 . A esta expresión la denominamos cociente
incremental.
Si m = 0 entonces la recta es paralela al eje X.
32
A la expresión y = mx + b la denominamos ecuación explícita de la recta.
33
Con excepción de las rectas paralelas a los ejes.

66
Intersección entre dos rectas

Si dos rectas no son paralelas (es decir, si sus pendientes son diferentes) entonces se
intersecan en un punto.
¿Cómo hallamos el punto de intersección de las rectas?
Proponemos un ejemplo.
Hallaremos la intersección entre las gráficas de y = 3x − 7 e y = 5x + 1, de la siguiente
manera:

a) Escribimos el sistema de ecuaciones:



y = 3x − 7
y = 5x + 1

b) Igualamos ambas ecuaciones:


3x − 7 = 5x + 1

c) Resolvemos la ecuación y obtenemos:

x = −4

d ) Reemplazamos el valor de x obtenido en ambas funciones, para verificar la


resolución de la ecuación. Debemos obtener en ambas funciones el mismo valor de y.

f (−4) = 3 · (−4) − 7 = −19

g(−4) = 5 · (−4) + 1 = −19

e) Escribimos el punto de intersección:

p ≡ (−4; −19)

Nota: este procedimiento es válido para encontrar la intersección entre cualquier par
de funciones, sean o no funciones lineales.

67
Construcción de la ecuación de la recta a partir de algunos datos

Existen distintas variantes para construir la ecuación de una recta a partir de ciertos
datos.
En algunos casos contamos con la gráfica de una recta, o con algunos datos acerca de ella
(por ejemplo un par de puntos, o un punto y su pendiente) y necesitamos hallar la
ecuación de dicha recta, entonces (conociendo el dato de que se trata de una función
lineal) planteamos la ecuación y = mx + b en la cual las incógnitas son m y b y
reemplazamos los datos.

Problema 210. Halle la ecuación de la recta determinada por los puntos (2; 3) y (5; 24).
Resolución. Planteamos la ecuación y = mx + b. Sabemos que, si x vale 2 entonces y
vale 3. Por lo tanto reemplazamos los valores de x e y en la ecuación y obtenemos

3=m·2+b

De la misma manera, obtenemos la ecuación

24 = m · 5 + b

Despejamos una de las incógnitas (m o b) en ambas ecuaciones, por ejemplo:



3 − 2m = b
24 − 5m = b

Igualamos y resolvemos:
3 − 2m = 24 − 5m
3m = 21
m=7
Reemplazamos el valor de m en ambas ecuaciones y obtenemos b = −11. Luego la
ecuación de la recta que buscada es

y = 7x − 11

Comprobamos que la expresión que hallamos es correcta. Para ello, reemplazamos en la


ecuación las coordenadas de los puntos dados. Obtenemos una identidad. Las cuentas
quedan a cargo del estudiante.
Otra forma de resolver el mismo problema es hallar la pendiente de la recta a partir de
los dos puntos dados.
Recordamos que la pendiente de una recta se define como
∆y
m=
∆x
entonces planteamos
24 − 3
m=
5−2
21
m= =7
3
Reemplazamos el valor obtenido de m y las coordenadas de alguno de los puntos dados
obtenemos
y = mx + b
3=7·2+b
Despejamos el valor de b:

3 = 14 + b =⇒ 3 − 14 = b =⇒ b = −11

68
Escribimos la ecuación de la recta buscada:
y = 7x − 11
Podemos verificar que la ecuación obtenida es correcta reemplazando en ella las
coordenadas del otro punto dado.

Comentario. En general, la ecuación de la recta con pendiente m que contiene al punto


(x0 ; y0 ) es
y − y0 = m(x − x0 )
∆y
Si conocemos dos puntos de la recta, podemos hallar m como el cociente incremental ∆x .

Función cuadrática

Una función es cuadrática si es de la forma34 f (x) = ax2 + bx + c. Algunas de sus


características son:
Su representación gráfica es una parábola.
Depende de tres parámetros, a, b y c que pueden ser números reales cualesquiera,
siempre que35 a 6= 0.
El número a indica la concavidad de la parábola: si a es positivo, entonces la
parábola es cóncava positiva; si a es negativo, entonces la parábola es cóncava
negativa.
La ordenada al origen c indica el valor en el cual la parábola corta al eje Y . Es decir
que la parábola siempre interseca al eje Y en el punto (0; c).
La raíz de una función es el valor de x para el cual f (x) = 0. Es decir, es el valor de
la abscisa en el cual su gráfica interseca al eje X. En el caso de una función
cuadrática, las raíces36 son

−b ± b2 − 4ac
x1;2 =
2a
Las parábolas tienen un punto (denominado vértice) que es el extremo absoluto de
la curva. La coordenada x del vértice es el promedio entre las raíces, es decir:
x1 + x2
xV =
2
Podemos verificar (o calcular el valor de xV si la función no tiene raíces) mediante
b
xV = −
2a
El valor de yV lo obtenemos calculando f (xV ), es decir reemplazando en la fórmula
de la función el valor de xV hallado37 .

34
Decimos que esta es la expresión polinómica. Existen expresiones equivalentes de la misma ecuación,
que también definen una función cuadrática. Las mostraremos más adelante.
35
Si a es igual a cero, estamos en presencia de una función lineal.
36
Eventualmente puede tener una sola raíz, si b2 − 4ac es igual a cero, o ninguna, en el caso en que
2
b − 4ac sea negativo.
37
Recuerden que el vértice es un punto; y como tal, tiene dos coordenadas.

69
Forma factorizada

La fórmula de una función cuadrática la podemos expresar en función de sus raíces; esta
expresión se denomina forma factorizada. Si las raíces de la función f (x) = ax2 + bx + c
son x1 y x2 entonces f (x) = a(x − x1 )(x − x2 ).

Forma canónica

También podemos expresar la ecuación de una función cuadrática, en función de las


coordenadas del vértice. Si el vértice de la función f (x) = ax2 + bx + c es el punto
V = (xV ; yV ) entonces f (x) = a(x − xV )2 + yV .

Teorema fundamental del álgebra

Sea p(x) un polinomio con coeficiente principal a de grado n con n raíces reales
(x1 ; x2 ; · · · ; xn ). Entonces p(x) = a(x − x1 )(x − x2 ) · · · (x − xn ).

Gráfica de las funciones polinómicas de grado mayor a dos

Para graficar una función polinómica de grado mayor a 2, consideramos la forma


factorizada de la ecuación (para ello, debemos hallar las raíces). Según la multiplicidad de
las raíces, pueden suceder tres cosas:

La raíz es de multiplicidad uno.


La raíz es de multiplicidad par.
La raíz es de multiplicidad impar mayor a uno.

En el primer caso, la función interseca al eje X en esa raíz.


En el segundo caso, la función rebota en el eje X.
En el tercer caso, la función interseca al eje X pero además posee (en dicho punto) un
punto de inflexión. Gráficamente:

En el gráfico observamos que x = A es una raíz de multiplicidad 1, x = B es una raíz de


multiplicidad par y x = C es una raíz de multiplicidad impar mayor a 1.
Un punto que resulta útil para completar la gráfica es la ordenada, es decir el punto
(0; f (0)).

Problema 211. Hallen la ecuación de una función lineal que pase por los puntos
p ≡ (1; −2) y q ≡ (3; 4).

70
Problema 212. Identifiquen sin hacer cuentas qué sistema corresponde a cada gráfico
(puede haber sistemas o gráficos que no tengan correspondencias).
( ( ( (
x+y =4 y =x+4 y = −x + 4 y = −2x + 2
a) b) c) d)
x−y =2 y = 2x + 2 y =x+2 y =x−2

Problema 213. Hallen la expresión de la función lineal f al que la recta correspondiente


cumpla que:

a) Su pendiente es 3 y contiene al punto (−1; 2).


2
b) Su pendiente es 3 y contiene al punto (4; −1).
c) Contiene a los puntos (3; 2) y (−2; 4).
d ) Contiene a los puntos (−4; 0) y (5; −1).
e) Contiene a los puntos (3; 2) y (−2; 2).
f ) Contiene a los puntos (4; 2) y (4; 4).
g) Contiene al punto (20; 15).
h) Encuentren (deduzcan, calculen) la ecuación de una recta que pasa por los puntos
(50; 10) y (104; 40).

Problema 214. Identifiquen de las siguientes tablas cuáles representan datos que pueden
modelizarse como una función lineal.

Para las tablas que hayan determinado que responden a modelos lineales:

a) Construyan nuevos pares de datos que conserven la linealidad.


b) Escriban una ecuación que puede modelizar esos datos. En los otros casos expliquen
por qué no responden a modelos lineales.

Problema 215. Un auto sale de una ciudad que está sobre la Ruta Nacional 2, entre
Buenos Aires y Mar del Plata. En la siguiente tabla se informa sobre la distancia a la que
se encuentra de Buenos Aires en distintos momentos de su viaje. Se supone que el auto
viaja siempre a la misma velocidad.

71
a) ¿Es cierto que a las tres horas de salir está a 320 km de Buenos Aires?
b) ¿A qué distancia de Buenos Aires estará a las tres horas y media de haber salido?
c) ¿A qué distancia de Buenos Aires se encuentra la ciudad de donde partió?
d ) Realicen un gráfico que represente la distancia del auto a Buenos Aires a medida
que transcurre el tiempo de viaje. Justifiquen.
e) ¿A qué velocidad viaja el auto?
f ) Propongan una fórmula que permita calcular la distancia del auto a Buenos Aires en
función del tiempo transcurrido.
g) Otro auto parte, al mismo tiempo que el primero, desde otra ciudad que está en la
ruta entre Buenos Aires y Mar del Plata, ubicada a 10 km de Buenos Aires. Este
auto también viaja siempre a la misma velocidad: 120 km/h. ¿Se van a cruzar estos
dos vehículos? En caso afirmativo, ¿en dónde y en qué momento? h) Realicen, en un
mismo sistema de ejes, los dos gráficos de manera que cada uno represente la
distancia de cada uno de los autos a Buenos Aires en función del tiempo.

Problema 216. El siguiente gráfico representa la distancia d de un auto a Buenos Aires en


función del tiempo t a medida que recorre la Ruta Nacional 2 en dirección hacia Mar del
Plata.

a) Armen una tabla de valores que representen distintos momentos del viaje y
márquenlos en el gráfico:
1) El momento en que parte.
2) Cuando pasaron 2 horas y media de viaje.
3) Cuando se encuentra a 300 km de Buenos Aires.
4) Cuando pasaron 4 horas de viaje.
5) Cuando se encuentra a 230 km de Buenos Aires.
6) El momento en que finaliza el viaje. ¿Llega a Mar del Plata?
b) Definan una función (armen una fórmula que establezca una relación entre t y d y
expresen un dominio) de manera tal que el gráfico anterior represente dicha función.

72
Problema 217. El gráfico representa el proceso de vaciado de un tanque de agua.

a) ¿Qué cantidad de agua tenía el tanque cuando empezó a vaciarse?


b) ¿Cuánto tardó en vaciarse?
c) ¿Cuántos litros por minuto salían del tanque mientras se vaciaba?
d ) Marquen sobre el gráfico el punto que representa el momento en que el tanque tenía
2000 litros. ¿Cuánto tiempo había transcurrido desde que comenzó a vaciarse?
e) Escriban una fórmula que calcule la cantidad de agua que había en el tanque a los x
minutos de haber comenzado a vaciarse.

Problema 218. Obtengan la ecuación de la recta y representen en cada uno de los


siguientes casos:

a) Es paralela a y = − x2 + 2 y pasa por (3; 4).


b) Es perpendicular a y = 3x − 2 y pasa por (3; 0).
c) Pasa por (0; 0) y por (3; 5).
d ) Interseca al eje y en y = 5 y tiene pendiente −1.
e) Interseca al eje x en x = 1 y tiene pendiente 2.
f ) Es horizontal y pasa por (1; −2).
g) Tiene raíz igual a −4 y ordenada al origen 3.
h) Corta al eje y en y = −2 y pasa por (1; −3).

Problema 219. Esta es una foto de un termómetro que puede medir la temperatura tanto
en la escala Fahrenheit como en la escala Centígrado. Lamentablemente la nitidez de la
foto no es clara y solo se ven algunos números.

a) ¿Cómo puede usarse la información de esta imagen para tradu-


cir a grados centígrados (◦ C) las temperaturas que en el texto
aparecen en grados fahrenheit (◦ F)?

b) ¿Cuántos ◦ C disminuiría la temperatura media, sin la presencia


de los gases de efecto invernadero?

c) ¿Qué fórmula pueden proponer para transformar una tempera-


tura medida en grados Fahrenheit a grados Celsius?

d ) ¿Existe alguna temperatura para la cual no sea necesario indicar


si está medida en Celsius o en Fahrenheit? ¿Cuál? ¿Por qué?

73
Problema 220. Decidan, en cada caso, si los pares de rectas correspondientes a las
siguientes funciones lineales, son transversales o paralelas. En caso de ser transversales,
hallen las coordenadas del punto de intersección. Interpreten la solución hallada en un
gráfico.

a) f : R −→ R, f (x) = 3x + 1 y g : R −→ R, g(x) = −3x + 4


−x+7
b) f : R −→ R, f (x) = − 13 x + 1 y g : R −→ R, g(x) = 2
c) f : R −→ R, f (x) = 3x + 1 y g : R −→ R, g(x) = 3x − 1
−x+3
d ) f : R −→ R, f (x) = − 13 x + 1 y g : R −→ R, g(x) = 3
−x+7
e) f : R −→ R, f (x) = −x + 1 y g : R −→ R, g(x) = 2

Problema 221. Una bolita de vidrio es lanzada hacia arriba desde 1 metro de altura y
con una velocidad de 15 m s . Si la variable t representa el tiempo (medido en segundos), la
altura (en metros) a la que estará la bolita en cada instante está determinada por la
fórmula f (t) = −5t2 + 15t + 1.

a) ¿A qué altura del piso estará la bolita 0,5 segundos después de ser lanzada?
b) ¿A qué altura estará 2 segundos después de ser lanzada?
c) ¿Se puede asegurar que la bolita estuvo ascendiendo durante los primeros dos
segundos desde que fue lanzada?
d ) Observen que, para esta función, es f (0) = 1. ¿Qué significa esta información en el
contexto del problema?
e) Observen que f (3) = 1 y f (4) = −19. ¿Qué significan estos valores en el contexto
del problema?
f ) ¿Cuánto tiempo tarda la bolita en llegar al piso?

Problema 222. Un productor tiene una plantación de una hectárea con 40 naranjos. Cada
uno de ellos produce 500 naranjas por año. Si decide aumentar la cantidad de plantas,
sucede que por cada planta que incorpora a la parcela, la producción de cada naranjo
disminuye en 5 unidades.

a) Estudien y representen a través de una fórmula cómo varía la producción de


naranjas por planta a medida que cambia la cantidad de plantas.
b) Estudien y representen a través de una fórmula cómo varía la producción total de
naranjas a medida que cambia la cantidad de plantas.
c) En un determinado año se produjeron aproximadamente 17000 naranjas. ¿Cuántos
árboles había plantados en total?
d ) En otro año se calcula que la producción fue de 21500 naranjas. ¿Cuántos árboles
había en esa oportunidad?
e) ¿Qué otra pregunta consideran que puede resultar interesante para el productor?
¿Cómo responderían esa pregunta?

Problema 223. Un grupo de montañistas hace sus prácticas semanales; uno de los
ejercicios con que practican consiste en tirar de una cuerda elástica atada a un poste. La
tensión que se produce en el elástico, t segundos después de comenzar a tirar de la
cuerda, está dada por la siguiente fórmula: f (t) = 615t − 5t2 − 105.

a) Determinen dentro de qué dominio es válida esta función.


b) Calculen el valor de la tensión máxima y en qué momento se alcanza.

Problema 224. Tenemos que delimitar un terreno rectangular de 400 metros cuadrados
de área con tres paredes de cemento y una de alambre. El costo del metro de pared se
estima en $100 y el de alambre se estima en $30.

74
a) Determinen el costo de delimitar el terreno si mide 40 metros de largo por 10 metros
de ancho (suponga que el alambre es el lado menor).
b) Determinen el costo de delimitar el terreno si mide 40 metros de largo por 10 metros
de ancho (suponga que el alambre es el lado mayor).
c) Determinen el costo de delimitar el terreno si fuera cuadrado.
d ) Elija otras medidas posibles del terreno y calcule el costo de delimitarlo.
e) Si denominamos z a la medida de dos de las paredes y x a la medida de las otras dos
paredes. Supongan que la pared de alambre mide z. Encuentren una expresión en
función de x y de z que represente el costo de delimitar el terreno.
400
f ) Expliquen claramente porqué podemos afirmar que z = x
g) Hallen una función y = f (x) que represente el costo en función de la medida del
lado x.
h) Grafiquen la función f en el GeoGebra; encuentren las medidas que minimizan el
costo.

Problema 225. El área de un terreno rectangular mide 891 metros cuadrados. Además
sabemos que el perímetro es igual a 120 metros. Denominamos x al largo del terreno y z
al ancho.

a) Escriban una fórmula que represente el área del terreno.


b) Escriban una fórmula que represente el perímetro del terreno.
c) Escriban una función y = f (x) equivalente a la primera fórmula, y otra función
y = g(x) equivalente a la segunda.
d ) Grafiquen ambas funciones en un mismo sistema de ejes cartesianos.
e) Hallen la o las intersecciones entre f y g.
f ) Hallen la medida de la diagonal del rectángulo.
g) Hallen la medida de los ángulos que la diagonal determina con cada uno de los lados.

Problema 226. Hallen las raíces y la multiplicidad de cada una de ellas. Indiquen los
conjuntos de positividad y negatividad, Grafiquen en forma aproximada las siguientes
funciones polinómicas.

a) f (x) = x3 − 7x2 + 15x − 9 c) f (x) = (x − 3)4 · (x + 3)2


b) f (x) = (x2 − 4)2 · (x + 2) d ) f (x) = (x + 1)5 · (x − 2)2

Comentario. Pueden realizar la autocorrección del problema graficando las funciones en


el GeoGebra.

75
Problema 227. Consideren las funciones

f (x) = mx + b g(x) = −x2 − x + 6

a) Hallen las raíces y la ordenada de la función g.


b) Sea r1 la menor de las raíces de g y sea c la ordenada al origen de g; hallen los
valores de m y b de forma tal que la recta de ecuación f pase por r1 y por c. .
c) De la recta h sabemos que es paralela a f y que pasa por el punto (−2; 1). Hallen la
ecuación de h(x).
d ) Hallen los puntos de intersección entre la recta h y la parábola g.
e) Determinen el conjunto imagen de g.
f ) Hallen los intervalos de crecimiento y de decrecimiento de la función g.
g) Hallen los intervalos de positividad y negatividad de la función g.
h) Grafiquen las tres funciones en un mismo sistema de ejes coordenados.

Problema 228. Para cada ítem hallen una expresión de una función cuadrática f que
verifique que:

a) Sus raíces son 2 y −3.


b) Sus raíces son 2 y −3 y que además pase por el punto (1; 16).
c) Su vértice es el punto (1; 2).
d ) Su vértice es el punto (1; 2) y que además pase por el punto (2; 5).
e) f (2) = f (8) = 4 y que su coeficiente principal sea −2.
f ) f (1) = 2, f (3) = 6 y f (2) = 16.

3.3. Funciones homográficas

Las funciones homográficas son las de ecuación y = ax+b cx+d (o reducibles a ella) tales que
c 6= 0 y a · d 6= b · c. A su gráfica la denominamos hipérbola. Para representarla, sugerimos
hallar su raíz (si tiene), su ordenada (si tiene) y las ecuaciones de sus asíntotas (siempre
tiene una asíntota horizontal de ecuación y = ac y una asíntota vertical de ecuación
x = − dc ).

Problema 229. Hallen el dominio, las ecuaciones de las asíntotas, la raíz y la ordenad de
x+3
la hipérbola de ecuación f (x) = 2x−4 y grafíquenla en un sistema de ejes cartesianos.

Problema 230. Observen que en la definición de la función homográfica existen dos


condiciones.

a) ¿Qué sucede si en la ecuación dada c vale cero?


b) ¿Qué sucede si en la ecuación dada a · d = b · c?

Problema 231. Grafiquen cada una de las siguientes funciones homográficas. Indiquen su
dominio e imagen, la raíz (si existe), la ordenada (si existe) y las ecuaciones de las
asíntotas.
2x+3 x
a) f (x) = x+1 c) f (x) = x−2
x−1 2x+6
b) f (x) = x+1 d ) f (x) = 4x−1

76
3x2 +3x−6
Problema 232. Consideren la función f (x) = x2 −3x+2

a) Prueben38 que hay infinitos valores de x para los cuales la expresión de f es


equivalente a y = 3x+6
x−2 .

b) Hallen el dominio. e) Hallen el conjunto imagen.


c) Hallen la raíz y la ordenada.
d ) Hallen las ecuaciones de las asíntotas. f ) Grafiquen la función.

Problema 233. Sacamos del congelador el recipiente de los cubitos de hielo. Su


temperatura es de −15o C. Lo dejamos sobre la mesa y notamos que al cabo de 10
minutos empieza a descongelarse. Una hora y veinte minutos después se descongeló.
Quince minutos más tarde su temperatura es de 10 o C. Asumimos que la variación de la
temperatura es una relación lineal que depende del tiempo. Representen la función T (t)
de la temperatura T (en o C), en función del tiempo t (en minutos) durante los 105
minutos de observación. Escriban su expresión analítica.

Problema 234. Hallen una función y = f (x) de la cual sabemos que:

Es un polinomio de tercer grado.


Interseca al eje X en los puntos de abscisas x = −2; x = 1 y x = 3.
Interseca al eje Y en el punto (0; 3).

Problema 235. Determinen el dominio, las intersecciones con los ejes, intervalos de
positividad y negatividad, y grafiquen las siguientes funciones:

a) f (x) = x2 + 2x − 8 c) f (x) = |x| − 2


b) f (x) = x4 − 2x2 +1 d ) f (x) = x3 − x

Problema 236. La ecuación que describe la relación entre la rapidez del sonido y la
temperatura del aire es S = 0, 61T + b. Cuando la temperatura es 15 o C, un investigador
determina que la velocidad del sonido es 340, 55 m
s . Determinen el valor de b y escriban la
ecuación.

Problema 237. En un circuito eléctrico el voltaje V (en volts), y la corriente I (en


Amperes), están relacionados linealmente. Cuando I = 4A el voltaje es V = 2v. Si la
corriente es I = 12A entonces V = 6v.

a) Determinen V como función de I.


b) Encuentren el voltaje cuando la corriente es de 10A.

Problema 238. La presión P de un volumen constante de gas, en centímetros de


mercurio, está relacionado linealmente con la temperatura T , en grados Celsius. En un
experimento con aire seco, se encontró que P = 90 cuando T = 40, y que P = 100 cuando
T = 80. Expresen P como una función de T .

Problema 239. Una fórmula utilizada a menudo en la física hidráulica es


Q = 3340b3 + 18704b2 x, donde b es una constante.

a) Investiguen el significado de la constante b y de las variables Q y x.


b) ¿Cómo es la gráfica de esta función?
c) Hallen los elementos y su gráfica para b = 1?
38
Factoricen, simplifiquen...

77
Problema 240. Sea h la altura de una pelota lanzada verticalmente hacia arriba desde el
piso. Dicha altura está dada por la fórmula h = −4, 9t2 + 58, 8t (h está medida en metros
y t es el tiempo transcurrido en segundos).

a) ¿Cuánto tarda la pelota en alcanzar su máxima altura?


b) ¿Cuál es esa altura?

Problema 241. Durante una colisión, la fuerza F (en Newtons), que actúa sobre un
objeto varía con el tiempo t, de acuerdo con la ecuación F = 87t − 21t2 , donde t está en
segundos.

a) ¿Para qué valor de t es máxima la fuerza?


b) ¿Cuál es el valor máximo de la fuerza?

Problema 242. La relación entre el voltaje en los extremos de una resistencia y su


corriente eléctrica es de la forma V = I · R.

a) ¿Cómo se llama dicha ley?


b) Escriban la fórmula sabiendo que la resistencia es constante, igual a 5 ohms. ¿Es
una función lineal?
c) Representen gráficamente la relación. Escriban el dominio y el conjunto imagen.

Problema 243. Consideramos la fórmula del problema anterior. Si el voltaje es constante


e igual a 12 volts.

a) ¿Cómo es la relación entre la corriente y la resistencia?


b) ¿Es una función?
c) Representen gráficamente, hallen su dominio y su conjunto imagen.

Problema 244. La relación entre la fuerza que se aplica a un cuerpo de masa m


(constante) y su aceleración a está dada por la fórmula F = m · a.

a) ¿Cómo se llama dicha ley? ¿Es una función?


b) Representen gráficamente la función para m = 4.
c) ¿Cómo es la relación entre la fuerza que se aplica a un cuerpo de aceleración
constante de a = 5 sm2 y su masa? ¿Es una función?
d ) Representen la relación gráficamente.

Problema 245. La relación entre la presión de un gas y su volumen cuando la


temperatura es constante es conocida como Ley de Boyle – Mariotte. Su ecuación es
P · V = k siendo k una constante.

a) ¿Cuáles son las variables de la función?


b) Representen gráficamente la ley y escriban su dominio y su imagen para k = 2.
c) ¿Tiene asíntotas la función? ¿Cuáles?

Problema 246. Investiguen las relaciones entre:

a) El peso de un fluido (líquido o gas) y su densidad (busquen tablas de densidades).


¿Cuánto pesa un litro de aire? ¿Cuánto pesa un litro de agua? ¿Cuánto pesa un litro
de Hg?
b) La presión que se realiza mediante una fuerza F (fija) sobre una superficie S
(variable) ¿Cuál es la presión que soporta una superficie circular cuando se aplica
una fuerza fija de 2 Newtons si su radio es r?

78
c) La presión que soporta un objeto sumergido en un líquido (de peso específico ρ a
una profundidad h ¿Cuál es la presión de un objeto sumergido en...
1) ...agua a 10 metros de profundidad
2) ...agua a 20 metros de profundidad.
3) ...agua salada a 10 metros de profundidad.
4) ...mercurio a 5 metros de profundidad.

Problema 247. Realicen un cuadro para clasificar las leyes mencionadas según sus
fórmulas.

3.4. Funciones exponenciales y logarítmicas

Antes de encarar la lectura de este apunte, te sugerimos que repases la definición de


potenciación, sus propiedades y muy especialmente las propiedades de potencias de igual
base.

Problema 248. Encuentren la función exponencial y = k · ax cuya gráfica pasa por los
puntos (1, 6) y (3, 24).

Problema 249. Una población de aves cuenta inicialmente con 200 individuos y se duplica
cada 3 años. Si x representa la cantidad de años transcurridos y P (x) la población de
aves después de x años, ¿cuál es la fórmula asociada a la función P (x)? Sugerencia: sin
tener aún la fórmula, con la información ofrecida ya se puede saber el número de aves a
los 0, 3, 6 y 9 años transcurridos. Tengan en cuenta que 200 = 200 · 20 . Construyan una
tabla con los valores x y P (x) y a partir de allí deduzcan la fórmula para P (x).

Problema 250. La población de una ciudad se triplica cada 50 años. En el tiempo t = 0,


esta población es de 10.000 habitantes. Den una fórmula para la población P como
función del tiempo t. ¿Cuál es la población después de 100 años, 150 años y 200 años?
¿Es posible hallar en forma aproximada la población después de 173 años? ¿En qué
momento la población es de un millón de individuos?

Problema 251. Tenemos un elemento radiactivo que decae de acuerdo a la siguiente


expresión f (t) = 60 · 2−0,02t . Respondan:

a) ¿Cuál es la cantidad de ese elemento al inicio del proceso?


b) ¿Qué cantidad queda después de 500 años?
c) ¿Y luego de 1000 años?
d ) ¿Y luego de 2000 años?

Problema 252. Si el pH de una sustancia es p entonces la concentración de iones de


hidrógeno, h, en átomos gramo por litro puede representarse por medio de la ecuación
p = log h1 .

a) Indiquen el dominio de la función.


b) Muestren, aplicando las propiedades de los logaritmos, que la ecuación de la función
dada es equivalente a la ecuación p = −log(h).
c) Grafiquen la función y = p(h).
d ) Representen esta ecuación en forma exponencial.

79
Problema 253. Grafiquen las funciones logarítmicas.

a) f (x) = log2 x c) h(x) = −2log(x) + 5


b) g(x) = log2 (x + 5) d ) j(x) = ln(x)

3.5. Composición de funciones

La composición de funciones es una operación que aparece naturalmente en varias


situaciones. En este apartado, presentaremos algunos de los resultados más importantes
relacionados con esta operación. Mostraremos también como se aplican estos resultados
resolviendo varios ejercicios.

Definición y propiedades

Consideremos la función f (x) = 1 + x2 que está definida para todos lo números reales.
Supongamos que queremos calcular f (2). Entonces tenemos que proceder por pasos:

Calculamos el cuadrado de dos: 22 = 4.


Al resultado le sumamos uno: 4 + 1 = 5.

Al resultado le extraemos la raíz cuadrada: 5.

Podemos√ interpretar este procedimiento de la siguiente manera: la función


f (x) = 1 + x2 resulta de la aplicación sucesiva de las siguientes funciones:
elevar al cuadrado sumar uno extraer raíz cuadrada
p
x −−−−−−−−−−−→ x2 −−−−−−→ x2 + 1 −−−−−−−−−−−−−→ x2 + 1

Si llamamos u a la función de elevar al cuadrado; es decir, u(x) = x2 , h a la función de


sumar uno; es decir, h(x) = x + 1, g a la función de extraer la raíz cuadrada; es decir,

g(x) = x, entonces podemos escribir
u h g
p
x −−−→ u(x) = x2 −−−→ h(u(x)) = x2 + 1 −−−→ g(h(x)) = x2 + 1

La operación de aplicar sucesivamente dos o más funciones en un orden determinado da


origen a otra función llamada composición de las funciones.
Del ejemplo anterior, nos damos cuenta que son necesarias algunas condiciones para
poder calcular g(h(u(x))):

x debe pertenecer al dominio de u.


u(x) debe pertenecer al dominio de h.
h(u(x)) debe pertenecer al domino de g.

Todas estas consideraciones las formalizaremos en la siguiente definición:


Sean dos funciones fA→R y gB→R , tales que Im(f ) ⊂ B, llamamos composición de f con
g a la función, denotada g ◦ f , que se obtiene por la aplicación sucesiva de f y g en ese
orden. Es decir: (g ◦ f )(x) = g(f (x)).
(Extraido del documento que pueden encontrar en el siguiente link:
http://www.mate.unlp.edu.ar/practicas/52_7_26092016102641.pdf.)

Problema 254. En la atmósfera terrestre existe un gradiente térmico variable en función


de la altitud con respecto al nivel del mar. Para los primeros 12000 metros de altura
sobre el nivel del mar, se calcula que la temperatura disminuye 1◦ C por cada 154 m de
altura (a mayor altura, menor temperatura).

80
a) Determinen, dando su fórmula y su dominio, la función T que da la temperatura
T (x) para cada altura x sobre el nivel del mar, en un lugar donde la temperatura
del nivel del mar es de 18◦ C.
b) La altura adecuada39 para practicar saltos de paracaidismo es de unos 4000 m, y
debe abrir el paracaídas 1000 metros antes de llegar al suelo. Supongamos que un
avió sobrevuela a dicha altura en el lugar mencionado en el ítem anterior, y que en
ese momento un paracaidista salta al vacío. Si consideramos que su caída libre no
está afectada por ningún rozamiento, la altura en función del tiempo está dada por
h(t) = −5t2 + 4000 donde t es el tiempo medido en segundos.
El paracaidista lleva, entre su equipo, un termómetro. ¿Qué temperatura marcará al
momento de abrir el paracaídas?
c) Determinen, dando su fórmula y su dominio, la función que da la temperatura para
cada instante de tiempo, desde el momento en el que el paracaidista salta, hasta que
llega al suelo.
d ) Grafiquen las tres funciones involucradas en los ítems anteriores. Expliquen si tiene
sentido graficarlas en un mismo sistema de ejes cartesianos.

Problema 255. Determinen las funciones f y g, tales que h(x) = f [g(x)]


√ q
a) h(x) = x2 − 2 c) h(x) = x+1 3
1 x+1
b) h(x) = x3 −5
d ) h(x) = (x+1)2 +2

Problema 256. Consideren las funciones dadas en cada ítem y calculen f ◦ g.


x+3
a) f (x) = 3x + 2 g(x) =
2x + 1
1 2x − 1
b) f (x) = g(x) =
2x − 1 2x + 1
x+2 √
c) f (x) = g(x) = x
2x + 1
d ) f (x) = sen2 x g(x) = cot2 (5x)

Problema 257. Consideren las funciones del item (a) del problema anterior. Calculen
g ◦ f . ¿Coincide la función obtenida con f ◦ g?

Problema 258. Consideren las funciones del Problema 256.. Hallen el dominio de g de
manera que quede adecuadamente definida la composición f ◦ g.

3.6. Función inversa

Definición

Decimos que la función inversa de f es f −1 si se verifica que: f (x) = y ⇒ f −1 (y) = x.


Análogamente, si la función f está determinada por todos los pares (x; y), entonces la
función inversa de f (es decir, f −1 ) está determinada por todos los pares (y; x).

Problema 259. Obtengan la inversa de cada una de las siguientes funciones.

39
Si fuera mayor, es posible que se necesiten tubos de oxígeno para respirar, y si fuera menor, podría
suceder que no alcance el tiempo para abrir el paracaídas.

81
a) y = x + 2 1
h) y =
x+3 x
b) y =
x−2 1
i) y =
c) y = 2x − 3 2x − 1
d ) y = 3 − 2x j ) y = e2x+1
e) y = x2 √
k) y = x
f ) y = x2 − 6x + 4 con x ≥ 3

g) y = x2 − 6x + 4 con x ≤ 3 l) y = 3 x − 1

Resolución. Resolvemos, a modo de ejemplo, los ítems (a) y (b).


La función f está definida como el conjunto de todos los pares ordenados (x; y) tales que
y = x + 2. Entonces la función inversa de f es aquella formada por todos los pares (y; x)
que cumplen dicha condición.
Para encontrar el conjunto de todos los pares (x; y) que pertenecen a f −1 , sustituimos
x ↔ y. Así, f −1 (x) = {(x; y) : x = y + 2}. Despejamos la variable dependiente y de la
ecuación y resulta:
f −1 (x) = {(x; y) : y = x − 2}
Entonces f −1 (x) : y = x − 2.
Como conclusión, podemos sintetizar la siguiente regla práctica para hallar la inversa de
una función.

Reemplazamos la y por x y todas las x por y.


Despejamos y de la ecuación obtenida.

Resolvemos, utilizando la regla práctica, el ítem (i).


x+3
f (x) : y =
x−2
Reemplazamos la y por x y todas las x por y.
y+3
x=
y−2
Despejamos y de la ecuación obtenida.

x(y − 2) = y + 3

xy − 2x = y + 3
xy − y = 3 + 2x
y(x − 1) = 3 + 2x
3 + 2x
y=
x−1
3 + 2x
Luego, f −1 (x) = .
x−1

Para verificar que la función obtenida es la inversa de la función dada, podemos tomar un
punto de la función f (para ello elegimos un valor cualquiera del dominio de f y lo
reemplazamos en la fórmula). Por ejemplo:

Elegimos x = 7.
Calculamos f (7):
7+3 10
f (7) = = =2
7−2 5

82
Entonces el par (7; 2) pertenece a la función f .
Luego, el par (2; 7) pertenece a la función f −1 .
Si la fórmula obtenida es correcta, debe verificar que f −1 (2) = 7. La ponemos a prueba:
3+2·2 3+4
f −1 (2) = = =7
2−1 1
Esto no nos asegura que la fórmula obtenida esté bien; pero si nos hubiera dado un
número distinto de 7, entonces tendríamos la seguridad de que está mal.

Problema 260. Consideren las funciones f y f −1 de los ítems (a) y (b) del problema
anterior.

a) Calculen la composición f ◦ f −1 .
b) Discutan el resultado obtenido.
c) Calculen la composición f −1 ◦ f .
d ) Extraigan conclusiones.

Problema 261. Hallen la inversa de la siguiente función



x si x < 1

f (x) = x2 si 1 ≤ x ≤ 4
 √

8 x si x ≥ 4

Problema 262. Representen la función f (x) = ln(x2 − 4) y su inversa en un mismo


sistema de ejes cartesianos.

83
Respuestas

201. Resoluciones
El dominio de las funciones (a), (b), (c) y (d) es el conjunto de los números reales, ya
que son funciones polinómicas. Entonces escribimos:

a) Df = R b) Dg = R c) Dh = R d ) Df = R.

e) El denominador de la función no puede ser cero. Por lo tanto Df = R − {0}.


f ) El denominador no puede ser cero. Entonces planteamos:

x + 2 6= 0 → x 6= −2

Por lo tanto Dg = R − {−2}.


g) El denominador no puede ser cero, entonces:

x2 − 1 6= 0 → (x − 1)(x + 1) 6= 0 → x 6= 1 ∧ x 6= −1

Por lo tanto Dh = R − {−1; 1}.

Comentario: en este problema notamos -una vez más- la importancia de saber


factorizar de expresiones algebraicas.
h) En el numerador sen(x) no presenta problemas, entonces planteamos:
5 1 5 1 1 5 2
x − 6= 0 → x 6= → x 6= : → x 6=
2 7 2 7 7 2 35
2
Por lo tanto Df = R − 35 .

Nota: Cuando excluimos del dominio una cantidad finita de elementos usamos
llaves. En estos casos nunca utilizamos paréntesis o corchetes.
i) Planteamos:
sen(x) 6= 0 → sen(x) 6= kπ, ∀k ∈ Z
Comentario: f (x) = sen(x) se anula (es decir que vale cero) para cualquier múltiplo
de π. Por lo tanto Df = R − {kπ} con k ∈ Z.

j ) La función y = x tiene una raíz de índice par, por lo tanto su argumento tiene que
ser mayor o igual que cero. Planteamos:

x≥0

Entonces Df = [0; +∞).



k ) La función y = 2x + 9 tiene una raíz de índice par, por lo tanto su argumento
tiene que ser mayor o igual que cero. Planteamos:
−9
2x + 9 ≥ 0 → 2x ≥ −9 → x ≥
2
Entonces Df = − 92 ; +∞ .
 

l ) Planteamos:
3 3 3 3
−7x +≥ 0 → −7x ≥ − → x ≤ − : (−7) → x ≤
7 7 7 49
3

Entonces Df = −∞; 49 .

84

3
m) La función y = x tiene una raíz de índice 3, que es impar, por lo tanto escribimos
Df = R.

4
n) La función y = x3 tiene índice par, entonces planteamos:

x3 ≥ 0 → x ≥ 0 → x ≥ 0
3

Por lo tanto Df = [0; +∞)


ñ) La fórmula y = log(x) corresponde a la función logaritmo en base 10, entonces su
argumento tiene que ser mayor que cero, entonces planteamos:

x>0

Por lo tanto Df = (0; +∞)


o) La fórmula y = ln(x) corresponde a la función logaritmo en base e, entonces su
argumento tiene que ser mayor que cero, entonces planteamos:

x>0

Por lo tanto Df = (0; +∞)


p) Para y = ln(4x − 1), planteamos:
1
4x − 1 > 0 → 4x > 1 → x >
4
1

Entonces Df = 4 ; +∞
q) Para y = log2 (−x + 3), planteamos:

−x + 3 > 0 → −x > −3 → x < 3

Entonces Df = (−∞; 3).


√ 
r ) Para y = ln − 23 x + 11 , planteamos:

√ √ √
 
3 3 3 2 11
− x + 11 > 0 → − x > − 11 → x < − 11 : − →x<
2 2 2 3
 √ 
Por lo tanto Df = −∞; 2 311 .
1
s) Para y = ln(x) , tenemos que considerar que el logaritmo está en el denominador.
Entonces planteamos que el logaritmo tiene que tener argumento mayor que cero y
hallamos cuándo este se anula, ya que ese valor también lo excluimos del dominio de
la función. Proponemos dos condiciones:
El denominador tiene que ser distinto de cero.

ln(x) 6= 0 → x 6= e0 → x 6= 1

El argumento del logaritmo tiene que ser mayor que cero.

x>0

Entonces el dominio de la función surge de considerar simultáneamente los dos


resultados hallados. Por lo tanto: Df = (0; 1) ∪ (1; +∞).

−5x+0,8·log3 (38x)
t) Para y = −x2 +1
, planteamos las siguientes condiciones:
El radicando de la raíz cuadrada tiene que ser mayor o igual que cero.
8
−5x + 0, 8 ≥ 0 → −5x ≥ −0, 8 → x ≤ −0, 8 : (−5) → x ≤
50

85
El argumento del logaritmo tiene que ser mayor que cero.

38x > 0 → x > 0 : 38 → x > 0

El denominador tiene que ser distinto de cero.

−x2 + 1 6= 0 → 1 6= x2 → x 6= 1 ∧ x 6= −1

Entonces el dominio de la función surge de considerar simultáneamente los tres


8

resultados hallados. Por lo tanto: Df = 0; 50 .

202. Resoluciones

a) La función y = −5x en [2; 8] tiene dominio, Df = [2; 8], ya que es polinómica (no
presenta restricciones).
b) La función f (x) = cos(x) en [0; π] tiene dominio, Df = [0; π], ya que no presenta
denominadores, raíces de índice par ni logaritmos.
x
c) La función g(x) = x+2 en [−1; 2] tiene dominio, Dg = [−1; 2], ya que el único valor
que anula el denominador es x = −2, pero no pertenece al intervalo indicado en el
enunciado.
x
d ) La función h(x) = x+2 en [−3; 10]. Por lo tanto su dominio es
Dh = [−3; −2) ∪ (−2; 10], ya que el denominador tiene que ser distinto de cero.
Entonces planteamos y resolvemos: x + 2 6= 0 → x 6= −2.
1
e) La función y = x3 −x
en [−9; +∞) tiene denominadores, por lo tanto planteamos y
resolvemos:
x3 − x 6= 0
x(x2 − 1) 6= 0
x(x − 1)(x + 1) 6= 0
x 6= 0 ∧ x 6= −1 ∧ x 6= 1
Por lo tanto el dominio resulta ser Df = [−9; −1) ∪ (−1; 0) ∪ (0; 1) ∪ (1; +∞) .
f ) La función f (x) = ln(−x + 8) en (−∞; 7], presenta un logaritmo, por lo tanto su
argumento tiene que ser mayor que cero. Entonces planteamos y resolvemos:

(−x + 8) > 0 → 8 > x

Por lo tanto Df = (−∞; 7].

203 - Resoluciones.

a) Corresponden a funciones valor absoluto y como no presentan restricciones, en


ambos casos Df = R.
b) Ídem anterior.
c) Es una función a tramos y cada una de las expresiones dadas no presenta,
denominadores, raíces de índice par o logaritmos. Atención que x = 1 no pertenece
al dominio de la función, ya que se propone x > 1 ∧ x < 1 . Por lo tanto
Dg = R − {1}.
d ) Para el primer tramo de la función planteamos y resolvemos:

x − 3 6= 0 → x 6= 3

y como consideramos x > 2, entonces tenemos que excluir el valor hallado.


Por lo tanto (2; 3) ∪ (3; +∞) Para el segundo tramo planteamos y resolvemos:

x + 3 6= 0 → x 6= −3

86
y como consideramos x < 2, entonces tenemos que excluir el valor hallado.
Por lo tanto (−∞; −3) ∪ (−3; 2) Por último tenemos que notar que en la función que
nos proponen x = 2 no pertenece al dominio de la función, ya que sólo se indica
x < 2 y x > 2.
Por lo tanto Df = (−∞; −3) ∪ (−3; 2) ∪ (2; 3) ∪ (3; +∞).
Otra forma de escribir lo mismo y quizás más fácil es: Df = R − {−3; 2; 3}.
e) El primer tramo de la función es de dominio vacío, ya que el argumento del
logaritmo tiene que ser mayor que cero y esto no se cumple. El segundo tramo de la
función es polinómico y como nos indican que vale para x ≥ 0, entonces el dominio
es Dg = [0; +∞).

204 - Resolución.

a) Como el lado de un cuadrado tiene que ser positivo entonces Df = (0; 5).
b) El lado de un rectángulo tiene que ser positivo, es decir que x > 0. Además ningún
lado puede ser mayor que 50 (dado que superaría los 100 centímetros de perímetro).
Entonces x < 50. Luego, el dominio es Df = (0; 50).
c) El lado de un rectángulo tiene que ser positivo, entonces Df = (0; +∞).
d ) La única restricción es que los lados no deben ser negativos, entonces
Df = [−8; +∞).

205. Tanto f + g como f · g son pares.


206. Si: la función f (x) = 0. La justificación queda a cargo del alumno.
207. a) F b) V c) F d) V e) F f ) V
208.

a) Impar c) Ninguna e) Par


b) Impar d ) Par f ) Par e impar.

209. Respuestas
1)

a) Raíz: x = − 83 ; C + = − 38 ; +∞ ; C − = −∞; − 83 .
 

b) No tiene.
c) A cargo del alumno.
d ) Es creciente en todo su dominio.

2)

a) Raíz: x = 0; C + = (−∞; 0) ; C − = (0; +∞).


b) Es impar.
c) A cargo del alumno.
d ) Es decreciente en todo su dominio.

3)

a) No tiene raíces. C + = R.
b) Es par.
c) A cargo del alumno.
d ) Es creciente en el intervalo (0; +∞) y decreciente en (−∞; 0).

4)

87
a) Raíces: x1 = −2; x2 = 4; C + = (−∞; −2) ∪ (4; +∞); C − = (−2; 4).
b) No tiene.
c) A cargo del alumno.
d) Es creciente en el intervalo (1; +∞) y decreciente en (−∞; 1).
5)
a) Raíces x1 = 3; x2 = −1; C + = (−∞; −1) ∪ (3; +∞); C − = (−1; 3).
b) No tiene.
c) A cargo del alumno.
d) Es creciente en el intervalo (1; +∞) y decreciente en (−∞; −1).
6)
Raíz: x = − 32 ; C + = −1; − 23 ; C − = −∞; −1) ∪ − 23 ; +∞ .
 
a)
b) No tiene.
c) A cargo del alumno.
d) Es decreciente en todo su dominio.
7)
a) Raíces: No tiene. Es positiva en todo su dominio.
b) No tiene.
c) A cargo del alumno.
d) Es creciente en el intervalo (0; +∞) y decreciente en (−∞; 0).
210. y = 3x − 5.
218.
a) Las raíces son x = 2 y x = −3. La ordenada al origen es y = 6.
b) m = 2; b = 6.
c) h(x) = 2x + 5.
 √ √   √ √ 
d ) −3+2 13 ; 2 + 13 , −3−2 13 ; 2 − 13
e) I = −∞; 25

4 ;
f ) Crece en −∞; − 12 y decrece en − 12 ; +∞
 

g) C + = (−3; 2); C − = (−∞; −3) ∪ (2; +∞)


h) Lo pueden verificar con el GeoGebra.
222.
a) 7,25 metros.
b) 11 metros.
c) No es cierto que haya subido durante los dos primeros segundos.
d) Fue lanzada desde un metro de altura.
e) f (3) = 1 significa que a los 3 segundos se encuentra a 1 metro de altura y
f (4) = −19 significa que a los 4 segundos se encuentra a 19 metros debajo el punto
de referencia tomado inicialmente.
f ) 3,065 segundos.
223. a. Si x es el número de árboles que agregamos, y P es l número de naranjas por
árbol, entonces P (x) = 500 − 5x.
b. f (x) = (40 + x)(500 − 5x).
c. 109 árboles.
d. 95 árboles.
229.

88
a) y = a(x2 + x − 6) ∀a 6= 0. d ) y = 3x2 − 6x + 5.
b) y = −4x2 − 4x + 24. e) y = −2s2 + 20x − 28.
c) y = a(x − 1)2 + 2 ∀a 6= 0. f ) y = −12x2 + 50x − 36.

230 - Resolución. Buscamos el dominio de la función. Para ello40 planteamos la


inecuación
2x − 4 6= 0
x 6= 2
Entonces el dominio de la función es Df = R − {2}. Buscamos la raíz de f . Para ello
planteamos la ecuación
f (x) = 0
x+3
=0
2x − 4
x+3=0
x = −3
Luego la raíz de la función es x = −3.
Buscamos la ordenada al origen de la función. Es decir y = f (0). Entonces la ordenada es
0+3
f (0) =
2·0−4
3
f (0) = −
4
Entonces la ordenada al origen es y = − 43 .
La función tiene una asíntota vertical de ecuación x = − dc = − −4
2 = 2 (que es el único
valor de x que no pertenece al dominio), y tiene una asíntota horizontal de ecuación
y = ac = 12 .
Luego, su gráfica es

231. a) f es una función lineal. b) f es una función constante.


232. Las gráficas las pueden revisar con el GeoGebra.

a) D = R − {−1}; Im = R − {2}; Raíz en x = − 23 ; Ordenada en


y = 3. AV : x = −1; AH : y = 2
b) D = R − {−1}; Im = R − {1}; Raíz en x = 1; Ordenada en
y = −1. AV : x = −1; AH : y = 1
40
Sugerimos leer la sección correspondiente.

89
c) D = R − {2}; Im = R − {1}; Raíz en x = 0; Ordenada en
y = 0. AV : x = 2; AH : y = 1
d ) D = R − 41 ; Im = R − 12 ; Raíz en x = −3; Ordenada en
 

y = −6. AV : x = 41 ; AH : y = 21

233.

a) Se puede probar factorizando el numerador y el denominador, y simplificando el


factor (x − 1).
b) Df = R − {1; 2}.
c) Raíz en x = −2 y ordenada en y = −3.
d ) Las asíntotas son las rectas de ecuaciones x = 2; y = 3.
e) Im = R − {3}.
f ) A cargo del alumno (o del GeoGebra).

234. 
−15
 si 0 ≤ x ≤ 10
f (x) = 3x − 270
16 si x ∈ (10; 90)

2
3 x + 60 si 90 ≤ x ≤ 105
235. f (x) = 12 x3 − x2 − 25 x + 3.
146.

a) D = R; Raíces en x = 2 y en x = −4. Ordenada en


y = −8; C + = (−∞; −4) ∪ (2; +∞); C − = (−4; 2).
b) D = R; Raíces en x = 1 y en x = −1. Ordenada en
y = 1; C + = R − {−1; 1}; C − = ∅.
c) D = R; Raíces en x = 2 y en x = −2. Ordenada en
y = −2; C + = (−∞; −2) ∪ (2; +∞); C − = (−2; 2).
d ) D = R; Raíces en x = −1; x = 1 y en x = 0. Ordenada en
y = 0; C + = (−1; 0) ∪ (1; +∞); C − = (−∞; −1) ∪ (−4; 2).

237. b = 331, 4.
238. a) V = 21 I. b) V = 5v.
239 . P = T4 + 80.
240. a) A cargo del alumno. b) Es una recta. c) m = 18704; b = 3340.
241. a) 6seg; b) 176, 4m.
242. a) 2, 07 seg b) 90, 11 N .
243.
a) Ley de Ohm. b) V = 5I. c) Es una función lineal. d) D = Im = (0; +∞).
244. a) I = 12v
R b) Si c) D = I = (0; +∞).
245.

a) Segunda ley de Newton41 . Sí. c) F = 5 m


s · m. Sí.
b) F = 4a. d ) A cargo del alumno.

246.

a) La presión y el volumen.
b) P · V = 2. D = I = (0; +∞).
c) Las asíntotas son las rectas x = 0 e y = 0.
41
O principio de masa.

90
249. y = 3 · 2x .
x
250. P (x) = 200 · 2 3 .
t
251. P (t) = 10000 · 3 50 ; p(100) = 90000; p(150) = 270000; p(200) = 810000 Sí, es
posible. A cargo del alumno.
252. a) 60. b) 0, 059. c) 5, 72 · 10−5 . d) 5, 45 · 10−11 .
253. a) D = (0; +∞); d) h = 10−p .
255.
√ √ x+1
a) f (x) = x; g(x) = x2 + 2 c) f (x) = x; g(x) = 3
b) f (x) = x1 ; g(x) = x3 − 5 d ) f (x) = x
x2 +1
; g(x) =x+1

Aclaración: Existen infinitos pares de funciones f y g que verifican el enunciado. Las


respuestas proporcionadas son sólo un ejemplo.
259.

a) y = x − 2 c) y = 21 x + 32 f) y = 3+ 5+x
d ) y = − 21 x + 3
3 + 2x √ 2 √
b) . e) y = x g) y = 3 − 5+x
x−1

261. 
x


si x < 1
−1
f (x) = x si 1 ≤ x ≤ 16
 x2

64 si x ≥ 16

91
***

Límites y
continuidad
4. Límites y continuidad

El Límite es el objeto matemático mediante el cual nos adentramos en el Cálculo


diferencial e integral. El límite está presente en la definición de la continuidad de una
función, como así también de las asíntotas, la derivada y la integral. Y como es de esperar
se presentan diversas situaciones en las que tenemos que calcular límites mediante el uso
de sus propiedades, recursos algebraicos y toda la batería de conocimientos previos de
que disponemos, para dar solución a los problemas propuestos.

2
Problema 301. Investiguen el comportamiento de la función f (x) = x x−x−2
2 −4 para valores
de x cercanos a 2. (Sugerencia: completen la siguiente tabla y analicen la situación;
acompañen el trabajo con un gráfico de la función, que pueden hacer con GeoGebra).

x 1,9 1,99 1,999 1,9999 1,99999 2,1 2,01 2,001 2,0001 2,00001
f (x)

Problema 302. Repitan el problema anterior con valores de x cercanos a 3 y con valores
de x cercanos a (−2). Extraigan conclusiones y comenten los resultados obtenidos con los
compañeros.
x+1
Problema 303. Comparen la función del problema anterior con la función g(x) = x+2 .

a) ¿Coinciden en todos los puntos de su gráfica?


b) Grafiquen f y g en GeoGebra. ¿Son idénticas sus gráficas?
c) Busquen en GeoGebra la intersección de f con la recta x = 2.
d ) Busquen en GeoGebra la intersección de g con la recta x = 2.
e) Discutan los resultados obtenidos en los dos últimos ítems.
f ) Busquen en forma analítica las intersecciones mencionadas en los ítems anteriores.
g) Discutan los resultados obtenidos.

Decimos que un límite es indeterminado si suceden alguna de las situaciones siguientes, a


las que llamamos indeterminaciones. (Aclaración: existen otras indeterminaciones que
no evaluaremos durante esta cursada):

Cociente de infinitésimos. Si el límite es un cociente de dos funciones y ambas


tienden a cero en el valor dado, entonces decimos que la indeterminación es un
cociente42 de infinitésimos43 .
Cociente de infinitos. Si el límite es un cociente de dos funciones y ambas tienden
a infinito en el valor dado, entonces decimos que la indeterminación es un cociente
de infinitos.
Límites exponenciales. Si el límite es una potencia de funciones tales que la base
tiende a 1 y el exponente tiende a infinito, entonces la indeterminación es
exponencial.

Más adelante, mostraremos cómo resolver un límite indeterminado.

4.1. Álgebra de límites

Definición. Un límite es determinado si no encontramos alguna de las indeterminacio-


nes mencionadas.
42
Resultado de la división.
43
Expresiones que tienden a cero.

93
Algunas propiedades de los límites determinados

a) Si k es una constante, entonces:


lı́m k = k
x→a

b) Límite de la función identidad:


lı́m x = a
x→a

c) Si k es una constante, entonces:

lı́m [k · f (x)] = k · lı́m f (x)


x→a x→a

d ) Si f y g son dos funciones definidas44 en x = a entonces:

lı́m [f (x) ± g(x)] = lı́m f (x) ± lı́m g(x)


x→a x→a x→a

e) Si f y g son dos funciones definidas en x = a entonces:

lı́m [f (x) · g(x)] = lı́m f (x) · lı́m g(x)


x→a x→a x→a

f ) Si f (a) 6= 0 y g(a) = 0 entonces:

f (x)
lı́m =∞
x→a g(x)

g) Si lı́m f (x) ∈ R y lı́m g(x) = ∞ entonces:


x→a x→a

f (x)
lı́m =0
x→a g(x)

Resolvemos un límite a modo de ejemplo:

L = lı́m [x · (2x + 5)]


x→3

Aplicamos la propiedad e)
L = lı́m x · lı́m (2x + 5)
x→3 x→3

Para resolver el primer límite aplicamos la propiedad b), y para el segundo aplicamos las
propiedades a), c) y d)
L = 3 · (2 · 3 + 5) = 33
A continuación mostramos cómo resolver las indeterminaciones que evaluaremos en esta
unidad.

44
Es decir, que podemos reemplazar la variable x por el número a en las funciones f y g.

94
Cociente de infinitésimos

Evaluamos tres casos:

Primer caso

Si lı́m fg(x)
(x)
siendo f y g dos polinomios tales que f (a) = g(a) = 0.
x→a
Para salvar la indeterminación simplificamos la expresión. Esto lo hacemos
dividiendo45 numerador y denominador por (x − a).
Por ejemplo:
x3 − 5x − 12
L = lı́m 2
x→3 2x − 7x + 3

En primer lugar, observamos que tanto el numerador como el denominador tienden


a cero, ya que
33 − 5 · 3 − 12 = 0
y
2 · 32 − 7 · 3 + 3 = 0
Entonces dividimos numerador y denominador por (x − 3) y obtenemos

x2 + 3x + 4
L = lı́m
x→3 2x − 1
Aplicamos las propiedades de los límites y obtenemos

32 + 3 · 3 + 4
L=
2·3−1
Entonces
22
L=
5

Segundo caso

Si lı́m fg(x)
(x)
siendo f o g funciones con raíces cuadradas tales que f (a) = g(a) = 0.
x→a
Para salvar la indeterminación racionalizamos la expresión irracional y luego
procedemos como el caso anterior.
Por ejemplo: √
3x + 10 − 4
L = lı́m 3
x→2 x − 5x2 + 12

En primer lugar, observamos que tanto el numerador como el denominador tienden


a cero. Como el numerador tiene una raíz cuadrada, multiplicamos al numerador y
al
√ denominador por la expresión conjugada del numerador, es decir, por
3x + 10 + 4. Obtenemos
√ √
3x + 10 − 4 3x + 10 + 4
L = lı́m 3 2
·√
x→2 x − 5x + 12 3x + 10 + 4
Aplicamos la propiedad distributiva en el numerador pero no en el denominador.
√ 2 √ √
3x + 10 − 4 · 3x + 10 + 4 · 3x + 10 − 16
L = lı́m √ 
x→2 (x3 − 5x2 + 12) · 3x + 10 + 4
45
En virtud del teorema del resto, sabemos que el resto de la división es cero.

95
Cancelamos los términos opuestos en el numerador y también la raíz cuadrada con
el exponente (dado que el argumento es positivo).
3x + 10 − 16
L = lı́m √
x→2 (x3 − 5x2 + 12) · ( 3x + 10 + 4)
Podemos reescribir el límite
3x − 6 1
L = lı́m ·√
x→2 x3 − 5x2 + 12 3x + 10 + 4
Notamos que obtenemos un producto en el que el primer factor es una
indeterminación similar a la vista en el primer caso, y el segundo factor es
determinado (tiende a 81 ). Entonces queda por resolver la indeterminación
dividiendo numerador y denominador por (x − 2). La resolución queda a cargo el
alumno.

Tercer caso
Si lı́m fg(x)
(x)
siendo f o g funciones racionales tales que f (a) = g(a) = 0.
x→a
Sumamos o restamos las expresiones racionales y luego procedemos como el caso
anterior.
Por ejemplo:
1
−1
L = lı́m 3 x 4
x→4 x + x − 68
Restamos en el numerador
4−x
4x
L = lı́m
x→4 x3 + x − 68
Reescribimos la expresión
4−x
L = lı́m
x→4 4x · (x3 + x − 68)
Y obtenemos un límite similar al visto en el primer caso. Entonces podemos dividir
numerador y denominador por (x − 4) y resolver la indeterminación.

Cociente de infinitos

Para salvar la indeterminación dividimos al numerador y al denominador por xn siendo n


el mayor entre todos los exponentes de x, aplicamos la propiedad distributiva de la
división con respecto a la suma y a la resta, y aplicamos la propiedad (g) de los límites
determinados.
Por ejemplo:
3x2 − 2x + 1
L = lı́m
x→∞ 7x − 5x2 − 6
Dividimos al numerador y al denominador por x2 .
3x2 −2x+1
x2
L= lı́m 7x−5x 2 −6
x→∞
x2
Aplicamos la propiedad distributiva de la división con respecto a la suma y a la resta.
3x2
x2
− 2x
x2
+ 1
x2
L= lı́m 7x 5x 2 6
x→∞ − x2 −
x2 x2
Simplificamos y aplicamos las propiedades (g), (c) y (d). Entonces:
3−0+0
L = lı́m
x→∞ 0 − 5 − 0
3
L=−
5

96
4.2. Límites de funciones definidas a tramos

Cuando queremos calcular el límite de una función definida a tramos, con la variable
tendiendo al número a, y la función cambia su fórmula en x = a, debemos calcular los
límites laterales; es decir que tenemos que evaluar qué sucede con f (x) cuando x tiende
al número a por la derecha (es decir con x > a) y cuando x tiende a a por izquierda (es
decir con x < a). Si estos límites laterales son iguales, entonces el límite existe y es igual
a dicho número. Si los límites laterales son diferentes, entonces no existe el límite.
Problema 304. Consideren la función

x + 4 si x < 2

f (x) = x(x + 1) si 2 < x < 3

 3
x si x ≥ 3
y calculen: lı́m f (x) y lı́m f (x).
x→2 x→3

Resolución.
Para calcular el primer límite, debemos evaluar los límites laterales, ya que la función
está definida a tramos y la fórmula cambia en x = 2. Entonces
L− = lı́m f (x) = lı́m (x + 4)
x→2− x→2−
Ya que la fórmula que corresponde a los valores de x menores a 2 es la primera de las
tres. Luego aplicamos las propiedades b), d) y e) y resulta:
L− = lı́m f (x) = lı́m (x + 4) = 2 + 4 = 6
x→2− x→2−

Entonces L− = 6.
L+ = lı́m f (x) = lı́m x(x + 1)
x→2+ x→2+
Ya que la fórmula que corresponde a los valores de x mayores a 2 pero menores a 3 es la
segunda. Luego aplicamos las propiedades a), b), d) y e) y resulta:
L+ = lı́m f (x) = lı́m x(x + 1) = 2(2 + 1) = 6
x→2+ x→2+

Entonces L+ = 6.
Observamos que L+ = L− = 6; entonces afirmamos que
L = lı́m f (x) = 6
x→2
Nota: observamos que no resulta relevante la existencia (o no) de f (2).
Calculamos ahora el segundo límite.
Por las mismas razones, debemos evaluar los límites laterales.
L− = lı́m f (x) = lı́m x(x + 1)
x→3− x→3−
Ya que la fórmula que corresponde a los valores de x menores a 3 pero mayores a 2 es la
segunda. Luego aplicamos las propiedades a), b), d) y e) y resulta:
L = lı́m f (x) = lı́m x(x + 1) = 3 · 4 = 12
x→3− x→3−

Entonces L− = 12.
L+ = lı́m f (x) = lı́m x3
x→3+ x→3+
Ya que la fórmula que corresponde a los valores de x mayores a 3 es la tercera. Luego
aplicamos las propiedades c) y e), y resulta:
lı́m f (x) = lı́m x3 = 27
x→3+ x→3+

Entonces L+ = 27.
Observamos que L+ 6= L− ; entonces afirmamos que no existe lı́m f (x).
x→3

97
Teorema del Sandwich

Si una función f verifica que: h(x) ≤ f (x) ≤ g(x) y lı́m h(x) = lı́m g(x) = L entonces
x→a x→a
lı́m f (x) = L.
x→a

Este teorema resulta útil para calcular, por ejemplo, el siguiente límite:
sen(x)
Problema 305. Calculen el límite L = lı́m x .
x→∞

Resolución. Notamos46 que −1 ≤ sen(x) ≤ 1. Consideramos h(x) = −1 x , g(x) =


−1
x y
sen(x)
g(x) = x . Estas funciones verifican las hipótesis planteadas, ya que

−1 1
lı́m= lı́m =0
x→∞ x x→∞ x

Además h(x) ≤ f (x) ≤ g(x) para los valores en los que sen(x) es positivo, y
g(x) ≤ f (x) ≤ h(x) en los que sen(x) es negativo. Entonces lı́m sen(x)
x = 0.
x→∞

sen(x)
Problema 306. Calculen el límite L = lı́m x .
x→0

Problema 307. Calculen los siguientes límites. Verifiquen con la calculadora el resultado
obtenido en cada uno de ellos.
x2 +x−2 x2 −3x+2
a) lı́m 2 = c) lı́m 2 =
x→1 −2x+1
x x→2 x −4
x4 −6x2 +8x−3 x3 −6x2 +5x
b) lı́m 4 3 = d ) lı́m 4 3 =
x→1 x −2x +2x−1 x→1 −x +x−1
x

1
Respuestas. a) L = ∞ b) L = 2 c) L = 4 d) L = −2

Problema 308. Calculen los siguientes límites. Para ello recuerden los pasos sugeridos:
racionalizar (multiplicando por la expresión conjugada que corresponda), simplificar la
expresión obtenida (pueden emplear el algoritmo de Ruffini o el algoritmo de la división
de polinomios) y aplicar las propiedades. Verifiquen los resultados con la calculadora.
√ √ √ √
3+x− 3 4x+13− 3x+16
a) lı́m √
x
= c) lı́m x2 −x−6
=
x→0 x→3

3 +7−3 3x+6
b) lı́m 2xx3 −1 = d ) lı́m √ =
x→1 x→−2 7− x+51

1 1
Respuestas. a) L = 0 b) L = 3 c) L = 50 d) L = −42

Problema 309. Calculen los siguientes límites. Sugerimos dividir al numerador y al


denominador por un término de la forma xn (eligiendo convenientemente el valor de n).
Verifiquen los resultados obtenidos con la calculadora. Extraigan conclusiones que le
faciliten la deducción del resultado del límite en función de los exponentes de la expresión.
4 q
a) lı́m −2x4x−2x+1 = d ) lı́m 1+4x2
=
x→∞ 2x2 −3x+8 x→∞
x3 −2x2 +x x3 +6x−2
b) lı́m 2x 3 −πx+5 = e) lı́m 1 − 2x4 −12x3 +2x−7
=
x→∞ x→∞

3x3 −x4 −2 2
√ x +x−2
c) lı́m 5x5 −7x2 −2x+6
= f ) lı́m =
x→∞ x→∞ x2 −2x+1


Respuestas. a) L = − 12 b) L = 1
2 c) L = 0 d) L = 2 e) L = 1 f ) L = 1

46
Esto es verificable, por ejemplo, observando la gráfica de la función.

98
Problema 310. Calculen los siguientes límites evaluando primero L+ y L− . Determinen el
resultado del límite (si existe) o justifiquen si no existe.
x2 +x−2
a) lı́m 2x+3 = c) lı́m 2x+3 = e) lı́m 2 =
x→0 x→∞ x→∞ −2x+1
x
1 1 √
b) lı́m 2 x+3 = d ) lı́m 2 x+3 = f ) lı́m x−2=
x→∞ x→−3 x→2

Respuestas.

a) Existe: L = 8. d ) No existe: L+ = ∞; L− = 0.
b) Existe: L = 1. e) Existe: L = 1.
c) No existe: L+ = ∞; L− = 0. f ) No existe el L− .

Problema 311. Calculen lı́m f (x) para:


x→a
(
x2 − 1 si x < 2
a) f (x) = Con a = 2.
x3 − 5(x − 1) si x > 2
(
2(x − 1)2 + 3 si x < 3
b) f (x) = Con a = 3.
(x − 1)2 − 4 si x > 3

Respuestas. a) L = 3 b) L+ = 0; L− = 11. No existe L.

Problema 312. Resuelvan los siguientes límites, salvando la indeterminación


correspondiente. Verifiquen el resultado obtenido empleando la calculadora.
x3 +7x−8 3x+6 sen(πx)
a) lı́m 2 = j ) lı́m √ = r ) lı́m =
x→1 −4x+3
x x→−2 7− x+51 x→0 x
2 x
b) lı́m x x−3x+4 = k ) lı́m 3x2 −7x+23
= s) lı́m =
x→2
2 −4 2
x→∞ 5x +9x−8 x→0 sen(x)
4 2 +7x−35 x2 −1
c) lı́m x −25x
2x2 −8x−10
= l ) lı́m2x3 −176x2 +1
= t) lı́m =
x→5 x→∞ 5x 2 −5x+1000
x→1 x−1
x+2   x2 +x−2
d ) lı́m 5 = m) lı́m 2x2
− x 3
= u) lı́m =
x→−2 x −15x+2 2x−4 x2 −6
2
x→−2 x +5x+6
x→∞
x2 −3 1
− a1
e) lı́m
√ 18−2x4
= n) lı́m ax2 +bx+c
=
x→ 3 x3 −1 v ) lı́m x
=
x→∞ x→a x−a
√ √
5x2 −10x−15 2x5 +πx5 −7
f ) lı́m x2 −5x+6
= ñ) lı́m 5 4 = w) lı́m 2x−2 =
x→3 x→∞ 7x −3x +1 x→4 x −16

lı́m 25x+4−3 lı́m 1 + x+5

g) 5 = o) x = x ) lı́m 4x
=
x→1 x +x −2 x→∞ x→0 3x−1
√ √
h) lı́m x− a = p) lı́m sen(x) = y) lı́m (−2 − x2 − x) =
x→a x−a x→0 5x x→0
√ √ √
sen(5x)
i) lı́m 4x+13−
x 2 −x−6
5x+10
= q) lı́m 7x = z ) lı́m 4x + 5 =
x→3 x→0 x→1

Respuestas.
1 2+π 1
a) L = −5 h) L = √
2 a
ñ) L = 7 u) L = 5
b) L = ∞ i) L= 1
− 50 o) L = 2 v ) L = − a12
257 1
c) L = 12 j) L = −42 p) L = 5 1
w) L = 32
1 5
d) L = 65 k) L = 35 q) L = 7
1 x) L = 0
e) L = − 12 l) L=∞ r) L = π
f ) L = 20 m) L=2 s) L = 1 y) L = −2
5
g) L = 42 n) L=0 t) L = 2 z) L = 3

99
Problema 313. Expliquen por qué no existe el límite cuando x tiende a ∞ de la función
y = cos(x).

Problema 314. Tracen la gráfica de una función y = f (x) que sea negativa para x < 2,
positiva para x > 2 y tal que no exista lı́m f (x).
x→2

Problema 315. Tracen la gráfica de una función y = f (x) que sea negativa para x < 2,
positiva para x > 2 y tal que exista lı́m f (x). ¿Cuánto debe valer ese límite?
x→2

Problema 316. Analicen la existencia de los siguientes límites (Sugerencia: calculen los
límites laterales y si es posible, realicen un gráfico).
x 2x+1
a) lı́m = e) lı́m =
x→0 |x| x→1 x−1
1
b) lı́m 2 = 1
x→1 (x−1) f ) lı́m 2 x =
√ x→0
c) lı́m x=
x→0
1

x2
√ 
d ) lı́m = g) lı́m x2 +1
+ 9 − x2 =
x→0 x x→3

Respuestas.

a) @L; L+ = 1; L− = −1. e) L = ∞.
b) L = ∞.
f ) @L; L+ = ∞; L− = 0.
c) @L; L+ = 0 pero no existe L− .
d) L = ∞. g) L− = 9
10 pero no existe el límite L+ .

Problema 317. Construyan un mapa de los límites y las indeterminaciones que hemos
trabajado hasta este momento. Anoten en el resumen los pasos a seguir, los tipos de
indeterminaciones y sus estrategias de resolución.

4.3. ¿Cómo evaluamos la continuidad de una función?

A continuación veremos cómo evaluar la continuidad de una función en un punto.


En el ejemplo que proponemos a continuación, tenemos que determinar si la función es o
no continua en un valor de x dado47 .
Para ello seguiremos los siguientes pasos:

a) Evaluamos la existencia de la función en el valor dado de x. Es decir, evaluamos la


existencia de f (a). Si existe, entonces calculamos F = f (a).
b) Evaluamos la existencia de los siguientes límites:

L+ = lı́m f (x)
x→a+

L− = lı́m f (x)
x→a−

Si sucede que ambos límites son iguales, entonces denominamos

L = lı́m f (x)
x→a

En el caso de las funciones definidas a tramos es importante calcular los limites


laterales observando cuidadosamente la fórmula que corresponde a cada tramo del
dominio de la función.
47
Más adelante veremos cómo determinar los posibles puntos de discontinuidad.

100
c) En caso de que existan y sean finitos los valores de L y F entonces nos fijamos si
F = L.

¿Qué puede suceder?

Si se cumplen (a), (b) y (c) entonces la función es continua en x = a.


Si L → ∞ entonces la función tiene una asíntota vertical con ecuación x = a. En
este caso decimos que la función presenta una discontinuidad esencial.
Si L+ 6= L− entonces la función tiene un salto. En este caso, también decimos que la
función presenta una discontinuidad esencial.
Si L+ = L− pero L 6= F (o bien si no existe F ), entonces la función tiene un
agujerito en x = a. En este caso, decimos que la función presenta una discontinuidad
evitable.
x−2
Problema 318. Decidan si la función y = x2 −4
es continua en x = 2.

Resolución.

a) ¿Existe f (2)?
No, porque x = 2 anula el denominador y no está definida la división por cero.
Entonces no existe F . En otras palabras x = 2 no pertenece al dominio de la función.
b) ¿Existen L+ y L− ? Sí; en ambos casos, los límites son iguales a 41 . Entonces L = 14 .
Aclaración: el cálculo de los límites queda a cargo del alumno.
c) ¿Se verifica F=L? No, ya que no existe F .

 f es discontinua evitable en x = 2 (es decir que


Por lo visto anteriormente, la función
tiene un agujerito en el punto 2; 14 .

Problema 319. Determinen si la función


(
2x + 5 si x < 5
f (x) =
x2 − 1 si x ≥ 5

es continua en x = 5.

a) ¿Existe f (5)? Sí, dado que el segundo tramo de esta función incluye el valor x = 5.
Entonces calculamos f (5) = 52 − 1 = 24. Entonces F = 24.
b) ¿Existe el límite? Como la función está partida, precisamente en el valor en el cual
debemos calcular el límite, entonces analizamos los límites laterales.

lı́m f (x) = lı́m (2x + 5) = 2 · 5 + 5 = 15


x→5− x→5−

Entonces L− = 15

lı́m f (x) = lı́m (x2 − 1) = 52 − 1 = 24


x→5+ x→5+

Entonces L+ = 24.
Observamos que L+ 6= L− por lo tanto no existe L.
c) No tiene sentido, ya que no existe L.

Según lo visto, la función es discontinua esencial en x = 5 y presenta un salto.

101
¿Dónde evaluamos la continuidad de una función?

En el caso que nos pidan evaluar la continuidad de una función, pero no nos indiquen en
qué punto o puntos debemos hacerlo, entonces tenemos que evaluar la continuidad en
todos los candidatos a puntos de discontinuidad.
¿Cuáles son los candidatos a puntos de discontinuidad?
Son los valores de x que cumplan con al menos, una de las siguientes condiciones:

Los que no pertenecen al dominio.


Si la función es una función a tramos, entonces cada uno de los valores en los cuales
la función se parte es un candidato a punto de discontinuidad.

Por ejemplo, en la función


(
1
x2 −4
si x < 1
f (x) =
x2 + 1 si x ≥ 1

Los candidatos a puntos de discontinuidad son

x = −2 ya que no pertenece al dominio.


x = 1 ya que la función está partida en x = 1.

Notemos que x = 2 no es un candidato, ya que -según la definición de la función-


f (2) = 22 + 1.
Una vez determinados los valores de x que son candidatos a puntos de discontinuidad,
procedemos con los pasos (a), (b) y (c) del análisis de la continuidad de la función.

Propiedades y consideraciones
La suma de dos funciones continuas es una función continua.
La resta de dos funciones continuas es una función continua.
El producto de dos funciones continuas es una función continua.
f
El cociente g de dos funciones continuas (con g 6= 0) es una función continua.
La composición de dos funciones continuas es una función continua.
Una función es continua en un intervalo si es continua en cada uno de sus puntos.
Todas las funciones polinómicas son continuas.

Problema 320. Determinen los posibles puntos de discontinuidad en las siguientes


funciones. Para cada punto, analicen si la función es discontinua o no. En caso afirmativo,
clasifiquen la discontinuidad.
 3
x−2
a) f (x) = x2 −4x+5 x −64
 x2 −16 si x < 3

d ) f (x) = 3 si 3 ≤ x < 6
√ 
 x
b) f (x) = x+2 2 si x > 6
x−3
2x−π
e) f (x) = (x2 +1)(x2 −1)
1
( (
x
si x < 2
x2 +1
( 12 ) x si x > 0
c) f (x) = f ) f (x) =
(x − 1)2 − 35 si x > 2 3x si x ≤ 0

Respuestas.

a) Es continua en todos sus puntos.


b) No existe f (3), no existe el límite. Es discontinua esencial en x = 3

102
c) Discontinua evitable en x = 2. Se redefine la función asignado para f (x) = 0, 4 si
x = 2.
d ) A cargo del alumno.
e) Discontinua esencial en x = 1 y x = −1.
f ) Discontinua esencial con salto finito en x = 0.

Problema 321. Hallen el valor de a para que f (x) sea continua en x = 2.

a) c)
(
ea(x−2) si x ≤ 2
(
x3 −2x2 −3x+6
x2 −x−2
si x > 2 f (x) =
f (x) = x + 2a si x > 2
a si x ≥ 2

b) d)
( (
x2 + ax si x ≤ 2 x2 − 1 si x ≤ 1
f (x) = f (x) = x+2a
a − x2 si x > 2 ax+1 si x > 1

4.4. Asíntotas

Definiciones

Una recta de ecuación y = b es asíntota horizontal de una función f si:

lı́m f (x) = b
x→∞

Una recta de ecuación x = a es asíntota vertical de una función f si:

lı́m f (x) = ∞
x→a

Una recta de ecuación y = mx + b es asíntota oblicua de una función f si:

lı́m [f (x) − (mx + b)] = 0


x→∞

Para hallar los valores de m y b aplicamos las propiedades de los límites.

lı́m f (x) = lı́m (mx + b)


x→∞ x→∞

Dividimos ambos miembros por x:


 
f (x) b
lı́m = lı́m m +
x→∞ x x→∞ x

f (x)
lı́m =m
x→∞ x
Reemplazamos en la definición el valor de m obtenido48 y despejamos b, luego
restamos mx en ambos miembros:

lı́m [f (x) − mx] = lı́m b


x→∞ x→∞

lı́m [f (x) − mx] = b


x→∞

48
Notamos que, si m = 0 entonces la asíntota es horizontal.

103
Problema 322. Hallen las ecuaciones de las asíntotas de las siguientes funciones.
Observación: pueden tener asíntotas horizontales y/o verticales. Verifiquen las ecuaciones
halladas graficando la función en el GeoGebra.
x2 −2
a) f (x) = x−1
x2 −3x+2
f ) f (x) = 5−x k ) f (x) = x+1
x
b) f (x) = x2 −4 g) f (x) = 32 5x2 +1
6x−3x2 l ) f (x) = x−4
c) f (x) = 1
5 + 2x+3 h) f (x) = 3x − 1
5x−1 i) f (x) = 4 m) f (x) = log(2x − 6)
d ) f (x) = 2x−3
sen(x)
e) f (x) = 5x j ) f (x) = x n) f (x) = ln(x2 − 4)

Respuestas.

a) x = 2; y = 0 f) y = 0 k ) x = 1; y = x − 1
b) x = 0; y = − 13 g) y = 0
l ) x = 4; y = 5x + 20
c) x = − 23 ; y = 5 h) No tiene.
3 5 m) x = 3
d) x = 2; y = 2 i) No tiene.
e) y = 0 j) y = 0 n) x = −2; x = 2

Teorema de Bolzano. Sea f una función continua en el intervalo [a; b] tal que
f (a) · f (b) < 0 entonces existe al menos un c ∈ (a; b) tal que f (c) = 0.

Problema 323. Analicen en grupos el significado de este teorema. Propongan ejemplos.

Problema 324. Encuentren cuatro intervalos disjuntos en cada uno de los cuales la
función f (x) = 2x4 − 14x2 + 14x − 1 tenga una raíz.

Problema 325. ¿Existen números a, b, c y d reales (con a 6= 0) tales que la ecuación


ax3 + bx2 + cx + d = 0 no tenga solución?. Justifiquen su respuesta.

Problema 326. Demuestren que la ecuación x3 + x2 − 9x + 2 = 0 tiene al menos una raíz


mayor que 0 y menor que 1.

Problema 327. La función y = tan(x) toma valores de distinto signo en los extremos del
intervalo [ π4 ; 3π
4 ]. Sin embargo, no existe c en dicho intervalo tal que f (c) = 0 ¿Contradice
esto el teorema de Bolzano?

Resolución.
La función f (x) = x3 + x2 − 9x + 2 es continua en [0, 1]. Además, f (0) = 2 > 0 y
f (1) = −6 < 0. Por el teorema de Bolzano, existe un c ∈ (0, 1) tal que f (c) = 0, o de
forma equivalente: la ecuación dada tiene al menos una raíz mayor que 0 y menor que 1.

Problema 328. Demuestren que la ecuación x − cos(x) = 0 tiene al menos una solución
en el intervalo (0, π).

Resolución.
La función f (x) = x − cos(x) es continua en R, por tanto lo es en [0, π]. Además,
f (0) = −1 < 0 y f (π) = π + 1 > 0. Por el teorema de Bolzano, existe un c ∈ (0, π) tal que
f (c) = 0, o de forma equivalente: la ecuación dada tiene al menos una raíz en (0, π).

Problema 329. (Opcional) Sean a, b ∈ R tales que 0 < a < 1 y b > 0. Demuestren que la
ecuación x = a · sen(x) + b tiene al menos una raíz positiva menor o igual que a + b.

Resolución.
La función f (x) = x − a · sen(x) − b es continua en R, por tanto lo es en [0, a + b].
Además, f (0) = −b < 0, f (a + b) = a(1 − sen(a + b)). Si sen(a + b) = 1, entonces

104
f (a + b) = 0 y a + b es la raíz pedida. Si sen(a + b) 6= 1, entonces sen(a + b) < 1 y al ser
0 < a < 1 : f (a + b) = a(1 − sen(a + b)) > 0. Por el teorema de Bolzano, existe un
c ∈ (0, a + b) tal que f (c) = 0, y c es la raíz pedida.

Problema 330. (Opcional) Demuestren que toda ecuación polinómica real de grado impar
tiene al menos una raíz real.

Resolución.
Sea f (x) = an xn + an−1 xn−1 + · · · + a1 x + a0 una función polinómica real de grado n
impar. Supongamos que an > 0. Entonces:
(i) lı́m f (x) = lı́m xn (an + an−1 x−1 + · · · + a1 x−1 + a0 x−n ) = −∞.
x→−∞ x→∞
(ii) lı́m f (x) = lı́m xn (an + an−1 x−1 + · · · + a1 x−1 + a0 x−n ) = +∞. Por las
x→+∞ x→∞
definiciones de estos límites, deducimos de (i) que existe x1 ∈ R tal que f (x) < 0 si
x < x1 y de (ii), que existe x2 ∈ R tal que f (x) > 0 si x > x2 . Podemos por tanto elegir
a, b reales con a < b tales que f (a) < 0 y f (b) > 0.

Problema 331. Demuestren que la función f (x) = x3 + x2 − 3x + 2 toma el valor π en el


intervalo (1, 2).

Resolución.
La función f es continua en [1, 2]. Además f (1) = 1 + 1 − 3 + 2 = 1 y
f (2) = 8 + 4 − 6 + 2 = 8. Dado que 1 < π < 8, por el teorema Bolzano para funciones
continuas, existe c ∈ (1, 2) tal que f (c) = π.

Problema 332. (Opcional) Demuestren que si f es continua en [0, 1] y ahí satisface


0 ≤ f (x) ≤ 1, entonces f tiene un punto fijo; esto es, existe un número c en [0, 1] tal que
f (c) = c. Sugerencia: aplique el teorema de Bolzano a g(x) = x − f (x).

Problema 333. Una banda elástica estirada cubre el intervalo [0, 1]. Los extremos se
sueltan y la liga se contrae de modo que cubre el intervalo [a, b] con a ≥ 0 y b ≤ 1.
Demuestre que esto resulta en un punto de la liga (en realidad exactamente un punto)
que estará en donde estaba originalmente.

Problema 334. Un excursionista inicia a las 4 a. m., la escalada hacia la cima de una
montaña, a donde llega al mediodía. Al día siguiente regresa a por la misma ruta,
partiendo a las 5 a. m. A las 11 de la mañana llega al pie de la montaña. Demuestre que
en algún punto a lo largo de la ruta su reloj mostraba la misma hora en ambos días.

Problema 335. Demuestren que la ecuación x5 − 4x3 − 3x + 1 = 0 tiene al menos una


solución entre x = 2 y x = 3.

Si necesitás más problemas de la aplicación de los límites y los teoremas de valor medio,
podés consultar el sitio
https://www.cimat.mx/ciencia_para_jovenes/bachillerato/libros/%5BPurcell
,Varberg,Rigdon%5DCalculo/%5BPurcell,Varberg,Rigdon%5DCalculo_cap1.pdf
3
Problema 336. Consideren la función f (x) = x2x+4 . Determinen cuál de las siguientes
afirmaciones es cierta. Justifiquen su respuesta.

Tiene asíntotas verticales cuyas ecuaciones son x = 2 y x = −2.


Tiene una asíntota horizontal en y = 0.
No es continua en R.
Tiene una asíntota oblicua cuya ecuación es y = x.

Respuestas. Las tres primeras afirmaciones son falsas. La cuarta es verdadera.

105
Problema 337. Consideren la función
(
− x2 si x < 2
f (x) =
x − 3 si x ≥ 3

Determinen cuál de las siguientes afirmaciones es cierta. Justifiquen su respuesta.

La función es continua en R.
La función es continua en x = 2.
La función presenta una discontinuidad evitable en x = 2.
La función presenta una discontinuidad esencial en x = 2.

4.5. Estudio de las funciones a tramos

En esta sección mostramos, paso a paso, cómo realizar el análisis completo de una
función a tramos, indicando todos sus elementos, y una gráfica de la función.

Problema 338. Analicen y grafiquen la función


( 3
x +7x−8
x2 +3x−4
si x < 3
f (x) = 2
2x si x > 3

Resolución.

a) Determinamos el dominio de f .
Resolvemos la inecuación
x2 + 3x − 4 6= 0
Esta inecuación (que es válida para los x < 3, nos indica que x 6= −4; x 6= 1.
Según la definición de la función, x 6= 3.
Luego Df = R − {−4; 1; 3}.
b) Analizamos la continuidad en los tres valores de x que no pertenecen al dominio49 .
Notamos que:
20
lı́m f (x) = 18 lı́m f (x) = 7
x→3+ x→3−

Luego, la función tiene un salto finito (discontinuidad inevitable o esencial) en x = 3.

lı́m f (x) = 2 lı́m f (x) = 2


x→1+ x→1−

Pero x = 1 no pertenece al dominio de la función. Por lo tanto, la función tiene una


discontinuidad evitable (agujerito) en el punto (1; 2).

lı́m f (x) = ∞ lı́m f (x) = ∞


x→−4+ x→−4−

Entonces la función tiene una asíntota vertical (discontinuidad esencial o inevitable)


de ecuación x = −4.
c) Analizamos la existencia de asíntotas horizontales u oblicuas. Para ello sugerimos
leer el apunte asíntotas. Entonces:

49
Sugerimos releer la sección ¿Cómo se evalúa la continuidad de la función?.

106
f (x)
m = lı́m x =1 b = lı́m (f (x) − mx) = −3
x→−∞ x→−∞

Luego, la asíntota oblicua (a izquierda) de la función es la recta de ecuación


y = x − 3.
03 +7·0−8
d ) La ordenada al origen es el valor de f (0) = 02 +3·0−4
= 2. Entonces la función
interseca al eje Y en y = 2.
e) Para hallar las raíces, igualamos a cero ambos tramos de la función (y verificar que
la solución obtenida pertenezca al tramo en cuestión). Entonces:
x3 +7x−8 x2 +x+8
x2 +3x−4
= x+4 = 0 no tiene solución50 .
2x2 = 0 ⇒ x = 0. Sin embargo, x = 0 no pertenece a este tramo de la función,
por lo tanto la función no tiene raíces.
f ) Graficamos la función:

Problema 339. Determinen el dominio de las siguientes funciones a tramos. Esbocen una
gráfica de cada una de las funciones en una hoja cuadriculada o milimetrada. Usen una
escala 1:1 cm.
 (
1
−x si x < 0

c) h(x) = √x+2 si x < 4
a) f (x) = x2 si 0 6 x 6 1 x − 10 si x > 4

1 si x > 1
 (
1
x+1 si x ≤ 2
d ) f (x) = 2
x −9x+14
 si x > 2
−x si x < 0
 (
x2 −5x+6
3
b) g(x) = x si 0 < x < 1 x−1 si x ≥ −1
 e) h(x) = x2 −x−2
1 si x ≥ 2 si x < −1

x2 −4

Respuestas.
a) Df : R. Es continua y no tiene asíntotas.
b) Df : R − [1; 2). Es discontinua y no tiene asíntotas.
c) (−∞; −2) ∪ (−2; 4) ∪ [10; +∞). Es discontinua esencial en x = −2; x = 4 y x = 10.
Tiene una asíntota vertical en x = −2.
d ) Asíntotas verticales en x = −1 y x = 3. Asíntotas horizontales en y = 0 y en y = 1.
Es discontinua esencial en x = −1, x = 2 y en x = 3. Df : R − {−1; 2; 3}.
e) Asíntotas verticales x = −2; x = 1. Asíntotas horizontales: y = 0; y = 1. Es
discontinua esencial en x = −2, x = −1 y en x = 1. Df : R − {−2; 1}
50
La primera igualdad la obtenemos simplificando la expresión. Para ello dividimos numerador y deno-
minador por (x − 1) -por ejemplo, aplicando el algoritmo de Ruffini-. El hecho de que no tiene solución es
resultado de que la ecuación cuadrática resultante tiene discriminante negativo.

107
*****

Derivadas
5. Derivadas
Razón de cambio medio. Dada la función y = f (x) continua en el intervalo [a; b],
decimos que la razón de cambio51 de la función f entre x = a y x = b es igual a
∆y f (b) − f (a)
=
∆x b−a
En el siguiente gráfico observamos una interpretación geométrica de la derivada.

Problema 340. Hallen la razón de cambio promedio para:


a) f (x) = 2x2 , entre x = 1 y x = 4.
b) g(x) = 3x , entre x = 21 y x = 2.
c) h(x) = −2x3 , entre x = 0 y x = 3.

Respuestas. a) 10. b) 3. c) − 18.

5.1. Definición de la derivada

Derivada en un punto. Dada la función y = f (x) continua en el intervalo [a; b],


decimos que la derivada de la función f en x = x0 es igual a
0 f (x) − f (x0 )
f(x=x 0)
= lı́m
x→x0 x − x0
Función derivada. Si consideramos h = ∆x = x − x0 resulta la expresión equivalente
f (x + h) − f (x)
f 0 (x) = lı́m
h→0 h
Gráficamente

51
También lo denominamos cociente incremental.

109
Problema 341. Calculen (utilizando la definición) la derivada de la función y = 2x2 + 3x.

Resolución. Por definición de derivada, sabemos que

2(x + h)2 + 3(x + h) − (2x2 + 3x)


f 0 (x) = lı́m
h→0 h
Resolvemos el cuadrado del binomio y aplicamos la propiedad distributiva de la
multiplicación con respecto a la suma.

2x2 + 4xh + 2h2 + 3x + 3h − 2x2 − 3x


f 0 (x) = lı́m
h→0 h
Cancelamos los términos opuestos en el numerador y extraemos factor común h.
h(4x + 2h + 3)
f 0 (x) = lı́m
h→0 h
Cancelamos h y observamos que h → 0. Entonces

f 0 (x) = 4x + 3

Problema 342. Calculen la derivada de las siguientes funciones utilizando la definición.


Luego, determinen los valores de las derivadas para los valores de x que se especifican:

a) f (x) = x2 + 1 en x = 2. f ) f (x) = x3 + 3x en x = 1.

b) f (x) = x − 2x2 en x = 1. g) f (x) = x en x = 4.
x

c) f (x) = x−2 en x = 3. h) f (x) = x + 1 en x = 8.
8 x−1
d ) f (x) = en x = 2. i) f (x) = x+1 en x = 0.
x2
3 1
e) f (x) = x en x = 1. j ) f (x) = x + x en x = 1.

Respuestas.
1
a) 4. c) -2. e) 3. g) 4. i) 2.
1
b) -3. d ) -2. f ) 6. h) 6. j ) 0.

Problema 343. Calculen la derivada de la función y = sen(x) a partir de la definición de


la función derivada.

Resolución. Según la definición de la derivada, sabemos que


sen(x + h) − sen(x)
[sen(x)]0 = lı́m
h→0 h
Aplicamos la propiedad del seno de la suma de dos ángulos
sen(x)cos(h) + sen(h)cos(x) − sen(x)
[sen(x)]0 = lı́m
h→0 h
Asociamos convenientemente
sen(x)[cos(h) − 1] + sen(h)cos(x)
[sen(x)]0 = lı́m
h→0 h
Por la propiedad de la suma de límites
sen(x)[cos(h) − 1] sen(h)cos(x)
[sen(x)]0 = lı́m + lı́m
h→0 h h→0 h

110
Multiplicamos el numerador y el denominador del primer término por cos(h) + 1
sen(x)[cos(h) − 1] cos(h) + 1 sen(h)cos(x)
[sen(x)]0 = lı́m · + lı́m
h→0 h cos(h) + 1 h→0 h
Operamos en el primer término del segundo miembro
sen(x)[cos2 (h) − 1] sen(h)cos(x)
[sen(x)]0 = lı́m + lı́m
h→0 h[cos(h) + 1] h→0 h
Aplicamos la relación pitagórica; reemplazamos cos2 (h) − 1 por −sen2 (h)
sen(x)[−sen2 (h)] sen(h)cos(x)
[sen(x)]0 = lı́m + lı́m
h→0 h[cos(h) + 1] h→0 h
Aplicamos la propiedad asociativa
sen(h) −sen(x) sen(h)
[sen(x)]0 = lı́m · sen(h) · + lı́m · cos(x)
h→0 h cos(h) + 1 h→0 h
Observamos que el segundo factor del primer término tiende a 0 y los otros factores están
acotados, por lo tanto el primer término es igual a cero. Luego
sen(h)
[sen(x)]0 = lı́m · cos(x)
h→0 h
El primer factor del segundo miembro es igual a 1, entonces

[sen(x)]0 = cos(x)

Problema 344. Hallen la función derivada de la función y = ln(x).

Resolución. Según la definición de la derivada, sabemos que


ln(x + h) − ln(x)
f 0 (x) = lı́m
h→0 h
Aplicamos la propiedad del logaritmo de una potencia.
 
1 x+h
f 0 (x) = lı́m ln
h→0 h x
 1
0 x+h h
f (x) = lı́m ln
h→0 x
Aplicamos la propiedad distributiva de la división con respecto a la suma y simplificamos.
 1
0 h h
f (x) = lı́m ln 1 +
h→0 x
Multiplicamos y dividimos el exponente por x.
 x
0 h xh
f (x) = lı́m ln 1 +
h→0 x
Aplicamos la definición del número e.
1
f 0 (x) = lı́m ln e x
h→0

Equivalentemente
1
f 0 (x) =
x
Como queríamos demostrar.

111
5.2. Tabla de derivadas

Sugerencia: Agreguen todas las funciones (con sus derivadas) que consideren que les
pueden ser útiles, así como los ejemplos que les puedan servir para entender las
propiedades enunciadas.

No Función Derivada Ejemplo / comentario


1 k (k ∈ R) 0 (5)0 = 0 . La derivada de una constante da cero.
2 x 1
3 a · x (a ∈ R) a (8x)0 = 8
0
4 xn (n ∈ R) n · xn−1 x5 = 5x4
1
5 − x12
√x 1
6 x √
2 x
7 sen(x) cos(x)
8 cos(x) −sen(x)
9 tan(x) sec2 (x)
10 cotan(x) −cosec2 (x)
11 sec(x) sec(x) · tan(x)
12 cosec(x) −cosec(x) · cotan(x)
1
13 ln(x) x
1
14 logb x x·ln(b) b ∈ R+ − {1}
15 ex ex
16 ax ax · ln(a) a ∈ R+
k · f 0 (x) 7 0
3 · x = 3 · (x7 )0 = 3 · 7x6 = 21x6

17 k · f (x)
18 f (x) ± g(x) f 0 (x) ± g 0 (x) [sen(x) + 7x]0 = [sen(x)]0 + (7x)0 = cos(x) + 7
19 f ·g f 0 · g + f · g0 [2x5 · tan(x)]0 = [10x4 · tan(x)] + [2x5 · sec2 (x)]
h i0
f 0 ·g−f ·g 0
20 f
g (g 6= 0) g2
2x−5
cos(x) = 2·cos(x)−(2x−5)·[−sen(x)]
cos2 (x)
√ 0
21 f [g(x)] f 0 (g) · g 0 (x) 3x2 − 7x + 1 = 2√3x21−7x+1 · (6x − 7)

112
Problema 345. Calculen la derivada de las siguientes funciones aplicando las propiedades
de las derivadas. Anoten al margen en cada paso, el número de la propiedad que
aplicaron.

a) f (x) = −x2 + 3 ñ) y = tan(x)


b) f (x) = x2 +x+8 o) r(s) = x
3s2
− 5s
2x
c) f (t) = 5t3 − 3t5 p) f (x) = 3x2 − 5sen(x)
d ) w = 3z 7 − 7z 3 + 21z 2 q) f (x) = ln(x) · cosec(x)
3
e) y = 4x3 − x r ) f (x) = sen2 (x) + cos2 (x)
3 2
f) y = x3 + x2 + x4 s) f (x) = 3cos(x)
ln(x)
g) w = 3z −2 − z1 t) y = x4 − ex + 4x
h) s = −2t−1 + t42 √
sen(x) x
u) f (x) = cos(x)+1
i) y = 6x2 − 10x − 5x−2
v ) f (x) = (x + 2)(x3 − 5x + 1)
j ) y = 4 − 2x − x−3
1
x 5s w) y = x+3 + x3 · e x
k ) r(x) = 3s2
− 2x
12 4 1 x ) y = (x + 3)2
l ) r(α) = α − α3 + α4
2x−5
m) y = x · sen(x) y) f (x) = sen(x)
1

3

4
n) f (x) = cos(x) − ln(x) z ) f (x) = ( x7 )(x + x5 )

Respuestas.

a) f 0 (x) = −2x ñ) f 0 (x) = sec2 (x)

b) f 0 (x) = 2x + 1 o) r0 (s) = − 3s
2x
3 −
5
2x

c) f 0 (t) = 15t2 − 15t4 p) f 0 (x) = 6x − 5cos(x)

d) w0 (z) = 21z 6 − 21z 2 + 42z q)


cosec(x)
e) y 0 (x) = 4x2 − 1 f 0 (x) = x − ln(x) · cosec(x) · cotan(x)

f) y 0 (x) = x2 + x + 1
4 r) f 0 (x) = 0
3cos(x)
−3sen(x)ln(x)−
g) w0 (z) = −6z −3 + z −2 s) f 0 (x) = ln2 (x)
x

h) s0 (t) = 2t−2 − 8t−3 t) y 0 (x) = 4x3 − ex + 4x ln(4)

i) y 0 (x) = 12x − 10 + 10x−3 u) A cargo del alumno.

j) y 0 (x) = −2 + 3x−4 v) f 0 (x) = 4x3 + 6x2 − 10x − 9

k) r( x) = 1
3s2
+ 5s
2x2
l) w) y 0 = − (x+3)
1 3 2
2 + (x + 3x ) · e
x

r0 (α) = − α122 + 12
α4
− 4
α5
x) y 0 (x) = 2(x + 3)
2sen(x)−(2x−5)cos(x)
l) r0 (s) = − 3s22 + 5
2s2
y) f 0 (x) = sen2 (x)
10 73 43 31
m) y 0 = sen(x) + x · cos(x) z) f 0 (x) = 3 x + 12 x
12

sen(x)
n) f 0 (x) = cos2 (x)
− 1
x

Si necesitan más problemas resueltos para practicar, corregirse y autoevaluarse pueden


consultar los siguientes links:

113
http://fernandorevilla.es/blog/2014/03/20/algebra-de-derivadas/
http://fernandorevilla.es/blog/2014/03/22/
derivacion-de-funciones-exponenciales-y-logaritmicas/
http://fernandorevilla.es/blog/2014/03/21/
derivacion-de-funciones-trigonometricas-y-circulares-inversas/

Movimiento a lo largo de una recta

Supongamos que un objeto se mueve a lo largo de una recta, de manera que conocemos
su posición s en esa recta como una función del tiempo t : s = s(t).

El desplazamiento del objeto en el intervalo de tiempo que va de t a (t + 4t) es:


4s = s(t + 4t) − s(t), y la velocidad promedio del objeto en ese intervalo de tiempo es:

desplazamiento 4s s(t + 4t) − s(t)


vprom = = =
tiempo de recorrido 4t 4t

Observamos que la velocidad promedio (o velocidad media) del cuerpo es el cociente entre
la variación total de la posición (s(t + ∆t) − s(t)) y el tiempo en el cual se produce dicha
variación (∆t).
Para calcular la velocidad del cuerpo en un determinado instante t, tomamos el límite de
la velocidad promedio en el intervalo que va de t a (t + ∆t) cuando ∆t tiende a cero. Este
límite es la derivada de s(t) con respecto a t. . Es decir:

s(t + ∆t) − s(t)


v = lı́m = s0 (t)
∆t→0 ∆t
En otras palabras, la velocidad de un objeto es la derivada de su posición respecto del
tiempo.
Análogamente, la aceleración representa el cambio en la velocidad. Es decir:

v(t + ∆t) − v(t)


a = lı́m = v 0 (t) = s00 (t)
∆t→0 ∆t
O sea, la aceleración es la derivada de la velocidad, y por lo tanto, la derivada segunda de
la posición.

114
Problema 346. Sea s(t) = 12 at2 la función desplazamiento de un móvil. Si la aceleración a
m
es de 50 seg 2 y el móvil inició su movimiento con velocidad nula, calculen:

a) El desplazamiento total del móvil a los 10 segundos.


b) La velocidad promedio en ese lapso de tiempo.
c) La velocidad del móvil a los 5 segundos (velocidad instantánea).
m
d ) ¿Es cierto que la velocidad nunca fue superior a 300 s?
e) ¿Cuál fue la velocidad a los 8 segundos?
m
Respuestas. a) 2500m b) 250 seg c) 250 m
s

Caída libre

Se denomina caída libre al movimiento de un cuerpo bajo la acción única de un campo


gravitatorio. Ejemplo de este tipo de movimiento es la caída de un cuerpo al ser soltado
desde la terraza de un edificio. Suponiendo que el cuerpo se suelta, su velocidad inicial es
nula. Si a su vez se supone que su posición inicial también es cero, entonces su posición
en función del tiempo está representada por la ecuación52 :
1 m 2
s(t) = at2 = 4, 9 t
2 seg 2

Problema 347. Suponiendo que el cuerpo alcanza el suelo a los 7 segundos, estimen:

a) La altura desde donde cayó.


b) La velocidad final alcanzada por el cuerpo.
c) Su velocidad promedio a lo largo de toda la caída.
d ) Su velocidad a los 3 y 5 segundos respectivamente.

Respuestas. a) 240, 1m b) 68, 6 m


s c) 34, 3 m
s d) v(3) = 29, 4 m
s v(5) = 49 m
s

Problema 348. En la misión espacial Apolo 15, el astronauta David Scott dejó caer sobre
la superficie lunar un martillo y una pluma al mismo tiempo y desde una misma altura53 .
con la idea de demostrar que el razonamiento de Galileo Galilei (400 años antes) sobre la
caída libre de cuerpos era correcta. En ausencia de atmósfera, y sólo bajo la acción
gravitatoria lunar, el martillo y la pluma cayeron con la misma velocidad y al mismo
tiempo sobre el suelo selenita. Teniendo en cuenta que la gravedad lunar es una sexta
parte que la terrestre, y que Scott soltó ambos elementos desde una altura aproximada de
1,10m, calculen:

a) El tiempo que duró la caída.


b) La velocidad final de los objetos al llegar al suelo lunar.

Respuestas. a) t = 1, 16seg b) v = 1, 9 m
s

Problema 349. Calculen la aceleración de un móvil que se posiciona según

a) x(t) = 10 + 25t + 4t2 b) x(t) = 20 + 10t

¿Con qué movimientos se puede identificar cada una de las ecuaciones anteriores?
Respuestas. a) x00 (t) = 8 b) x00 (t) = 0
52 m
La aceleración de gravedad terrestre es 9, 8 seg 2.
53
Un lindo video sobre el experimento de Scott puede encontrarse en: http://www.youtube.com/watch
?v=KDp1tiUsZw8

115
Problema 350. Recordando que [sen(x)]0 = cos(x) y que [cos(x)]0 = −sen(x), calculen la
primera derivada de las siguientes funciones trigonométricas:

a) y = tan(x) b) y = cotan(x) c) y = sec(x) d ) y = csc(x)

Respuestas.

a) y 0 = sec2 (x) c) y 0 = sec(x)tan(x)


b) y 0 = −cosec2 (x) d ) y 0 = −cosec(x)cotan(x)

Problema 351. Calculen la primera derivada de las siguientes funciones aplicando las
propiedades correspondientes:
cotan(x)
a) y = −10x + 3cos(x) g) y = 1+cotan(x)
3
b) y = + 5sen(x) cos(x)
x
√ h) y = 1+sen(x)
c) y = csc(x) − 4 x + 7 4 1
1 i) y = cos(x) + tan(x)
d ) y = x2 cotan(x) − x2
cos(x) x
e) y = [sec(x) + tan(x)][sec(x) − tan(x)] j) y = x + cos(x)

f ) y = [sen(x) + cos(x)]sec(x) k ) y = x2 sen(x) + 2xcos(x)

Respuestas.
2
a) y 0 (x) = −10 − 3sen(x) cosec (x)
g) y 0 (x) = − [1+cotan(x)]2
b) y 0 (x) = − x32 + 5cos(x) h) y 0 (x) = − 1+sen(x)
1
c) y 0 (x) = −cosec(x)cotan(x) − √2
x i) y 0 (x) = 4tan(x)sec(x) − cosec2 (x)
d) y 0 (x) = 2x · cotan(x) − x2 cosec2 (x) + 2
x3 j) y 0 (x) = − xsen(x)+cos(x) + cos(x)+xsen(x)
x2 cos2 (x)
e) y 0 (x) = 0 0 2
k) y (x) = x cos(x) + 2cos(x)
f) y 0 (x) = sec2 (x)

Problema 352. Recordando que (ex )0 = ex y que [ln(x)]0 = x1 , calculen la derivada de las
siguientes funciones:

a) y = 2x e) y = 7x − 9ln(x) + 3
lnx+9
b) y = log(x) f) y = ex
c) y = 3 · ln(x) g) y = logb x con b > 0; b 6= 1
2 −x
d) y = 5e h) y = bx con b > 0; b 6= 1

Respuestas

a) y 0 = 2x ln(2) d ) y 0 = − 25 e−x g) y 0 = 1
xlnb
b) y 0 = 1 e) y 0 = 7 − x9 h) y 0 = bx ln(b).
xln(10)
ex
−ex ln(x+9)
c) y 0 = 3
x f ) y0 = x
e2x

116
Regla de la cadena

Problema 353. Calculen la derivada de las siguientes funciones:


2
e4x +3x
p
3 g(x) = 3
f (x) = x2 − 2x · ln[sen(x)] x − 2x4 + 10

Resolución. Queremos calcular


p 0
f 0 (x) =
3
x2 − 2x · ln[sen(x)]

Aplicamos la derivada del producto entre dos funciones:


p 0 p
f 0 (x) = x2 − 2x · ln[sen(x)] + x2 − 2x · (ln[sen(x)])0
3 3

Aplicamos la regla de la cadena.


   
0 1 2 − 32
p
3 2
cos(x)
f (x) = (x − 2x) (2x − 2) · ln[sen(x)] + x − 2x ·
3 sen(x)
Cálculos auxiliares√
Para derivar z = 3 x2 − 2x escribimos a la raíz cúbica como una potencia. De esta
manera podemos derivarla aplicando la propiedad de la derivada de xn . Entonces
p3 1
z = x2 − 2x = x2 − 2x 3
1
Para aplicar la regla de la cadena, disfrazamos a la función z = u 3 donde:

u = x2 − 2x

u0 = 2x − 2
Por lo tanto
 1 0 1 1 1 2 1 2
z 0 = u 3 · u0 = u 3 −1 u0 = u− 3 u0 = (x2 − 2x)− 3 (2x − 2)
3 3 3
Entonces p 0 1 2
x2 − 2x = (x2 − 2x)− 3 (2x − 2)
3

3
Para derivar ln(sen(x)) llamamos z = ln(sen(x)) y disfrazamos a la función z = ln(u)
con
u = sen(x)
u0 = cos(x)
Entonces (aplicamos la regla de la cadena)
1 0 1 cos(x)
z 0 = (ln(u))0 · u0 = ·u = · cos(x) =
u sen(x) sen(x)
Entonces
cos(x)
(ln(sen(x))0 =
sen(x)

Calculamos la derivada de g (aplicamos la propiedad de la derivada de un cociente).


!  4x2 +3x 0 2
e 4x 2 +3x e · (x3 − 2x4 + 10) − e4x +3x · (x3 − 2x4 + 10)0
g 0 (x) = =
x3 − 2x4 + 10 (x3 − 2x4 + 10)2

117
Aplicamos la regla de la cadena en el primer término:
 2  2
e4x +3x (8x + 3) · (x3 − 2x4 + 10) − e4x +3x · (3x2 − 8x3 )
g 0 (x) =
(x3 − 2x4 + 10)2

Cálculos auxiliares
2
Para derivar z = e4x +3x (aplicamos la regla de la cadena) disfrazamos a la función
z = eu donde
u = 4x2 + 3x
u0 = 8x + 3
Entonces z 0 = (eu )0 · u0 = eu · u0 = e4x
2 +3x
(8x + 3).

Problema 354. Calculen la derivada de las siguientes funciones.

a) y = (2x + 1)5 m) y = log(4 − 3x)9


9 √
b) y = (4 − 3x) n) y = 3 3−x
−7
c) y = 1 − x7 ñ) y = sen3 (x)
−10
d ) y = x2 − 1 1 ln(x)
 2 4 o) y = x + 2ln(2) − x
e) y = x8 + x − x1 p) y = exln(x)
1 5
f ) y = x5 + 5x q) y = x2 · 2x


g) y = 3 − x r ) y = ln(e−x + xe−x )

h) y = 2x − x2 e
s) y = ln 1+e
x
x
i) y = 1
2x (3x − 2)7 √
4 t) y = x x[3ln(x) − 2]
j ) y = (5 − 2x)−3 + 1 2
x + 1 √
8 u) y = ln(x + x2 + 1)
−2
k ) y = (4x + 3)4 (x + 1) √ 
(1+x2 )−1
l) y = 1 v ) y = tan √ 2
(2x+1)5 (1+x )+1

Respuestas.

a) y 0 (x) = 10(2x + 1)4 l) y 0 (x) = − (2x+1)


10
6
b) y 0 (x) = −27(4 − 3x)8 m) y 0 (x) = − ln(10)(4−3x)
27
−8
c) y 0 (x) = 1 − 17 x √
3−x
−11 n) y 0 (x) = − ln(3)·3

d) y 0 (x) = −5 x2 − 1 2 3−x
 2 3 ñ) y 0 (3sen2 (x)cos(x)
e) y 0 (x) = x8 + x − x1 (x + 4 + 4x−2 ) o) y 0 (x) = ln(x)−2
4 1 x2
f) y 0 (x) = 5 x5 + x15 1 p) y 0 (x) = exln(x) · (1 + ln(x))

5 − 5x2
g) y 0 (x) = − 2√3−x
1 q) y 0 (x) = x · 2x · (2 + xln(2))
h) y 0 (x) = √ 1−x r) y 0 (x) = − 1+x
x
2x−x2
0 (3x−2)7 21 s) y 0 (x) = 1+e
1
x
i) y = − 2x2 + 2x (3x − 2)6 0 9 √
3 t) y (x) = 2 xln(x)
j) y 0 (x) = 6(5 − 2x)−4 − x12 x2 + 1 u) y 0 (x) = √x12 +1
4
k) y 0 (x) = 16(4x + 3)3 (x + 1)−2 − 2(4x+3) √
2
√1+x −1

(x+1)3 v) y 0 (x) = sec2 1+x2 +1
· √
2x
√ 2
1+x2 ( 1+x2 +1)

Si quieren más problemas resueltos, pueden consultar el link


http://fernandorevilla.es/blog/2014/03/25/
derivacion-de-funciones-compuestas-regla-de-la-cadena/.

118
Derivabilidad

Decimos que una función es derivable en un punto (x0 ; y0 ) si existe f 0 (x0 ). Es decir, si
existe el límite
f (x) − f (x0 )
L = lı́m
x→x0 x − x0
Hasta ahora, en el desarrollo de la guía, hemos tratado con funciones derivables. Pero
existen funciones que no son derivables54 . Por ejemplo:

Problema 355. Prueben que la función


(
−x − 1 si x < 0
f (x) =
x2 − 1 si x ≥ 0

no es derivable en x = 0.

Resolución. Según la definición, f es derivable si existe


f (x) − f (x0 )
L = lı́m
x→x0 x − x0
Debemos calcular los límites laterales para determinar la existencia del límite dado.
Entonces:
f (x) − f (0) (−x − 1) − (−1)
L− = lı́m = lı́m = −1
x→0− x−0 x→0− x−0
f (x) − f (0) (x2 − 1) − (−1)
L+ = lı́m = lı́m = lı́m 2x = 0
x→0+ x−0 x→0+ x−0 x→0+

Entonces, dado que L− 6= L+ , afirmamos que no existe la derivada y por ende, la función
no es derivable en x = 0.
Comentarios.

Si calculamos las derivadas de cada uno de los tramos de la función, notamos que a
la izquierda del cero, la derivada vale −1 y a la derecha del cero, la derivada vale 2x;
entonces, si evaluamos cada una de los tramos con x → 0, llegamos a la misma
conclusión.
Si calculamos la derivada de f en cualquier otro punto de su dominio, notaremos
que existe. Es decir que f es derivable en todo el dominio, excepto en x = 0.

Problema 356. Prueben que no existe la derivada de la función y = |x| en x = 0.

Teorema. Si una función es derivable en un punto x0 de su dominio, entonces es


continua en dicho punto.

Demostración. Si f es derivable, entonces existe el límite


f (x) − f (x0 )
L = lı́m
x→x0 x − x0
Multiplicamos ambos miembros de la igualdad por (x − x0 ). Entonces

lı́m L · (x − x0 ) = lı́m f (x) − f (x0 )


x→x0 x→x0

El primer miembro de la igualdad es igual a cero. Sumamos f (x0 ) en ambos miembros:

f (x0 ) = lı́m f (x)


x→x0
54
En algún punto, en varios puntos, e inclusive, funciones que no son derivables en ningún punto de su
dominio.

119
Que implica que f es continua en x = x0 .

Si querés resolver problemas relacionados con la derivabilidad de las funciones,


podés consultar http://www.e-matematicas.es/Ficheros/2BACH-CT/Problemas%
20resueltos%20de%20continuidad%20y%20derivabilidad.pdf.

5.3. Aplicaciones de las derivadas

Problema 357. Cierta función f verifica las siguientes dos condiciones:

(i) f 0 (3) = 5 (ii) f 0 (2) 6= 5

a) Inventen una posible fórmula para f .


b) Si f representa la posición de un vehículo en función del tiempo, ¿qué significan las
condiciones (i) y (ii)?

Problema 358. Un automóvil se desplaza durante 12 segundos con un movimiento


descripto por el siguiente gráfico de posición en función del tiempo:

a) Hallen la fórmula de la posición del automóvil en función del tiempo x(t).


b) ¿Qué velocidad tiene el automovil a los 8 segundos?
c) Grafiquen la función velocidad en función del tiempo v(t).
d ) Describan con un breve texto un posible comportamiento del automóvil.
e) ¿Cuál es su aceleración a los 2 segundos?
f ) Grafiquen la función aceleración en función del tiempo a(t).

Respuestas. a) x(t) = −2t(t − 10) b) − 12 e) − 4

La recta tangente

En esta sección mostraremos los pasos a seguir para hallar la ecuación de la recta
tangente a una función en un punto dado. Es decir, presentaremos una solución al
problema de encontrar una recta L de ecuación y = mx + b que sea tangente a una
función y = f (x) en un punto de abscisa x = a. En principio, consideramos dos
cuestiones:

120
Si la función f es tangente a la recta L entonces la pendiente de la recta L coincide
con la pendiente de f en el punto P de abscisa x = a.
Si la recta L es tangente a la curva f en x = a entonces se intersecan en ese punto.

Estas dos condiciones las podemos expresar en forma algebraica:

m = f 0 (a).
P ≡ (a; f (a)) ∈ L.

De esta manera los pasos a seguir para hallar la ecuación de la recta tangente son los
siguientes.

a) Hallar la pendiente de la recta: m = f 0 (a).


b) Hallar la ordenada del punto de tangencia: y = f (a).
c) Reemplazar en la ecuación general de la recta y = mx + b los datos obtenidos en (a)
y (b), y despejar b.

Problema 359. Hallen la ecuación de la recta tangente a la función y = x2 + 3x − 5 en el


punto de abscisa x = 4.

Resolución.

a) Hallamos la pendiente de la recta:

m = f 0 (a)
0
m = x2 + 3x − 5 x=4
m = (2x + 3)x=4
m=2·4+3
m = 11

b) Hallamos la ordenada del punto de tangencia: y = f (x):

y = f (4)

y = 42 + 3 · 4 − 5
y = 23

c) Reemplazamos en la ecuación general de la recta y = mx + b los datos obtenidos en


(a) y (b):
y = mx + b
23 = 11 · 4 + b
Y despejamos b:
23 − 44 = b
−21 = b

Entonces la ecuación de la recta L es y = 11x − 21.


Es muy importante verificar el resultado obtenido. Para esto podemos graficar la función
f y la recta L en un mismo sistema de ejes cartesianos para comprobar que la recta es
tangente a la curva en el punto dado.

121
Notamos a partir del gráfico hecho en GeoGebra la veracidad del resultado obtenido.

Eventualmente pueden surgir problemas en los que haya que encontrar el punto de
tangencia a partir de la ecuación de la recta tangente (o de una recta paralela a la
tangente). Para ello, simplemente invertimos los pasos.

Problema 360. Determinen el o los puntos de la gráfica de f (x) = x3 − x2 + 3 en los


cuales la recta tangente es paralela a la recta L : y = 8x − 5.

Resolución. Planteamos:

a) m = f 0 (a). En este caso conocemos el valor de la pendiente, que por hipótesis es 8.


Entonces:
(x3 − x2 + 3)0 = 8
3x2 − 2x = 8
4
x1 = 2 ; x2 = −
3
b) Para x = 2 tenemos que:
f (2) = 23 − 22 + 3
f (2) = 7
Entonces
y = mx + b
7=8·2+b
−9 = b

Y la primera recta que buscamos es

L1 : y = 8x − 9

La segunda recta la obtenemos de forma análoga, considerando x = − 43 . Resulta

257
L2 : y = 8x +
27

122
Problema 361. Hallen la ecuación de la recta tangente a la función f (x) en el punto de
abscisa dada, si existe. En tal caso, grafiquen la función y la recta tangente en el punto
dado55 :

a) f (x) = x2 − 5 en el punto p ≡ (3; f (3))


b) f (x) = ln(x) en el punto p ≡ (1; f (1))

c) f (x) = x en el punto p ≡ (1; f (1))
2x−5
d ) f (x) = 4−x2
en el punto p ≡ (0; f (0))
e) f (x) = e x2 −1 en el punto p ≡ (1; f (1))
f ) f (x) = x · tan(x) en el punto p ≡ (π; f (π))
g) f (x) = ln(x3 + 2x + 1) en el punto p ≡ (0; f (0))

h) f (x) = 3 x en el punto p ≡ (0; f (0))

Respuestas.

a) y = 6x − 14 d ) y = 12 x − 54 g) y = 2x
b) y = x − 1 e) y = 2x − 1
c) y = 12 x + 21 f ) y = πx − π 2 h) x = 0.

Problema 362. Hallen (si existen) los puntos de la gráfica de f (x) en los cuales la recta
tangente es paralela a la recta L.

a) f (x) = x2 + 2x − 4 L : y = 3x − 1
b) f (x) = x3 − 3x2 + 5x − 1 L : y = 14x + 9
c) f (x) = xex L:y=3
d ) f (x) = 2x − 9 L : y = 3x + 1
x
e) f (x) = x+1 L:y =x−2

f ) f (x) = x · ln(x) − x + 5 L : y = 12 x + 7

Respuestas.
1 11

a) p ≡ 2; − 4 d ) No existe.
b) p1 ≡ (3; 14); p2 ≡ (−1; −10) e) p1 ≡ (0; 0); p2 ≡ (−2; 2)
√ √ 
c) p ≡ −1; − 1e f) p ≡ e; − 2e + 5


Problema 363. Calculen los valores de a y b tales que la recta de ecuación y = 54 x − 3 es


x2
tangente al gráfico de la función f (x) = ax+b en el punto de abscisa x = 1.
76 48
Respuestas. a = − 49 ; b = 49 .

Problema 364. Sea f (x) = x3 + 4ax2 + bx + 4, determinen los valores de a y b tales que la
recta de ecuación y = 2x + 5 sea tangente al gráfico de f en (1; f (1)).
Respuestas. a = − 34 ; b = 5.

Problema 365. Hallen b ∈ R tal que la función f (x) = (x2 − b) · ln(x2 − b) tenga un
extremo relativo en x = 3, y determinen si es un máximo o un mínimo.
Respuestas. b = − 1e + 9.

55
Eventualmente pueden utilizar algún software como el GeoGebra para la verificación gráfica.

123
Problema 366. La recta tangente al gráfico de una cierta función f en x = 0 tiene
ecuación y = 5x − 2.

a) ¿Cuánto vale f 0 (0)?


b) Si en x = 3 la recta tangente tiene ecuación y = −2x + 3, ¿cuánto vale f (3)?
c) ¿Cómo puede ser la fórmula de la función f ? ¿Es única la respuesta?

Respuestas. a) f 0 (0) = 5 b) f (3) = −3. c) Hay infinitas funciones posibles.

Problema 367. Si f (x) = x2 − 3x + 1, ¿es cierto que la ecuación de la recta tangente a f


en (2; −1) es y = 2x − 3?
Respuestas. No es cierto. La recta tangente en el punto dado es y = x − 3.

Problema 368. ¿Existe alguna función cuadrática tal que la ecuación de recta tangente a
su gráfica en algún punto sea igual a la derivada de dicha función?
Respuestas. Sí. Por ejemplo, f (x) = x2 + 1 en el punto (1; 2).

Problema 369. La recta de ecuación y = −2x − 1 es la recta tangente de alguna de las


siguientes funciones:

f (x) = x2 − x + 6 g(x) = −x2 − x + 1 h(x) = x3 − 5x − 3

a) Decidan a cuál de las funciones es tangente la recta dada en el punto (−1; 1).
Argumenten su decisión.
b) Hallen la ecuación de la recta tangente a la función h en el punto (2; −5).
c) Sin hacer las cuentas, expliquen porqué h(2, 01) es aproximadamente igual al valor
de y en la recta tangente para x = 2, 01.

Respuestas. a) y = h(x) b) y = 7x − 19

Problema 370. La recta de ecuación y = 2x + 3 es la recta tangente a una función f (x)


en x = 5.

a) ¿Cuánto vale f (5)?


b) ¿Cuánto vale f 0 (5)?
c) Escriban una posible fórmula para f (x).

Respuestas. a) f (5) = 13 b) f 0 (5) = 2

Problema 371. De una función f sabemos que:

Es una función cuadrática.


La ordenada al origen de f es y = −1.
La recta tangente a f en el punto de abscisa x = 3 es la recta de ecuación y = 2x + 5

a) Hallen las coordenadas en las que la recta dada es tangente a la función f .


b) Hallen la ecuación de la función f .

Respuestas. a) (3; 11) b) f (x) = − 23 x2 + 6x − 1.

Problema 372. Consideren la función f (x) = x3 − 3x2 + 1. Determinen:

a) La inclinación en el punto de abscisa x0 = −0, 5


b) La inclinación en los puntos de intersección con la recta x = 1
c) ¿En qué puntos la pendiente es igual a la pendiente de la recta 15x − 4y − 15 = 0?

124
d ) Los puntos en los cuales la tangente es paralela a la recta identidad.
e) Los puntos en los cuales la tangente es horizontal.

Respuestas.

a) 75, 07o .
b) 108, 43o .
5
c) Es el punto de abscisa x1 = 2 y el de abscisa x2 = − 21 .

2 3
d ) Son los puntos de abscisas x = 1 ± 3 .
e) Es el punto de abscisa x1 = 0 y el de abscisa x2 = 2

Problema 373. Consideramos el circuito esquematizado a continuación.

Cargamos el capacitor con una fuente de 12 Volts. Luego conmutamos la llave y lo


descargamos a través de una resistencia. El capacitor se descarga según la función
t
Vc (t) = V0 · e− R·C

Siendo:

Vc la tensión en el capacitor (medida en Voltios).


V0 la tensión del capacitor (en el instante en el que conmutamos la llave).
t el tiempo.

a) Hallen la tensión en el capacitor, con V0 = 12 y t = 3 milisegundos, para los


siguientes valores:
R (Resistencia en Ω C (Capacidad en F ) τ =R·C Vc (Voltios)
10 0, 001 F
1000 0, 001 F
106 0, 001 F
10 106 F
1000 106 F
106 106 F
b) Representen en un mismo sistema de coordenadas cada una de las funciones que se
obtienen de reemplazar los parámetros dados en el ítem anterior (pueden utilizar un
software, por ejemplo, el GeoGebra).
c) Analice la rapidez del decrecimiento de cada una de las funciones dadas en función
de los parámetros.

125
Problema 374. Calculen el tiempo para el cual el capacitor se descargó más de un 99 %
(Sugerencia: sigan los siguientes pasos. . . ).

a) Hallen la ecuación de la recta tangente a la función y = Vc (t).


b) Hallen la intersección entre la recta tangente y el eje de abscisas.
c) Verifquen que dicho valor es τ = R · C.
d ) Calculen t = 5τ .
e) Calculen Vc (5τ ).
f ) Verifiquen que el valor hallado en el ítem anterior es menor al 1 % del valor V0 .
g) Grafiquen el el GeoGebra la función y = Vc (t) y la recta tangente obtenida en el
primer ítem56 .

Una aplicación de la recta tangente

Una aplicación de la recta tangente a una curva en un punto es la de aproximar el valor


de la función en un punto cercano al punto de tangencia. A modo de ejemplo proponemos
el siguiente problema:

Problema 375. Halle el valor aproximado de 67.

Resolución. En primer lugar notamos que la curva de ecuación f (x) = x y su recta
tangente son similares en puntos cercanos al punto de tangencia57 .


Como queremos calcular 67 proponemos√hallar la ecuación de la recta tangente a f en
x = 64 (dado que es conocido el valor de 64). Si seguimos los pasos sugeridos
1
anteriormente, obtenemos que dicha recta tangente tiene ecuación y = 16 x + 4. Entonces
el valor de f (67) es un número cercano a la ordenada de la recta cuando x = 67. Por esto
podemos afirmar que

∼ 1 · 67 + 4
67 =
16

67 ∼
= 8, 1875
Notamos que el resultado es bastante preciso, ya que según la calculadora

67 ∼
= 8, 1853

56
De ser necesario, pueden consultar la guía de GeoGebra en https://app.geogebra.org/help/
geogebraquickstart_es.pdf.
57
Esto sucede con cualquier función; en algunos casos la similitud es mucho mayor, en otros la curva y
la recta se diferencian más rápidamente.

126
Problema 376. Consideren la función f (x) = x2 .

a) Calculen el cociente incremental en el intervalo (3; 3, 02).


b) Demuestren, utilizando la definición de la derivada, que f 0 (x = 3) = 6.
c) Hallen la ecuación de la recta tangente a f en x = 3.
d ) A partir de lo realizado en el ítem anterior, hallen en forma aproximada 3, 022 .
e) Comparen el resultado hallado en d) con el resultado que arroja la calculadora.
¿Cómo calificarían al error cometido?

Respuestas. a) 6, 02 c) y = 6x − 9 d) 9, 12

El polinomio de Taylor
1
Problema 377. Grafiquen en algún software la función f (x) = x+1 y las funciones
polinómicas siguientes:

y1 = 1 − x.
y2 = 1 − x + x 2 .
y3 = 1 − x + x 2 − x 3 .

Problema 378. Comparen las gráficas de las funciones polinómicas con la gráfica de la
función f prestando especial atención en las cercanías del punto (0; 1).

Problema 379. Repitan el proceso con las siguientes funciones:


1
y1 = 2 − 14 (x − 1).
1
y2 = 2 − 14 (x − 1) + 18 (x − 1)2 .
1
y3 = 2 − 14 (x − 1) + 18 (x − 1)2 − 1
16 (x − 1)3 .

Presten especial atención al gráfico en las cercanías del punto 1; 12




Problema 380. Elijan un valor a del dominio de f . Grafiquen el polinomio


3
X f (j) (x = a) · (x − a)j
f (x) =
j!
j=0

Problema 381. Comparen la gráfica obtenida con la gráfica de la función f y extraigan


conclusiones. Consideren el número b = a + 0,2. Utilicen los resultados obtenidos para
hallar en forma aproximada el valor de f (b).

Problema 382. Leopoldo necesitaba hallar rápidamente el valor de e0,3 (aunque sea, en
forma aproximada). Para ello, contaba con los siguientes datos

Leopoldo sabe que e0 = 1


Leopoldo sabe sumar, restar, multiplicar y dividir fracciones.
Leopoldo tiene algunos conocimientos acerca de las derivadas y sus aplicaciones.

a) ¿Qué puede hacer Leopoldo para lograr su objetivo?


b) Muestren que con una aproximación lineal el error cometido es menor a 0, 01.
c) ¿Qué polinomio permite hallar e0,3 con un error menor a 0, 005 Ayuda: El número
favorito de Taylor es el 3.

127
¿Qué hay detrás del botón sin de tu calculadora?

En general, las calculadoras básicas (e inclusive las científicas) sólo saben sumar, restar,
multiplicar y dividir. Pero entonces. . . ¿Cómo hacen para calcular el seno de cualquier
número que ingresemos?
Detrás de cada una de las teclas grises que utilizamos habitualmente para calcular senos,
cosenos, logaritmos, potencias o raíces, hay guardadas ciertas funciones polinómicas que
aproximan los valores de dichas funciones en entornos cercanos a cualquier número. Por
ejemplo, si presionamos ln(1, 2) nuestra calculadora guarda, debajo de la tecla de la
2 3 4 5
función ln(x), el polinomio Pn (x) = (x − 1) − (x−1)
2 + (x−1)
3 − (x−1)
4 + (x−1)
5 − ···.

Problema 383. Consideren la función f (x) = ln(x).

a) Calculen f (k) en x = 1 para k ∈ I6 .


P6 f (k) (x=1)·(x−1)k
b) Muestren que el polinomio P6 del problema anterior es igual a k=0 k! .
c) Verifiquen usando el GeoGebra que el polinomio p aproxima a la función f en un
entorno cercano a x = 1.
d ) Aproximen el ln(1,2) utilizando el polinomio hallado. Respuesta. 0, 18232.
e) ¿Resulta útil el polinomio hallado para aproximar el resultado de ln(15, 32)? ¿Por
qué?

Problema 384. Hallen el polinomio de Taylor en un entorno de x = a, de orden n, para


las siguientes funciones.

a) f (x) = sen(x) con n = 8; a = 0.


1
b) f (x) = x−1 con n = 4; a = 2.
c) f (x) = cos(x) con n = 6557318; a = 0.

Respuestas.
x3 x5 x7
a) P (x) = x − 6 + 120 − 5040 .
b) P (x) = 1 − (x − 2) + (x − 2)2 − (x − 2)3 + (x − 2)4 .
(−1)k 2k
c) P (x) = 3278659
P
k=0 (2k)! x

Problema 385. Consideramos la función f (x) = sen(x), de la cual sabemos –entre otras
cosas- que es una función impar58 . Miren atentamente el polinomio hallado en el ítem (a)
del problema anterior. ¿Qué pueden notar?

Problema 386. Consideramos la función f (x) = cos(x), de la cual sabemos –entre otras
cosas- que es una función par59 . Miren atentamente el polinomio hallado en el ítem (c)
del problema anterior. ¿Qué pueden notar?

Problema 387. ¿Pueden extraer alguna conclusión acerca del motivo por el cual decimos
que una función es par o impar ?

Problema 388. Expliquen en menos de 153 palabras, en forma general, que la noción de
función par equivale a decir que f es par si su polinomio de Taylor asociado tiene
únicamente términos de grado par.

Problema 389. Expliquen en menos de 37 palabras, en forma general, que la noción de


función impar equivale a decir que f es impar si su polinomio de Taylor asociado tiene
únicamente términos de grado impar.
58
Sugerencia: revisen la noción de paridad.
59
Sugerencia: revisen la noción de paridad.

128
Problema 390. Relacionen las nociones discutidas anteriormente con las gráficas de las
funciones y sus simetrías.
Los alumnos que quieran algo de material extra, pueden consultar en algunos de los
siguientes links:

http://fernandorevilla.es/blog/2014/03/11/formula-de-taylor/
http://fernandorevilla.es/blog/2014/10/01/
desarrollo-en-serie-de-maclaurin-de-las-funciones-seno-y-coseno/
http://fernandorevilla.es/blog/2014/10/01/
desarrollo-en-serie-de-maclaurin-de-la-funcion-arco-tangente/
http://fernandorevilla.es/blog/2014/10/02/
desarrollo-en-serie-de-maclaurin-de-log1x/

Los teoremas del valor medio

Sugerimos la lectura de los siguientes apuntes:

Teorema de Rolle:
http://fernandorevilla.es/blog/2014/02/04/teorema-de-rolle/.
Teorema del valor medio (recíproco): http://fernandorevilla.es/blog/2015/12/
15/reciproco-del-teorema-del-valor-medio/.
Teorema de Cauchy: http://fernandorevilla.es/blog/2014/02/05/
teorema-del-valor-medio-de-cauchy/.

La regla de L’Hôpital

Les sugerimos la lectura del siguiente apunte:


http://fernandorevilla.es/blog/2014/04/18/regla-de-lhopital-para-00/.

Problema 391. Calculen los siguientes límites aplicando la regla de L’Hôpital.


sen(x) x3 −343 1 x

a) L = lı́m e) L = lı́m 2 i) L = lı́m 1 +
x→0 x x→7 x −8x+7 x→∞ x
3tan(5x) x3 −2x+1 1
b) L = lı́m 4x f ) L = lı́m 3x2 −2x3 +9 j ) L = lı́m (1 + x) x
x→0 x→∞ x→0
cos( πx ) x
c) L = lı́m x2 −12
g) L = lı́m e x+ln(x)−x
2 −2x+5 k ) L = lı́m2x−5
x→1 x→+∞ x→∞ 3x+1
1−cos(x)
d ) L = lı́m x2
h) L = lı́m x l ) L = lı́m xx
x→0 x→+∞ ln(x) x→0+

Respuestas.
a) L = 1 b) L = 15
4 c) L = − π4 d) L = 12 e) L = 49
2 f) L =
1
−2 g) L = +∞ h) L = +∞ i) L = e j) L = e k) L = 23 l) L = 1
Si necesitan más problemas de práctica de la regla de L’Hôpital, pueden consultar los
siguientes links:

http://fernandorevilla.es/blog/2015/10/25/
limite-llim_xrightarrow-0fracx-sin-xx3-por-tres-metodos/
http://fernandorevilla.es/blog/2014/04/20/
regla-de-lhopital-problemas-diversos/

129
Problema 392. Consideramos la función f (x) = xx .

a) Calculen L = lı́m f (x) (Sugerencia: utilice la regla de L’Hôpital).


x→0+
b) ¿Cuánto vale f (1)?
c) ¿Está definida la función en x = 0? Justifique su respuesta.
d ) Busquen al menos dos fuentes (libros, artículos On-Line, etc.) que validen (o
refuten) su respuesta a la pregunta inmediata anterior. Sugerencia: revisen los links
de la página web de Matemática I.
e) Determinen la validez de la siguiente afirmación:
Existen dos valores, a y b tales que f (a) = f (b).
f ) Justifiquen la existencia de un extremo local de la función f en el intervalo (0; 1) a
partir de lo argumentado anteriormente.
g) Determinen, con una cifra decimal exacta, el valor de c tal que f 0 (x) = 0. Expliquen
por qué puede responder esto sin calcular la derivada de f (x).
h) Calculen la derivada de f . Pueden utilizar algún software adecuado.
i) Hallen las coordenadas del mínimo absoluto de la función. Pueden utilizar el mismo
software que utilizó en el punto anterior.
j ) Determinen el conjunto imagen de la función f .

Respuestas. a) L = 1 b) f (1)
 = 1 1  c) Sí, está definida.
h 1 i
h) f 0 (x) = xx (ln(x) + 1) i) e−1 ; e− e j) Im = e− e ; +∞

5.4. Máximos y mínimos

Problema 393. Dada la función f (x) = x3 − 2x2 − 5x + 6 definida en el intervalo [−2; 3]:

a) Calculen los máximos y los mínimos. Clasifíquenlos en absolutos o relativos.


b) Hallen los intervalos de crecimiento y de decrecimiento.
c) Hallen los puntos de inflexión y los intervalos de concavidad y convexidad.
d ) Tracen la gráfica de la función.

Resolución. Calculamos los puntos críticos. Para ello resolvemos la ecuación

f 0 (x) = 0

Es decir
(x3 − 2x2 − 5x + 6)0 = 0
3x2 − 4x − 5 = 0
Aplicamos la fórmula de Bhaskara y obtenemos
p √
−(−4) ± (−4)2 − 4 · 3 · (−5) 4 ± 76
x= =
2·3 6
n √ √ o
Por lo tanto los puntos críticos son 4−6 76 ; 4+6 76 Determinamos si son máximos o
mínimos relativos (aplicamos el criterio de la segunda derivada).

Punto

crítico f 00 (x) = 6x − 4 f (x) Conclusión
4− 76 ∼ ∼
6√ = −0, 78 = 6 · (−0, 78) − 4 < 0 8, 2 Es un máximo relativo y absoluto.
4+ 76 ∼ ∼
6 = 2, 12 = 6 · 0, 78 − 4 > 0 −4, 06 Es un mínimo relativo y absoluto.
−2 0
3 0

130
Para hallar los intervalos de crecimiento y decrecimiento ubicamos en el eje X los puntos
críticos.

Y en función de las conclusiones anteriores, armamos la siguiente tabla.

 Intervalo Valor de prueba x f 0 (x) = 3x2 − 4x − 5 Conclusión


√ 
−2; 4−6 76 −1 f 0 (−1) =2>0 f crece.
 √ √ 
4− 76 4+ 76
; 0 f 0 (0) = −5 < 0 f decrece.
6 √ 6
4+ 76
6 ;3 2, 5 f 0 (2, 5) = 3, 75 > 0 f crece.

Para hallar los intervalos de concavidad resolvemos la ecuación


f 00 (x) = 0
Entonces
(3x2 − 4x − 5)0 = 0
6x − 4 = 0
4
x=
6
Simplificamos, y resulta x = 32 . Ubicamos en el eje X el valor obtenido y armamos la
siguiente tabla.

Intervalo Valor de prueba x f 00 (x) = 6x − 4 Conclusión


−2; 23 f 00 (0) = −4 < 0

0 f cóncava negativa.
2
3; 3 1 f 00 (1) = 2 > 0 f cóncava positiva.

Observamos que en x = 23 hay un punto de x y


inflexión ya que la gráfica cambia la −2 0
concavidad. 3√ 0
Construimos una tabla con los máximos, 4− 76
6

8, 2
mínimos, puntos de inflexión y extremos del 4+ 76
−4, 06
6
dominio. Entonces la gráfica es la siguiente: 2
2, 07
3

131
Problema 394. Dada la función f (x) = 14 x4 − 2x2 definida en el intervalo [−3; 3].

a) Hallen los ceros de f .


b) Calculen los puntos críticos de f .
c) Clasifíquenlos según sean máximos o mínimos.
d ) Hallen los intervalos de crecimiento y decrecimiento.
e) Hallen los puntos de inflexión y los intervalos de concavidad y convexidad.
f ) Tracen la gráfica de f .

Resolución. Calculamos los puntos críticos. Para ello resolvemos la ecuación

f 0 (x) = 0

Es decir:  0
1 4
x − 2x2 =0
4
x3 − 4x = 0
Extraemos factor común y obtenemos

x(x2 − 4) = 0

Factorizamos y resulta
x(x + 2)(x − 2) = 0
Tenemos un producto entre tres factores igualado a cero, es decir que alugno de los
factores debe ser cero. Luego:

x = 0 ∨ x = 2 ∨ x = −2

Por lo tanto los puntos críticos son {0; 2; −2}.


Clasificamos los máximos y mínimos.

Punto crítico f 00 (x) = 3x2 − 4 f (x) Conclusión


0 −4 0 Es un máximo relativo.
2 8>0 −4 Es un mínimo relativo y absoluto.
−2 8>0 −4 Es un mínimo relativo y absoluto.
9
−3 23 > 0 4 Es un máximo absoluto.
9
3 23>0 4 Es un máximo absoluto.

Para hallar los intervalos de crecimiento y de decrecimiento marcamos en el eje X los


puntos críticos junto con los extremos del dominio y armamos la siguiente tabla.

Intervalo Valor de prueba f 0 (x) = x3 − 4x Conclusión


[−3; −2) −2, 5 f 0 (−2, 5) = −5, 62 < 0 Decrece
(−2; 0) −1 f 0 (−1) = 3 > 0 Crece
(0; 2) 1 f 0 (1) = −3 < 0 Decrece
(2; 3] 2, 5 0
f (2, 5) = 5, 62 > 0 Crece

Para hallar los intervalos de concavidad resolvemos la ecuación

f 00 (x) = 0

Es decir:
(x3 − 4x)0 = 0
3x2 − 4 = 0

132
Aplicamos la fórmula de Bhaskara y obtenemos
r r
4 4
x1 = ; x2 = −
3 3
Entonces completamos la tabla dividiendo el dominio de la función en los valores
obtenidos anteriormente.

Intervalo
q  Valor de prueba f 00 (x) = 3x2 − 4 Conclusión

−3; − 43 x = −2 f 00 (−2) =8>0 Cóncava positiva.
 q q 
− 43 ; 43 x=0 f 00 (0) = −4 < 0 Cóncava negativa.
 q 
− 43 ; 3 x=2 f 00 (2) = 8 > 0 Cóncava positiva.

A partir de los datos obtenidos en sendas tablas podemos esbozar una gráfica:

Problema 395. En la siguiente gráfica pueden observar dos funciones (seno y coseno).
Determinen
 3π qué  valores máximos y mínimos alcanzan ambas funciones en el dominio

Df = − 2 , 2 . En dicho dominio, esos máximos y mínimos, ¿son absolutos o relativos?

Respuestas. En la función y = sen(x) el máximo es y = 1 en los valores x1 = − 3π2 y


x2 = π2 (ambos son máximos absolutos). El mínimo es y = −1 en los valores x3 = − π2 y
x4 = 3π
2 (ambos son mínimos absolutos).
En la función y = cos(x) el máximo es y = 1 en los valores x1 = 0 (es máximo absoluto).
El mínimo es y = −1 en los valores x2 = −π y x3 = π (ambos son mínimos absolutos).

Problema 396. Indiquen los máximos y mínimos, tanto absolutos como relativos, de la
función f (x) = x3 − 4x2 (evaluada entre x = −3 y x = 5).
Respuestas. x1 = −3 es un mínimo absoluto. x2 = 0 es un máximo relativo.
8
x3 = 3 es un mínimo relativo. x4 = 5 es un máximo absoluto.

133
Problema 397. Calculen los puntos críticos de g(x) = 5 − 3x2 − 2x3 (entre x = −3 y
x = 2) y determinen si se tratan de extremos absolutos o relativos.
Respuestas. x1 = −3 es un máximo absoluto.
x2 = −1 es un mínimo relativo. x3 = 0 es un máximo relativo. x4 = 2 es un
mínimo absoluto.

Problema 398. Determinen todos los puntos críticos de las siguientes funciones.
Clasifíquenlos en máximos o mínimos absolutos y/o relativos:

a) f (x) = x2 en [−2; 1] b) g(x) = 8x − x4 en [−2; 1] c) h(x) = sen(x) en −π; 65 π


 

d) p(x) = |x| en [−2; 2] e) q(x) = − x12 en [0, 5; 2] f) r(x) = − x1 en [−2; −1]

Respuestas.

a) x1 = −2 es un máximo absoluto x2 = 0 es un mínimo absoluto.


b) x1 = −2 es un mínimo absoluto. x2 = 1 es un máximo absoluto.
c) x1 = −π es un mínimo absoluto. x2 = − π2 es un máximo absoluto. x3 = π
2 es
un máximo absoluto.
d ) x1 = −2 y x2 = 2 son míaximos absolutos. x3 = 0 es un mínimo absoluto.
e) x1 = 0, 5 es un mínimo absoluto. x2 = 2 es un máximo absoluto.
f ) x1 = −2 es un mínimo absoluto. x2 = −1 es un máximo absoluto.

Problema 399. Determinen los intervalos de crecimiento y de decrecimiento de la función


cuya gráfica pueden observar. Identifiquen también los puntos críticos.

Respuestas. Conjunto de crecimiento: (2; 3, 25). Conjunto de decrecimiento:


(−1; 2) ∪ (3, 25; 4). Puntos críticos en x = 1; x = 2; x = 3, 25; x = 4; x = 6.

Problema 400. Hallen los extremos locales de las siguientes funciones y determinen los
intervalos de crecimiento y decrecimiento.

134
1
a) f (x) = x3 − 4x en [−3, 5] c) f (x) = x + x en [−1, 1]
b) g(x) = 5 − 3x2 − 2x3 en [−3, 2] d ) g(x) = x3 − 12x − 5 en [−5, 5]

Respuestas. n o
−2 √2
a) Dominio:[−3, 5]. Puntos críticos: −3, √ 3
, 3
, 5 . Máx. locales en
   
− √23 ; 3, 079 ; (5; 105). Mín locales en (−3; −15); √23 ; −3, 079 . Crece en los
     
−2 √2 ; 5 y decrece en el intervalo √ −2 √2
intervalos −3; √ 3
y 3 3
; 3
.
b) Dominio: [−3, 2]. Puntos críticos:{−3, −1, 0, 2}. Máx. locales en (−3; 32); (0; 5); mín.
locales en (−1; 4); (2; −23). Crece en el intervalo (−1; 0) y decrece en los intervalos
(−3; −1) y (0; 2).
c) Dominio: [−1, 0) ∪ (0, 1]. Puntos críticos :{−1, 1}. Máx. local en (−1; −2); ;Mín.
local en (1; 2). Decrece en los intervalos (−1; 0) y (0; 1).
d ) Dominio: [−5, 5]. Puntos críticos:{−5, −2, 2, 5}. Máx. locales en (−2; 11); (5; 60).
Mín. locales en (−5; −70); (2; −21). Crece en los intervalos (−5; −2) y en (2; 5) y
decrece en el intervalo (−2; 2).

La segunda derivada

Recordemos que una función f (x) es creciente si y sólo si se verifica que


x2 > x1 → f (x2 ) > f (x1 ). Análogamente decimos que f (x) es decreciente si y sólo si se
verifica que x2 > x1 → f (x2 ) < f (x1 ).
Si la derivada de una función en un punto es positiva, entonces podemos afirmar que en
un entorno de ese punto, la función es creciente. Sin embargo, no es válida la
implicación contraria: Como contraejemplo, podemos notar que y = x3 es creciente en
todo su dominio, sin embargo, y 0 (0) = 0. Por lo tanto usaremos esta herramienta para
decidir si una función es creciente o decreciente, siempre que verifiquemos que la segunda
derivada de la función f en el punto dado no sea cero.
Una función alcanza su valor máximo cuando deja de crecer y empieza a decrecer; y
alcanza su valor mínimo cuando deja de decrecer y empieza a crecer. Dichos puntos se
denominan puntos críticos o estacionarios, en éstos la derivada es cero o bien no existe.

Criterio de la derivada primera para extremos.

Si una función es creciente para valores menores a x = x1 y es decreciente para valores


mayores a x = x1 entonces alcanza un máximo en x = x1 . El valor máximo de la función
en un entorno del punto es f (x1 ) .
Si una función es decreciente para valores menores a x = x1 y es creciente para valores
mayores a x = x1 entonces alcanza un mínimo en x = x1 y el valor mínimo de la función
f en un entorno del punto es f (x1 ).
La condición necesaria para la existencia de estos puntos es que no existe f 0 (x) o que
f 0 (x) = 0.
La segunda derivada60 f 00 de una función f tiene un par de aplicaciones interesantes:

Si a ∈ Dom(f ) es un punto de inflexión de f entonces f 00 (a) = 0.


Si f 00 (a) > 0 pero f 000 (a) 6= 0 entonces f es cóncava positiva61 .
Si f 00 (a) < 0 pero f 000 (a) 6= 0 entonces f es cóncava negativa62 .
60
Es decir, la derivada de la derivada de f .
61
O cóncava hacia arriba.
62
O cóncava hacia abajo.

135
Una condición suficiente para que a sea la abscisa de un punto de inflexión de la función
f es que f 00 (a) = 0 y f 000 (a) 6= 0.

Problema 401. Hallen los puntos de inflexión y determinen los intervalos de concavidad
positiva y concavidad negativa que presentan las gráficas de las funciones del problema
anterior.
Respuestas.

a) Cónc. Positiva en (0; 5) ; Cónc. negativa en (−3; 0); Punto de inflexión en x = 0.


b) Cónc. Positiva en (−3; −1/2) ; Cónc. negativa en (−1/2; 2) ; Punto de inflexión en
x = − 21 .
c) Cónc. Positiva en (0; 1); Cónc. negativa en (−1; 0); No tiene puntos de inflexión.
d ) Cónc. Positiva en (0; 5); Cónc. negativa en (−5; 0); Punto de inflexión en x = 0.
x3 −1
Problema 402. Considere la función f (x) = x2 −4x+3

a) Hallen el dominio, las raíces y la ordenada al origen.


b) Hallen analíticamente tres puntos de la función.
c) Hallen los candidatos a puntos de discontinuidad.
d ) Hallen los puntos de discontinuidad y clasifíquelos.
e) Hallen los máximos y los mínimos.
f ) Halle los intervalos de crecimiento y de decrecimiento.
g) Hallen los intervalos de positividad y negatividad.
h) Grafiquen la función en el GeoGebra y comparen los resultados obtenidos. Tengan
en cuenta que con el mencionado software puede marcar los extremos locales y las
raíces.

Respuestas.

a) Domf = R − {1; 3} Ordenada al origen: y = − 31 ; No tiene raíces.


b) A cargo del alumno.
c) x = 1; x = 3.
d ) Discontinuidad evitable en x = 1; Discontinuidad inevitable en x = 3.
√ √
e) Máx. relativo en (3 − 13; −0, 21); Mín. relativo en (3 + 13; 14, 21).
√ √
f ) Crecimiento: (−∞; 3√− 13) ∪ (3 + 13; +∞) √
Decrecimiento: (3 − 13; 1) ∪ (1; 3) ∪ (3; 3 + 13).
g) C + = (3, +∞); C − = (−∞; 1) ∪ (1; 3).

136
5.5. Optimización

Para resolver un problema de optimización, sugerimos seguir los siguientes pasos. Es


importante asegurarse de haber concluido un paso antes de continuar con el siguiente, así
como es fundamental no saltear pasos.
Acompañamos la explicación con la resolución de un problema, a modo de ejemplo.
Paso 1. Leemos atentamente el problema, desde el principio hasta el final. Marcamos las
ideas relevantes, los datos y las incógnitas.
Paso 2. Hacemos un esquema del problema, indicando los datos que marcamos en el
paso anterior. El esquema debe ser legible y entendible para todos.
Paso 3. Elegimos una de las incógnitas del problema (a menudo son dos o más) y la
denominamos con la letra x. Ésta será de aquí en adelante, la variable independiente. Le
asignamos un nombre al resto de las incógnitas (se sugiere evitar la y, ya que la usaremos
para nombrar la función, más adelante. Éstas serán de ahora en adelante, las incógnitas
auxiliares. Escribimos las elecciones tanto en forma analítica como en el esquema
confeccionado en el paso 2.
Paso 4. Establecemos, a modo de ecuaciones, las relaciones que existen entre las
incógnitas auxiliares y la variable independiente x. Despejamos cada una de las variables
auxiliares en función de x.
Paso 5. Nos planteamos la pregunta: ¿Qué es lo que queremos optimizar? La respuesta a
esta pregunta será, para nosotros, la función f . Escribimos en forma analítica la función
f , que depende de x y de las variables auxiliares. Una vez hecho esto, reemplazamos cada
variable auxiliar por la expresión obtenida en el paso 4. De esta manera obtenemos una
función f que depende únicamente de x.
Paso 6. Queremos hallar el o los extremos de la función y = f (x). Para ello buscamos su
dominio y los puntos críticos. Entonces resolvemos la ecuación f 0 (x) = 0. Desechamos las
soluciones de esta ecuación que no pertenezcan al dominio.
Paso 7. Armamos una tabla de valores para la función f con los números obtenidos en el
paso 6 y los extremos del dominio de la función f . Los clasificamos según sean máximos o
mínimos (locales o absolutos).
Paso 8. Volvemos a leer el problema. Identificamos la o las preguntas. Determinamos
cuál de todos los valores de x hallados en el paso 6 satisface lo pedido. Reemplazamos el
valor de x obtenido en cada una de las incógnitas auxiliares y en la función f .
Paso 9. Respondemos en palabras la o las preguntas del problema. Es importante incluir
las unidades de medida, o qué representa cada uno de los valores obtenidos.

Problema 403. Con una cartulina de 8cm × 5cm queremos construir una caja sin tapa, de
volumen máximo, cortando en cada esquina de la cartulina un cuadradito, que nos
permita rebatir las cuatro caras laterales. Halle las dimensiones de la caja que maximizan
el volumen.

Resolución.
Paso 1. Leemos atentamente el problema, desde el principio hasta el final. Marcamos las
ideas relevantes, los datos y las incógnitas.
Paso 2. Hacemos un esquema del problema, indicando los datos que marcamos en el
paso anterior. El esquema debe ser legible y entendible para todos.

137
Paso 3. Elegimos una de las incógnitas del problema (a menudo son dos o más) y la
denominamos con la letra x. Ésta será de aquí en adelante, nuestra variable
independiente. Le asignamos un nombre al resto de las incógnitas (se sugiere evitar la y,
ya que la usaremos para nombrar la función, más adelante. Éstas serán de ahora en
adelante, nuestras incógnitas auxiliares. Asentamos las elecciones tanto en forma analítica
como en el esquema confeccionado en el paso 2.

Lado del cuadradito que recortamos → x


Base de la caja → b
Profundidad de la caja → h
Altura de la caja → x

Paso 4. Establecemos, a modo de ecuaciones, las relaciones que existen entre las
incógnitas auxiliares y la variable independiente x. Despejamos cada una de las variables
auxiliares en función de x.

b + 2x = 8
Luego...
b = 8 − 2x
También
h + 2x = 5
Entonces
h = 5 − 2x
Paso 5. ¿Qué es lo que queremos optimizar? La respuesta a esta pregunta será, para
nosotros, la función f . Escribimos en forma analítica la función f , que depende de x y de
las variables auxiliares. Una vez hecho esto, reemplazamos cada variable auxiliar por la
expresión obtenida en el paso 4. De esta manera obtenemos una función f que depende
únicamente de x.

138
Queremos optimizar el volumen de la caja, que se calcula multiplicando la medida de la
base por la medida de la altura, por la medida de la profundidad. Es decir:

f =b·h·x

Reemplazamos las incógnitas auxiliares y obtenemos

f (x) = (8 − 2x) · (5 − 2x) · x

Paso 6. Queremos hallar el o los extremos de la función y = f (x). Para ello buscamos su
dominio y los puntos críticos. Entonces resolvemos la ecuación f 0 (x) = 0. Desechamos las
soluciones de esta ecuación que no pertenezcan al dominio.
El dominio de la función es Df = [0; 2, 5] dado que h debe ser no negativo, y x también.
Armamos la ecuación f 0 (x) = 0 y resolvemos. Para ello es conveniente -en este caso-
aplicar la propiedad distributiva de la multiplicación con respecto a la resta.

f (x) = (8 − 2x) · (5 − 2x) · x

f (x) = 40x − 26x2 + 4x3


f 0 (x) = 40 − 52x + 12x2 = 0
Resolvemos la ecuación cuadrática y obtenemos x1 = 1; x2 = 10 3 . La segunda solución no
pertenece al dominio, entonces sólo nos interesa x = 1
Paso 7. Armamos una tabla de valores para la función f con los números obtenidos en el
paso 6. Clasificamos según sean máximos o mínimos (locales o absolutos).
Notamos que x1 = 1 es un máximo y los extremos del dominio son mínimos.
Paso 8. Volvemos a leer el problema. Identificamos la o las preguntas. Determinamos
cuál de todos los valores de x hallados en el paso 6 satisface lo pedido. Reemplazamos el
valor de x obtenido en cada una de las incógnitas auxiliares y en la función f .
El problema pide el volumen máximo, entonces nos quedamos con x = 1.
Paso 9. Respondemos en palabras la o las preguntas del problema. Es importante incluir
las unidades de medida, o qué representa cada uno de los valores obtenidos.
El problema pide las dimensiones de la caja, entonces reemplazamos el valor de x
obtenido en las incógnitas auxiliares h y b. Luego, las dimensiones de la caja son:

Base: 6 cm (ya que b = 8 − 2x = 8 − 2 · 1 = 8 − 2 = 6)


Profundidad: 3 cm (ya que h = 5 − 2x = 5 − 2 · 1 = 5 − 2 = 3)
Altura: 1 cm (ya que x = 1)

Y de esta manera queda resuelto el problema.

Problema 404. Sobre la orilla recta de un canal se precisa limitar un terreno rectangular
alambrando los 3 lados que no pertenecen a la orilla. Hay que emplear 1800 metros de
alambre tejido. ¿Cuáles deben ser las dimensiones del terreno para que tenga área
máxima?
Respuestas. 450 metros de ancho y 900 metros de largo.

Problema 405. Entre todos los pares de números positivos cuyo producto es 144
encuentren dos cuya suma es mínima.
Respuestas. Ambos números deben ser iguales a 12.

Problema 406. Con una cartulina de 80 centímetros de largo y 50 centímetros de ancho


queremos construir una caja rectangular sin tapa, cortando cuadrados en los vértices.
Calculen las dimensiones de la caja de manera que su volumen sea máximo.
Respuestas. La caja debe medir 60cm × 30cm × 10cm.

139
Problema 407. Entre los pares de números positivos cuya suma es 30, encuentren
aquellos dos cuyo producto es máximo.
Respuestas. Ambos deben ser iguales a 15.

Problema 408. Queremos fabricar latas cilíndricas con tapa de 250 centímetros cúbicos
de capacidad. ¿Cuáles deben ser las dimensiones que minimizan la cantidad de material?
Respuestas. r = 3, 42cm; h = 6, 84cm.

Problema 409. Una ventana tiene forma rectangular, coronada por un semicírculo sobre
uno de los lados. El perímetro es igual a 8 metros. Calculen las dimensiones de manera
que el área de la ventana sea máxima, y pueda pasar la mayor cantidad de luz.
Respuestas. El rectángulo debe medir 2, 24m × 1, 12m.

Problema 410. Inscribimos un rombo en un rectángulo de perímetro igual a 100. ¿Cuáles


deben ser las dimensiones del rectángulo que maximizan el área del rombo?
Respuestas. El rectángulo debe ser un cuadrado de lado igual a 25.

Problema 411. Consideramos la función f (x) = 3 − x2 y un punto M de su gráfica


situado en el primer cuadrante. Trazamos rectas paralelas a los ejes por M que los cortan
en los puntos A y B. Hallen las coordenadas del punto M tales que el rectángulo OAM B
tiene área máxima.
Respuestas. M = (1; 2).

Problema 412. De todos los prismas rectos de base cuadrada, tales que el perímetro de
una cara lateral es de 30 centímetros, hallen las dimensiones del que tiene volumen
máximo.
Respuestas. Es el prisma de 10 cm × 10 cm × 5 cm.

Problema 413. De todos los rectángulos de diagonal igual a 6 · 2 encuentren las
dimensiones del de perímetro máximo.
Respuestas. Es el cuadrado de lado igual a 6.

Problema 414. Se necesita diseñar una lata sin tapa con capacidad para un litro, con
forma de cilindro. ¿De qué dimensiones debe ser la lata para usar la menor cantidad de
material posible?
Respuestas. El radio es 6, 82cm al igual que la altura. La superficie mínima es
439, 39cm2 .

Problema 415. Queremos hacer una caja abierta cortando cuadrados en las esquinas de
una hojalata cuadrada que mide 30cm de lado, y doblando los lados hacia arriba. ¿Qué
tan grandes deben ser los cuadrados que se corten para que la caja tenga la máxima
capacidad posible?
Respuestas. 5 cm.

Problema 416. Un rectángulo se inscribe en un semicírculo de radio 2. ¿Cuál es el área


máxima que puede tener el rectángulo
√ y cuáles
√ son sus dimensiones?
Respuestas. El rectángulo es de 2 × 2 2 y su superficie es de 4 unidades cuadradas.

Problema 417. Dos postes de 12 y 28 metros de altura, distan 30 metros entre sí. Hay
que conectarlos mediante un cable que esté atado en algún punto del suelo entre los
postes. ¿En qué punto ha de amarrarse al suelo con el fin de utilizar la menor longitud de
cable posible? Respuestas. A 21 metros del poste más alto.

Problema 418. Deben hacerse dos chapas cuadradas de dos materiales distintos. Dichos
materiales tienen precios respectivamente de 2 y 3 pesos por centímetro cuadrado.
Calculen los lados de los cuadrados si queremos que el costo total sea mínimo y si además
se requiere que la suma de los perímetros de los dos cuadrados ha de ser de un metro.
3 1
Respuestas. Debe medir 20 m la de $2 y 10 m la de $3.

140
Problema 419. Una plataforma de perforación a 12 km de la costa debe ser conectada
mediante un oleoducto a una refinería que está a 16 km en línea recta desde el punto
costero más cercano a la plataforma. Si instalar la tubería debajo del agua cuesta
$500.000 por km, y en tierra cuesta $300.000 por km, ¿qué combinación de instalación
subactuática y terrestre ofrece la conexión más barata?
Respuestas. Son 7 km por tierra y 15 km por agua.

Problema 420. Calculen la base y la altura de √


un triángulo isósceles de perímetro 8 y
8 4 3
área máxima. Respuestas. Base= 3 ; altura= 3 .
Problema 421. Con una cartulina de 8 × 5 queremos construir una caja sin tapa, de
volumen máximo. Halle las dimensiones de la caja que maximizan el volumen.
Respuestas. 3 × 1 × 6.
Problema 422. Un rectángulo está acotado por los semiejes coordenados positivos y por
la gráfica de la recta y = 6−x
2 . Qué longitudes debe tener los lados del rectángulo para
que su área sea máxima? Respuestas. 3 × 1, 5.
Problema 423. ¿Qué puntos de la gráfica de la parábola de ecuación y = 4 − x2 están
más cerca del punto (0; 2)?
La distancia
p entre el punto de coordenadas (x0 ; y0 ) y el punto de coordenadas (x1 ; y1 ) es
d = (x1 − x0 )2 + (y1 − y0 )2 q
 
3 5
Respuestas. Los puntos son 2 2 y su simétrico respecto al eje Y .
;

Problema
√ 424. Un rectángulo está limitado por el eje X y por el semicírculo de ecuación
y = 25 − x2 . Determine las dimensiones√del rectángulo
√ que maximizan su área.
5
Respuestas. Es el rectángulo de lados 2 2 × 5 2.
Problema 425. Una página rectangular debe contener un dibujo rectangular de 24dm2 ,
con márgenes superior e inferior de 1, 5dm y márgenes laterales de 1dm. ¿Qué
dimensiones de la página requieren la mínima cantidad de papel? Respuestas.
6 dm × 9 dm.
Problema 426. Con cuatro metros de alambre queremos construir un círculo y un
cuadrado. Cuánto alambre hay que emplear en cada figura para lograr que la suma de las
áreas de ellas sea la mínima posible? Y si quisiéramos que la suma sea la máxima
posible?
Respuestas. El lado del cuadrado que minimiza el área es aproximadamente 0, 56m y el
radio del círculo es aproximadamente 0, 28 m. Para maximizar el área, el radio del círculo
es igual a π2 m y el cuadrado degenera en un punto.
Problema 427. Tenemos que diseñar una lata cilíndrica, con tapa, de 4 litros. Determine
las dimensiones para que su área total sea mínima.
Respuestas. La lata debe medir 17, 2 cm de alto y 8, 6 cm de radio.

141
Problema 428. Queremos inscribir un cilindro en una esfera de radio igual a 1 metro.
Hallen las dimensiones del cilindro de forma tal que:

a) El volumen sea máximo.


b) El área lateral sea máxima.

Respuestas. a) r ∼
= 0, 817 m; h ∼
= 1, 15 m b) r ∼
= 0, 707 m; h ∼
= 1, 41 m.

Problema 429. El alcance R de un proyectil lanzado con velocidad inicial v0 y un ángulo


de elevación θ es R = g1 · v0 2 · sen(2θ) siendo g la aceleración de la gravedad. Calculen el
ángulo θ que optimiza el alcance. Respuestas. θ = π4 .

Problema 430. Se lanza un cuerpo hacia arriba con velocidad inicial igual a 40 ms.
m
Calculen cuál es la máxima altura que alcanzará. Considere g = 10 s2 y la altura en
función del tiempo dada por la ecuación h(t) = vt − 12 gt2 . Respuestas. 80 metros.
Problema 431. Un túnel tiene una entrada cuyo corte
frontal es una porción de parábola, de coeficiente
1
principal igual a − . La altura máxima es de 8
2
metros. La concesionaria del túnel sabe que por él
entran camiones de, a lo sumo, 3, 5 metros.
Cierto día decide vender un espacio de publicidad
que consiste en un cartel rectangular luminoso. Los
vértices superiores del cartel deben estar amurados a
la entrada del túnel. Por supuesto, se espera que el
cartel no impida la entrada de los vehículos.

¿Cuál es la mayor área que puede tener dicho cartel publicitario?

5.6. Funciones implícitas y sus derivadas

Una correspondencia, relación o función está definida implícitamente cuando no está


despejada la variable dependiente y en función de la variable independiente x. Por
ejemplo, la ecuación 6x + 3y = 7 define en forma implícita a la recta de ecuación
y = −2x + 37 . Esto se puede verificar fácilmente, despejando y en la primera ecuación.
Pero existen casos en los que es muy difícil, o peor, imposible despejar la variable y. Por
ejemplo, en la ecuación y − x + ln(y + 1) = 0. Es decir, no existen herramientas
algebraicas que nos permitan hallar una expresión del tipo y = f (x) equivalente a la
anterior.
Sin embargo, la ecuación dada define una función en la que y depende de x. Su gráfica la
pueden obtener ingresando la función en cualquier graficador (por ejemplo, en el
GeoGebra).
En ocasiones necesitamos analizar ciertas características de dichas funciones, como sus
extremos locales, el crecimiento, etc. . . En esta sección nos proponemos calcular la
derivada de las funciones expresadas en forma implícita. Para ello es imprescindible tener
al alcance la tabla de las derivadas y sus propiedades.
La primera consideración que debemos hacer es la siguiente:

La variable y es una variable dependiente de x.

Entonces, la derivada de 3y 5 no es 3 · 5 · y 5−1 , ya que y es una función que depende de x.


Para calcular la derivada de 3y 5 aplicamos la regla de la cadena:

(3y 5 )0 = 3 · 5 · y 5−1 · y 0 = 15y 4 y 0

142
Problema 432. Calculen la derivada de la función implícita de ecuación
6x2 y + 3y 5 + 3x2 = 12 − x2 y 2 .

Resolución. Derivamos ambos miembros de la igualdad y aplicamos la propiedad de la


derivada de las sumas y restas de funciones.
(6x2 y)0 + (3y 5 )0 + (3x2 )0 = 120 − (x2 y 2 )0
Calculamos la derivada de cada uno de los términos.
En el primer término observamos un producto de funciones, por lo tanto aplicamos la
propiedad correspondiente.
(6x2 y)0 = (6x2 )0 · y + 6x2 · (y)0 = 12x2−1 · y + 6x2 · y 0 = 12xy + 6x2 y 0
El segundo término lo hemos derivado al comienzo de la sección.
El tercer término depende únicamente de x por lo tanto, su derivada es 6x.
En el miembro de la derecha, notamos que 120 = 0. Para derivar x2 y 2 aplicamos
nuevamente la propiedad de la derivada del producto entre dos funciones.
(x2 y 2 )0 = (x2 )0 · y 2 + x2 · (y 2 )0 = 2x · y 2 + x2 · 2y 2−1 · y 0 = 2xy 2 + 2x2 yy 0
Entonces resulta
(12xy + 6x2 y 0 ) + (15y 4 y 0 ) + (6x) = 0 − (2xy 2 + 2x2 yy 0 )
Si queremos despejar y 0 en función de x y de y obtenemos
−12xy − 6x − 2xy 2
y0 =
6x2 + 15y 4 + 2x2 y
Propiedad. Sea una función y = f (x) definida en forma implícita por la fórmula
0
F (x; y) = 0 entonces y 0 = − FFx0 .
y

Consideraciones:
La función F no es la misma que la función f .
Con Fx0 indicamos la derivada de la función F con respecto a la variable x (es decir
que la y es constante.
Con Fy0 indicamos la derivada de la función F con respecto a la variable y (es decir
que la x es constante.
Problema 433. Calculen la derivada de la función implícita de ecuación
6x2 y + 3y 5 + 3x2 = 12 − x2 y 2 utilizando la fórmula de la derivada implícita.

Resolución. Escribimos la ecuación que define la función.


6x2 y + 3y 5 + 3x2 = 12 − x2 y 2
Igualamos a cero, para definir la función F .
6x2 y + 3y 5 + 3x2 − 12 + x2 y 2 = 0
Entonces el miembro de la izquierda es la función F . Calculamos las dos derivadas63 :
Fx0 = 6y(x2 )0 + 0 + 6x − 0 + y 2 (x2 )0 = 12xy + 6x + 2xy 2
Fy0 = 6x2 · 1 + 15y 4 + 0 − 0 + x2 (y 2 )0 = 6x2 + 15x4 + 2x2 y
Reemplazamos en la fórmula y obtenemos
Fx0 12xy + 6x + 2xy 2
y0 = − = −
Fy0 6x2 + 15x4 + 2x2 y

Problema 434. Calculen las derivadas de las siguientes funciones.


63
Recordamos que, cuando derivamos respecto a una de las variables, la otra la consideramos constante.

143
 
a) x2 + xy = y 2 . e) ln x+y
=2
x2 −y 3
b) ln(xy 2 ) − y3 = x3 f ) sen(x + y) + ey = y
c) exy+y + xy = ln(2) g) x2 + 2xy + y 2 + y − x = 0

d ) 2x y − 3x2 + 7xy − 1 = 0 h) xy 2 − x3 + y − 1 = 0

Respuestas.
2x+y x2 +y 3 +2xy
a) y 0 = 2y−x e) y 0 = x2 +2y 3 +3xy 2
3x2 − x1
b) y 0 = 2
−3y 2
cos(x+y)
f ) y 0 = − cos(x+y)+ey −1
y
−yexy+x −y
c) y 0 = (x+1)exy+x +x g) y 0 = − 2x+2y−1
2x+2y+1

6x−7y−2 y
d ) y0 = x
√ +7x
−3x
h) y 0 = − y2xy+1
2 2
y

Si buscan más problemas de derivadas de funciones implícitas, pueden consultar en


http://fernandorevilla.es/blog/2014/04/02/
derivacion-de-funciones-implicitas/.

144
Autoevaluación

1. Calcule la derivada de la función f (x) = x+1


x aplicando la definición de la derivada.
2. Calcule la derivada de las siguientes funciones aplicando las propiedades de las
derivadas.

f (x) = x2 + 1 · e2x−4 g(x) = sen[ln(x)]

5x + 3 x

3. Halle los extremos locales y los extremos absolutos de la función f (x) = x2 · ex en el


intervalo [−2; 5].
4. Halle la ecuación de la recta tangente a la función f (x) = x3 + x − 1 en el punto
(2; f (2)).
5. Calcule el lı́m ln(x)−sen(πx)
x2 −1
.
x→1
6. Queremos construir un cantero con una superficie de 100m2 como indica la figura (el
triángulo es isósceles). Determine las medidas de cada uno de los lados, de manera tal
que el perímetro sea mínimo.

Respuestas. 1) f 0 (x) = − x12 .


 √ 
2) f 0 (x) = e2x−4 √xx2 +1 + 2 x2 + 1 .
cos(ln(x)) √  2
(5x + 3 x)−sen(ln(x))· 5x ln(5)+ 13 x− 3
·
g 0 (x)
= x
√ 2
(5x + 3 x)
3) Mínimo absoluto en (0; 0); máximo absoluto en (5; 25e5 ); mínimo relativo en (−2; 4e−2 .
4) y = 13x − 17. 5) L = 1+π 2 .
6) b = 6, 44 cm; h = 9, 10 cm; H ∼
∼ ∼ = 12, 87 cm.

145
******

Integrales
6. Integrales

Sugerimos la lectura del siguiente apunte teórico y sus ejemplos:


http://fernandorevilla.es/blog/2014/04/03/diferencial-de-una-funcion/

Las funciones primitivas

Decimos que la función F (x) es una primitiva de f (x) si se verifica que F 0 (x) = f (x).
Notamos que si F es una primitiva de f , entonces F + k es también una primitiva (para
cualquier k ∈ R).
Por ejemplo, las funciones F1 (x) = x3 y F2 (x) = x3 + 5 son primitivas de la función
f (x) = 3x2 ya que F1 0 = F2 0 = f
R la primitiva de una función f la denominamos también integral y la simbolizamos
A
f · dx. Más adelante comentaremos el origen de la notación y algunos de sus
significados y aplicaciones.

Problema 435. Verifiquen (derivando), que las funciones resultantes son las primitivas de
las funciones dadas64 .
x10
x9 dx =
R R
a) 10 g) sen(x)dx = −cos(x)
x10
R
x9 dx sen(x)dx = −cos(x) + ln(3)
R
b) = 10 +1 h)
sen(x)dx = −cos(x) + 23
R
x10 i)
x9 dx =
R
c) 10 −3
ex dx = ex
R
j)
R
d) cos(x)dx = sen(x)
ex dx = ex + sen(5)
R R
e) cos(x)dx = sen(x) − 4 k)
ex dx = ex + sen(5) + 0, 3
R R
f) cos(x)dx = sen(x) + 7 l)

Observemos que la única diferencia entre las tres primeras integrales es una constante,
que la representamos con C o bien k, y la denominamos constante de integración.
Observación: Cada función admite infinitas primitivas de f que difieren en una constante.

Problema 436. Utilicen las propiedades de las derivadas para verificar las siguientes
integrales indefinidas:
−1
(7x−2)4
(7x − 2)3 dx = (3x + 5)−2 dx = − (3x+5)
R R
a) 28 +C b) 3 +C

sec2 (5x − 1) dx = 15 tan (5x − 1) + C csc2 x−1 x−1


R R  
c) d) 3 dx = −3cot 3 +C

1 1 x
dx = 12 ln(x2 + 1) + C
R R
e) (x+1)2
dx = − x+1 +C f) x2 +1

Teorema Fundamental del Cálculo Integral

Si F1 y F2 son funciones primitivas de f , entonces difieren en una constante.


Demostración. F1 y F2 son funciones primitivas de f , entonces F10 = f y F20 = f .
Denominamos G = F1 − F2 y derivamos a ambos miembros:
G0 = F10 − F20 → G0 = f − f = 0. Por lo tanto G0 = 0 y G = C.
Otras formas de enunciar el Teorema fundamental del cálculo integral son:

Todas las funciones que tienen igual derivada difieren entre sí en una constante.
Todas las primitivas de una misma función difieren entre sí en una constante.
64
Los resultados que proponemos surgen de usar una tabla de integrales y suponer que para cada uno
de ellos la constante de integración tiene distintos valores.

147
Definición. Denominamos integral indefinida de una función a sus infinitas primitivas.

Las integrales inmediatas son las que calculamos usando directamente la tabla de
integrales.
Ejemplos (*):
4 3x
x3 dx = x4 + C 3x dx =
R R R
a) dx = x + C d) g) ln(3) +C
R√ 1,5
x2 e) xdx = x1,5 + C 8x
R
8x dx =
R
b) xdx = 2 +C h) ln(8) +C
x3
R √ 4
x2 dx = xdx = 3x43 + C
R 3
R
c) 3 +C f) i) 7 dx = 7x + C

¿Cómo sabemos si hemos integrado bien? Para verificar una integral tenemos que
derivar la función obtenida y verificar que coincide con la "función original".
2
Problema 437. Verifiquen la siguiente igualdad: xdx = x2 + C
R

x2
Resolución. Para verificar, derivamos 2 + C, entonces:
0
x2

1
+C = · 2 · x2−1 + 0 = x
2 2

Hemos obtenido x que es la función que debíamos integral, por lo tanto hemos integrado
bien.
R√ 1,5
Problema 438. Verifiquen la siguiente integral: xdx = x1,5 + C
x1,5
Para verificar, derivamos 1,5 + C, entonces:
0 3
x1,5 x 2 −1 √

1
+C = 1, 5 · + 0 = x2 = x
1, 5 1, 5

Una vez que hallamos todas las funciones tal que su derivada coincida con la expresión
dada hemos encontrado todas las funciones que cumplen esa condición.

Problema 439. Verifiquen las integrales que figuran en los ejemplos (*) en la página
anterior.

Problema 440. Encuentren una función primitiva para las siguientes funciones y
verifiquen el resultado a partir de la derivación.

a) f (x) = 5 e) f (x) = 2x + 1 i) f (x) = − 32 x−5/2


b) f (x) = 6x f ) f (x) = x7 − 6x + 8 j ) f (x) = 3sen x
5
c) f (x) = −3x−4 g) f (x) = 2 − x2 k ) f (x) = −πsen πx
√ √
d ) f (x) = 34 3 x h) f (x) = x + √1 l ) f (x) = cos πx
x 2

Respuestas.
3
a) I = 5x + k e) I = x2 + x + k i) I = x− 2 + k
b) I = 3x2 + k f ) I = 18 x8 − 3x2 + 8x + k j ) I = −3cos(x) + k
5
c) I = x−3 +k g) I = 2x + x k ) I = cos(πx) + k
4
2 32 1
l ) I = π2 sen πx

d) I = x + k3 h) I = 3x + 2x + k 2
2 +k

Problema 441. Habitualmente podemos resolver una integral aplicando las propiedades
enunciadas en la Tabla de integrales. Indiquen qué propiedad o propiedades aplicamos
para resolver cada una de las siguientes integrales.

148
3 2
(x2 + x)dx = x2 dx + xdx = x3 + x2 + C
R R R
a)
2
(x − ex )dx = xdx − ex dx = x2 − ex + C
R R R
b)
R 1 √ R√ 3
( x + x)dx = x1 dx + xdx = ln(x) + x32 + C
R
c)
2
9x
R x
[9 + cos(x)]dx = 9x dx + cos(x)dx = ln(9)
R R
d) + sen(x) + C
2
2xdx = 2 xdx = 2 · x2 + C = x2 + C
R R
e)
R R
f) edx = e dx = ex + C
R R
g) sen(5)dx = sen(5) dx = xsen(5) + C
5
πx4 dx = π x4 dx = π x5 + C
R R
h)
(x − 1)2 dx = (x2 − 2x + 1)dx = x2 dx − 2xdx + dx =
R R R R R
i)
3 2 3
= x3 − 2 x2 + x + C = x3 − x2 + x + C

Problema 442. Resuelvan las siguientes integrales aplicando las propiedades. Anoten en
cada paso, el número de la propiedad que aplicaron.

a) x3 dx = h) (6x − 1)3 dx =
R R

b) x+7
R
dx = R 4 3 −2
i) 3x −5x dx =
R 2x2−x x
c) 9 dx = h i
−2 + cos(x) dx =
R
R
d ) [−x + cos(x)]dx = j ) e cos(2)
R ex +(x+4)2 R √
 
e) π dx = k ) x + √10
dx =
R x
f ) [−(2x − 3) − πsen(x)]dx = √
R 3 2

5 − x43 − 5 9x dx =
R
g) [ln(7) − 7cos(x)]dx = l)

Soluciones.

a) x3 dx = 3 x1 dx = 3ln(x) + C
R R

Hemos extraído el número 3, de la integral, que es constante. Luego integramos por


tabla.
2 2
b) x+7 1 1
xdx + 7dx = 12 · x2 + 21 · 7x + C = x4 + 27 x + C
R R R R 
2 dx = 2 (x + 7)dx = 2
Hemos extraído el número 12 , de la integral, que es constante. Luego distribuimos la
integral en la suma. Finalmente integramos por tabla término a término,
distribuimos el factor 21 y realizamos los cálculos.
R x  x 2

2x 2
c) 2 9−x dx = 19 (2x −x)dx = 91 ( 2x dx− xdx) = 19 ln(2) 2
− x2 +C = 9ln(2) − x18 +C
R R R

Los pasos seguidos en la resolución de este ejercicio son similares a los realizados en
el ejercicio anterior).
2
d ) [−x + cos(x)]dx = −xdx + cos(x)dx = −x
R R R
2 + sen(x) + C
Distribuimos la integral en la suma. Luego integramos por tabla término a término.
R x 2
e) e +(x+4) dx = π1 e dx + (x + 4)2 dx = π1
R x R x
e dx + (x2 + 8x + 16)dx =
R  R 
π
1 x x3 x2
π (e + 3 + 8 2 + 16x + C
En este ejercicio a diferencia de los anteriores tuvimos que desarrollar el cuadrado
del binomio x + 4. Luego la resolución es similar a lo realizado en los ejercicios
precedentes.
f ) [−(2x−3)−πsen(x)]dx = −2 xdx+3 dx−π sen(x)dx = −x2 +3x+πcos(x)+C
R R R R
R R R
g) [ln(7) − 7cos(x)]dx = ln(7) dx − 7 cos(x)dx = xln(7) − 7sen(x) + C
h) (6x − 1)3 dx = (216x3 − 108x2 + 18x − 1)dx = 216 x3 dx − 108 x2 dx +
R R R R
4 3 2 4
18 xdx − dx = x4 − 108 x3 + 18 x2 − x + C = x4 − 36x3 + 9x2 − x + C
R R

En este ejercicio hemos desarrollado el cubo del binomio 6x − 1. Luego resolvimos de


la misma manera que en los ejercicios anteriores.

149
3x4 −5x3 −2
R 4 R 3 4
= 34 xx dx − 5 xx dx − 2 x1 dx = x4 − 53 x3 − 2ln(x) + C
R R
i) x dx
Aplicamos la propiedad distributiva de la división en la suma y en la resta. Luego
usamos propiedades de potencias de igual base y de las integrales. Finalmente
integramos por tabla.
R h −2 cos(x) i
e + cos(2) dx = e−2 dx + cos(2) 1
cos(x)dx = e− 2x + cos(2)
1
R R
j) sen(x) + C
R √  R√ 3 √
x + √10x dx = xdx + 10 √1x dx = 32 x 2 + 20 x + C
R
k)
R √
3 2
 R 2 5
5 − x43 − 5 9x dx = 5 dx − 4 x−3 dx − 95 x 3 dx = 5x + x22 − 13 x 3 + C
R R
l)

Problema 443. Calculen las siguientes integrales aplicando las propiedades de la


integración.
2
(3x2 − 9) · dx = ( x x+1 ) · dx =
R R
a) e)
1
· sen(x) + x3 − 5] · dx =
R
(x − 4)(2 − x3 ) · dx =
R
b) f) x [x
R
c) (x + 1) x12 + 4 · dx =
R 
g) dx =
R √5
d ) [3ex − cos(x) + 3] · dx =
R
h) x2 · dx =

Respuestas.

a) I = x3 − 9x + k e) I = 12 x2 + ln|x| + k
b) I = x2 − 15 x5 − 8x + x4 + k x3
f ) I = −cos(x) + 3 − 5ln|x| + k
c) I = ln|x| + 2x2 − x−1 + 4x + k g) I = x + k
7
d) I = 3ex − sen(x) + 3x + k h) I = 57 x 5 + k

Concluimos que aplicando las propiedades de las operaciones y de la integral, algunas


transformaciones algebraicas y usando escrituras equivalentes podemos transformar en
integrales inmediatas muchas integrales que no están en la tabla de integrales.
También es cierto que no todas las integrales se pueden transformar en inmediatas.
Entonces para resolver integrales no inmediatas tenemos que usar los llamados "métodos
de integración", que son procedimientos mediantes los cuales transformamos integrales no
inmediatas en inmediatas.

Ecuaciones diferenciales

Para comenzar esta sección, les sugerimos la lectura de los siguientes apuntes y les
recomendamos que resuelvan los problemas propuestos.

http://fernandorevilla.es/blog/2014/02/11/
concepto-de-ecuacion-diferencial-ordinaria/
http://fernandorevilla.es/blog/2014/02/24/
ecuacion-diferencial-de-variables-separadas/

150
Problema 444. Resuelvan las siguientes ecuaciones diferenciales:
( (
dy dy
= 2x − 7 = 3x−2/3
a) dx d ) dx
y(2) = 0 y(−1) = −5
( (
dy ds
= 10 − x = 1 + cos t
b) dx e) dt
y(0) = −1 s(0) = 4
( (
dy
= x12 + x ds
= sen t + cos t
c) dx f ) dt
y(2) = 1 s(π) = 1

Respuestas.

a) y(x) = x2 − 7x + 10 d ) y(x) = 9 3 x + 4
x2
b) y(x) = 10x − 2 −1 e) s(t) = t + sen(t) + 4
x2
c) y(x) = − x1 + 2 − 1
2 f ) s(t) = −cos(t) + sen(t)

Regla de Barrow. Si f es continua en el intervalo [a; b] y F es cualquier primitiva de f


en dicho intervalo, entonces
Z b
f (x) · dx = F (b) − F (a)
a

Suma de Riemann

Supongamos que queremos calcular el área encerrada entre la función y = x2 , las rectas
verticales x = 1; x = 3 y el eje X.

Una forma de aproximar el área65 es dividirla en pequeños rectángulos. De esta manera,


el área que buscamos es mayor que la suma de las áreas de los rectángulos que están
debajo de la curva...
65
En realidad estamos acotando el área entre dos valores.

151
pero menor que la suma de las áreas de los rectángulos que están por encima de la curva.

Notamos que los rectángulos tienen bases iguales a 0,5 y la altura varía dependiendo de
f (x) para cada x. Entonces podemos afirmar que, si el área que buscamos es A:

0, 5·f (1)+0, 5·f (1, 5)+0, 5·f (2)+0, 5·f (2, 5) < A < 0, 5·f (1, 5)+0, 5·f (2)+0, 5·f (2, 5)+0, 5·f (3)

Equivalentemente
6, 75 < A < 10, 75
Pero las aproximaciones66 que obtuvimos tienen márgenes muy amplios. ¿Qué podemos
hacer para obtener cotas más cercanas?
Una posibilidad consiste en dividir la región A en más rectángulos, de modo que la
diferencia entre la suma de los rectángulos superiores y la suma de los rectángulos
inferiores no sea tan amplia. Si tomamos ocho rectángulos obtenemos las siguientes
sumas.

66
O cotas.

152
Suma de las áreas de los rectángulos inferiores = 7,69.
Suma de las áreas de los rectángulos superiores = 9,69.

De esta manera, la cota del área es bastante mejor a la que obtuvimos anteriormente.
Evidentemente, cuanto mayor sea el número de rectángulos, mejor es la aproximación.
Mostramos la imagen con 40 rectángulos:

En este caso la suma de las áreas de los rectángulos inferiores es igual a 8,47 y la suma de
las áreas de los rectángulos superiores es 8,87. Esto, por ejemplo, nos asegura que el
primer dígito del número A que estamos buscando es igual a 8.
Observamos que, en general, la suma de las áreas de los rectángulos que están debajo de
la curva es
X n
Sinf = h·b
k=1

siendo h la altura de cada uno de los rectángulos y b la base correspondiente.


Para cada uno de los casos, la base b coincide con el ∆x y la altura es igual al valor de
(xk ) para cada k. Entonces
Xn
Sinf = f (xk ) · ∆x
k=1

Análogamente, la suma de los rectángulos que acotan superiormente al área debajo de la


curva es
n−1
X
Ssup = f (xk ) · ∆x
k=o

Si tomamos el límite con ∆x → 0 resulta que Sinf = Ssup y lo escribimos


Z 3
S= f (x) · dx
1

Hasta ahora tenemos una idea de cómo hallar el área buscada. Pero... ¿cuál es la relación
entre esto y las derivadas?
Recordamos que, según la definición de la derivada

f (x + ∆x) − f (x)
f 0 (x) = y 0 = lı́m
∆x→0 ∆x
Equivalentemente
dy
y0 =
dx
Entonces
y 0 · dx = dy

153
Integramos ambos miembros de la igualdad.
Z Z
0
y · dx = dy

El miembro de la derecha es igual a y.


Entonces concluimos que la integral es la aplicación inversa de la derivada.
La aplicación en GeoGebra de la integral la podés descargar en
https://drive.google.com/open?id=1Odriah2IiVKqHOi3-FOO8qLjm8OdSjoM. En el
archivo, podés modificar la función, los extremos de integración y (con el deslizador)
podés variar el número de rectángulos.

6.1. ¿Cómo calculamos el área determinada por curvas?

Una de las aplicaciones de las integrales tiene como propósito calcular el área
determinada por dos o más curvas (eventualmente, una de ellas puede ser el eje de
abscisas). Analizamos cinco situaciones.
a) Si toda la región se encuentra delimitada entre una curva y el eje X en un mismo
semiplano con respecto a dicho eje.
Por ejemplo: Halle el área delimitada por la gráfica de la función y = x2 entre x = 2
y x = 5.
Notamos (a partir del gráfico) que toda la región cuya área queremos calcular se
encuentra en el mismo semiplano con respecto al eje X. Entonces
Z 5
A= x2 · dx
2

Calculamos la primitiva y aplicamos la regla de Barrow.


 3 5
x
A= +C
3 2

53 23
A= −
3 3
Entonces A = 125 8
3 − 3 = 39
b) Si toda la región se encuentra delimitada entre una curva y el eje X en distintos
semiplanos con respecto a dicho eje.
Por ejemplo: Halle el área delimitada por la gráfica de la función y = x2 + 2x − 3
entre x = −1 y x = 3.
Notamos que una de las raíces de la función es x = 1 que es un valor que pertenece
al intervalo [−1; 3]; es decir que una parte de la región se encuentra debajo del eje X
y otra parte se encuentra por encima del eje X.

154
Entonces debemos partir el área en dos regiones, y calcular sus áreas por separado.
Recordemos que el área de una región siempre es un número positivo.
Calculamos el área A1 entre x = −1 y x = 1.
Z 1
A1 = (x2 + 2x − 3) · dx
−1

Buscamos la función primitiva


1
x3

A1 = + x2 − 3x + C
3 −1

Aplicamos la regla de Barrow


 3
(−1)3
  
1 2 2 16
A1 = +1 −3·1 − + (−1) − 3(−1) = −
3 3 3

Luego, A1 = 16 67
3 ya que el área es siempre un número positivo .
De la misma forma, calculamos el área A2 entre x = 1 y x = 3
Z 3
A2 = (x2 + 2x − 3) · dx
1

Buscamos la función primitiva


3
x3

A2 = + x2 − 3x + C
3 1

Aplicamos la regla de Barrow


 3   3 
3 2 1 2 32
A2 = +3 −3·3 − +1 −3·1 =
3 3 3

Luego, A2 = 323
Entonces el área de la región buscada es la suma entre las dos áreas:
16 32
A = A1 + A2 = + = 16
3 3
67
Es el valor absoluto de la integral.

155
c) El área se encuentra delimitada por dos funciones.
Por ejemplo: Hallen el área delimitada entre f (x) = x2 y g(x) = 4x − 3.
En este caso, no conocemos (en un principio) los extremos de integración. Pero
notamos a partir del gráfico, que las curvas se intersecan en dos puntos.

Entonces, los extremos de integración son los valores de las abscisas de dichos
puntos. Es decir que tenemos que hallar los valores de x en los cuales se intersecan
las curvas. Para ello resolvemos la ecuación f = g.

x2 = 4x − 3
x2 − 4x + 3 = 0
Aplicamos la fórmula de Bhaskara68 y obtenemos x1 = 1; x2 = 3.
El área delimitada entre dos curvas la podemos calcular como la integral de la
función techo menos la integral de la función piso 69 . Para determinar cuál es el techo
y cuál es el piso podemos evaluar las funciones en un valor cualquiera de x del
intervalo (1; 3); por ejemplo, en x = 2. Notamos que f (2) = 4; g(2) = 5; es decir que
en todo el intervalo70 sucede que f < g; entonces f es el piso y g es el techo. Luego
el área la podemos calcular como
Z 3
A= [(4x − 3) − (x2 )]dx
1
Z 3
A= [4x − 3 − x2 ]dx
1
Calculamos la primitiva
3
x3

2
A = 2x − 3x − +C
3 1
Aplicamos la regla de Barrow

33 13
   
2 2 4
A= 2·3 −3·3− − 2·1 −3·1− =
3 3 3
4
Luego A = 3
d ) El área se encuentra delimitada por dos funciones que se intersecan más de dos veces.
Por ejemplo: Halle el área delimitada por las funciones f (x) = x3 + 3x2 , g(x) = 4x.

68 −b± b2 −4ac
La fórmula resolvente de la ecuación cuadrática: Si ax2 + bx + c = 0 y a 6= 0 entonces x = 2a
.
69
Dadas dos funciones f y g tales que f < g decimos que en ese intervalo, f es la función piso y g es la
función techo.
70
Ésto es debido a que existen sólo dos puntos de intersección entre f y g y en virtud del teorema de
Bolzano.

156
Buscamos las intersecciones entre f y g y notamos que se intersecan en tres puntos,
cuyas abscisas son:
x1 = −4 x2 = 0 x3 = 1
Entonces calculamos el área de cada una de las regiones. Notamos que en la región
A1 el techo es la función f y el piso es la función g; en cambio en la región A2 el
techo es la función g y el piso es la función f .
Entonces Z 0
A1 = (f − g) · dx
−4
y Z 1
A2 = (g − f ) · dx
0
La resolución de las integrales quedan a cargo del alumno. Luego, el área buscada es
A = A1 + A2 .
e) El área se encuentra delimitada por más de dos funciones.
Por ejemplo: Halle el área delimitada por las funciones f (x) = x2 , g(x) = 6 − x,
h(x) = − 12 x, en el semiplano de abscisas positivas.
En este caso buscamos todas las intersecciones entre las curvas, y separamos la
región cada vez que cambia el techo o el piso.
Notamos según la gráfica que el piso es siempre la curva h pero que el techo, según
los valores de x es la curva f o la curva g.

Entonces buscamos los valores de x en los cuales se intersecan las curvas:

f =g

x2 = 6 − x
x1 = 2; x2 = −3

157
(nos interesa sólo el valor de x1 ).
f =h
1
x2 = − x
2
1
x1 = 0; x2 = −
2
(nos interesa sólo el valor de x1 ).
g=h
1
6−x=− x
2
x = 12
Separamos la región en dos áreas:
Z 2
A1 = [f (x) − h(x)] · dx
0
Z 12
A2 = [g(x) − h(x)] · dx
2
Y obtenemos
11
A1 =
3
A2 = 25
Luego, el área buscada es la suma de las dos integrales calculadas:
A = A1 + A2
11
A= + 25
3
86
A=
3
Problema 445. Encuentren las áreas de las regiones comprendidas entre cada una de las
siguientes funciones y el eje X:

a) y = x2 − 2x, para 3 ≤ x ≤ 5 d ) y = x3 − 4x, para −2 ≤ x ≤ 2


b) y = 3x2 − 1, para −2 ≤ x ≤ 2 e) y = x1/3 , para −1 ≤ x ≤ 8
c) y = x3 − 3x2 + 2x, para 0 ≤ x ≤ 2 f ) y = x1/3 − x, para −1 ≤ x ≤ 8

50 8 3
Respuestas. a) 3 . b) 12 + 9 . c) 12 . d) 8. e) 51
4 . f) 87
4 .

Problema 446. Hallen el área encerrada entre las gráficas de las funciones f (x) y g(x).
Sugerencia: hallen primero los puntos de intersección entre cada par de funciones y
determinen los límites de integración.
a) f (x) = x2 ; g(x) = 2x

b) f (x) = 3 x ; g(x) = 2x − 9 en el primer cuadrante.
c) 3
f (x) = x ; g(x) = x
d) f (x) = sen(x) ; g(x) = cos(x) ; − π2 ≤ x ≤ π2
x2
e) f (x) = −x2 + 4 ; g(x) = 2
√ √
Respuestas. a) 43 . c) 12 . d) 2 2. e) 32
9 6.
Problema 447. Sean las funciones f (x) y g(x) tales que:
Z2 Z5 Z5
f (x) dx = 4 f (x) dx = 6 g(x) dx = 8
1 1 1
Calculen las siguientes integrales definidas aplicando las propiedades pertinentes:

158
R2 R5
a) g(x) dx = e) f (x) dx =
2 2
R1 R5
b) g(x) dx = f) [f (x) − g(x)] dx =
5 1
R1 R5
c) f (x) dx = g) [4f (x) − g(x)] dx =
5 1
R2 R5
d) 3f (x) dx = h) af (x)dx =
1 2

Respuestas. a) 0. b) − 8. c) − 6. d) 12. e) 2. f ) − 2. g)16. h) 2a.

Problema 448. Sean dos funciones f (x) y h(x) integrables y tales que

Z9 Z9 Z9
f (x) dx = −1 f (x) dx = 5 h(x) dx = 4
1 7 7

Utilicen las propiedades de integrales definidas para calcular:

R9 R7
a) −2f (x) dx = e) f (x) dx =
1 1
R9 R9
b) [f (x) + h(x)] dx = f) (f (x) − 2x3 ) dx =
7 1
R9 R7
c) [2f (x) − 3h(x)] dx = g) [h(x) − f (x)] dx =
7 9
R1 R1
d) f (x) dx = h) f (x)dx =
9 7

Respuestas. a) 2. b) 9. c) − 2. d) 1. e) − 6. f ) − 3281. g) 1.
R2
Problema 449. Sea f (x) dx = 5.
1
Encuentren:

R2 R2
a) f (u) du = c) [−f (t)] dt =
1 1
2

R2 √ R3
b) 3f (z) dz = d) f (3x)dx =
1
1 3


Respuestas. a) 5. b) 5 3. c) − 5. d) 53 .

6.2. Longitud de un arco de curva

Denominamos arco de curva a una porción finita y continua de una curva, generalmente
asociada a la gráfica de una función.
En esta sección nos ocuparemos del cálculo de la longitud de un arco de curva.
Observamos la figura de análisis.

159
Notamos que es posible aproximar la longitud del arco de curva A (que va desde P0 hasta
Pn ) de la gráfica como la suma de los segmentos Pk Pk+1 . Es decir que:
n−1
X
Long(A) ≡ Pk Pk+1 (1)
k=0

Además cada segmento es la hipotenusa de un triángulo rectángulo, en los cuales uno de


los catetos mide xk+1 − xk y el otro mide yk+1 − yk . Entonces
2
Pk Pk+1 = (xk+1 − xk )2 + (yk+1 − yk )2

Equivalentemente q
Pk Pk+1 = ∆x2 + ∆y 2 (2)
Si esperamos que la aproximación sea la mejor posible entonces debemos minimizar la
longitud de los segmentos Pk . Para ello, tomamos el límite con ∆x → 0. De esta manera,
sustituimos (2) en (1) y obtenemos:
xn q
X
Long(A) = lı́m ∆x2 + ∆y 2
∆x→0
x1

Según la definición del diferencial de una función, resulta:


Z xn q
Long(A) = dx2 + dy 2 dx
x1

Dividimos y multiplicamos el argumento de la raíz cuadrada (en el segundo miembro) por


dx2 s
dy 2
Z xn
Long(A) = 1 + 2 dx
x1 dx
Y según la definición de la derivada resulta
Z xn q
Long(A) = 1 + y02
x1

160
√ la longitud del arco de curva comprendido en el intervalo x ∈ [0; 5]
Problema 450. Hallen
de la función y = x3 .

Resolución. Calculamos la derivada de la función dada:


 √ 0  3  0 3 1
y0 = x3 = x 2 = x 2
2
Entonces  2
02 3 1 9
y = x2 = x
2 4
Reemplazamos el cuadrado de la derivada en la fórmula obtenida anteriormente.
Z 5r
9
Long(A) = 1 + xdx
0 4

Resolvemos la integral: r
Z 5
4 + 9x
Long(A) = dx
0 4
Z 5√
1
Long(A) = 4 + 9xdx
2 0
Consideramos las tres sustituciones:
dt
t = 9x + 4 dt = 9 · dx = dx
9
Sustituimos en la integral, y simplificamos. Resulta
 5
1 2 3
Long(A) = · (9x + 4) 2
18 3 0

En consecuencia
335
Long(A) =
27
Problema 451. Calculen la longitud del arco de curva que se obtiene de graficar la
función f en el intervalo dado.
3 √
a) f (x) = 32 x 2 + 1 x ∈ [0; 4] Respuesta: L = 10
3 5 − 32

b) f (x) = ln(x) x ∈ [1; 2 2] Respuesta: L = 2, 12
x4 +48 53
c) y = 24x x ∈ (2; 4) Respuesta: L = 24

d ) y = ln(cos x) x ∈ (0; π3 ) Respuesta: L = ln(2 + 3)

Problema 452. Prueben que la longitud de una circunferencia de radio r es igual a 2πr.

Volumen de sólidos de revolución

En esta sección mostramos una estrategia para calcular el volumen de un sólido obtenido
a partir de girar una curva alrededor de alguno de los ejes. Comenzamos viendo qué
sucede si la curva gira alrededor del eje X.
Dada la gráfica de la función f y el sólido que se obtiene al girar dicha curva alrededor
del eje X:

161
Consideramos una porción infinitesimal de dicho sólido; es decir la parte del sólido que se
obtiene de considerar solamente una porción de la curva, digamos de base igual a una
pequeña parte de ella (es decir, la porción correspondiente a un dx).

El volumen de esta porción71 es:


dV = π · r2
siendo r el radio del disco. Pero notamos que el radio es variable, e igual al valor que toma
la imagen de x en cada uno de los puntos de la curva; es decir que r = f (x). Entonces

dV = π · [f (x)]2

Como el volumen que buscamos es la suma de los infinitos discos infinitesimales, y bajo el
mismo argumento esbozado al comienzo de este capítulo, entonces
Z b
V = π · [f (x)]2 · dx
a

Notamos que π es una constante, y reescribimos el cuadrado de la función f :


Z b
V =π· f 2 (x) · dx
a

En el caso de que el sólido de revolución sea generado por la rotación de una superficie
(digamos delimitada por dos curvas f y g) el razonamiento es análogo, y resulta
Z b
V = [πf 2 (x) − πg 2 (x)]dx
a

Si la superficie gira alrededor del eje Y , y a través de un razonamiento análogo,


obtenemos que
Z b
V = 2π x · [f (x) − g(x)] · dx
a
71
Sugerimos el repaso de la sección de volúmenes de cuerpos, en el primer capítulo.

162
Problema 453. Halle el volumen del sólido que se obtiene de girar la superficie delimitada
por las funciones f y g alrededor del eje indicado.

a) y = x2 con 0 ≤ x ≤ 2 alrededor del eje X


b) y = x2 con 0 ≤ x ≤ 2 alrededor del eje Y .
c) y = 1 + 2x con 1 ≤ x ≤ 3 alrededor del eje X.
d ) y = 1 + 2x con 1 ≤ x ≤ 3 alrededor del eje Y .
e) y = ln(x) con 1 ≤ x ≤ e alrededor del eje Y .

f ) f (x) = x2 g(x) = 8x alrededor del eje X.

g) f (x) = x2 g(x) = 8x alrededor del eje Y .
h) f (x) = 2x − x2 g(x) = 0 alrededor del eje Y .
1
i) f (x) = x2 g(x) = x alrededor del eje Y .
1
j ) f (x) = x2 g(x) = x alrededor del eje X.

6.3. Aplicaciones físicas de las integrales

Problema 454. La velocidad de un cuerpo que se mueve a lo largo de un eje s es


ds
dt = v = 9, 8t − 3.

a) Determinen el desplazamiento de dicho cuerpo en el intervalo de tiempo de t = 1 a


t = 3, dado que s = 5 cuando t = 0.
b) Hallen el desplazamiento del cuerpo de t = 1 a t = 3, dado que s = −2 cuando t = 0.
c) Hallen el desplazamiento del cuerpo de t = 1 a t = 3, dado que s = s0 cuando t = 0.

Respuestas. a) s(t) = 4, 9t2 − 3t + 5 b) 33, 2 c)33, 2.

Problema 455. Un cohete se eleva desde la superficie terrestre con una aceleración
constante de 20m/seg 2 . ¿Qué tan rápido irá luego de 1min?

m
Respuestas. v(t) = 20t; v(1min) = 1200 seg .

Problema 456. Una persona conduce un automóvil por una ruta a una velocidad
constante de 96km/h, cuando ve un accidente y frena de golpe. ¿Qué desaceleración
constante se requiere para detener el automóvil en 100m? Para averiguarlo, realicen los
siguientes pasos Sugenrencias: Tengan en cuenta las unidades del problema; midan el
tiempo y la distancia desde el momento en que se pisan los frenos.:

163
 2
d s
 dt2 = −k

a) Resuelvan el problema de valor inicial P : ds
dt = 96

s(0) = 0

b) Encuentren los valores de t que hacen ds/dt = 0 (la respuesta involucra a k).
c) Encuentren el valor de k que hace s = 96 para el valor de t que se encontró en el
paso anterior)

Problema 457. La ecuación estándar para la posición s de un cuerpo que se mueve con
aceleración constante a lo largo de una recta coordenada es s = 21 at2 + v0 t + s0 , donde v0
y s0 son la velocidad y la posición del cuerpo en el instante inicial t = 0. Deduzcan esta
ecuación resolviendo el problema de valor inicial
 2
d s
 dt2 = a

P : ds
dt = v0

s(0) = s0

Problema 458. Sabemos que la velocidad de un móvil está dada por la ecuación
v(t) = 2t3 − cos(tπ) + 1.

a) Prueben que parte del reposo.


b) Hallen la ecuación de la posición del móvil.
c) Hallen la distancia recorrida en los primeros 5 segundos.

Respuestas. b) s(t) = 12 t4 − π1 sen(πt) + t. c) s(5) − s(0) = 317, 5.

Problema 459. Para estirar un resorte debemos aplicar una fuerza F variable, que -según
la Ley de Hooke- definimos F = k · x donde x es el estiramiento y k es una constante real
que depende del resorte. El trabajo realizado por la fuerza es W = F · x; notamos que, a
medida que aumenta la extensión del resorte, la fuerza necesaria también aumenta.
Entonces
Zx
W = F (x)dx
0

Demuestren que el trabajo coincide con el área bajo la recta de ecuación f (x) = kx que
define la fuerza F .

6.4. Métodos de integración

Integración por sustitución

El método por sustitución para integrar consiste en transformar una integral no


inmediata en otra inmediata a través de una sustitución de variables.
Supongamos que se presenta la función compuesta (f og)(x) = f [g(x)] y tenemos que
integrar: Z
I= f [g(x)]g 0 (x)dx

En estas condiciones podemos aplicar el método de integración por sustitución.


Los pasos que tenemos que realizar son:

a) Una parte de la integral la denominamos t, siendo t = g(x).


b) Calculamos el dt = t0 dx, porque la variable de integración debe coincidir con la
variable del diferencial. (No podemos resolver una integral cuya variable sea t y el
diferencial sea dx).

164
f (t) · t0 dx = f (t)dt , si dt = t0 dx).
R R R
c) Escribimos f (t)dt (ya que
d ) RObtenemos una integral inmediata cuya variable es t. Resolvemos la integral
f (t)dt.
e) Finalmente expresamos el resultado en términos de la variable original que es x.

Problema 460. Calculen (x + 4)2 dx aplicando el método de sustitución.


R

Resolución.
Consideramos: I = (x + 4)2 dx −→ u = (x + 4) −→ du = dx Sustituimos (x + 4) por t y
R

calculamos dt:
dt = t0 · dx = (x − 4)0 · dx = 1 · dx = dx
Luego integramos por tabla:
t3
Z
I= +C t2 dt =
3
Finalmente expresamos el resultado con respecto a la variable original que es x:
(x + 4)3
I= +C
3
Problema 461. Calculen las siguientes integrales indefinidas usando el método de
sustitución:

j ) x12 cos2 x1 dx =
R R 
a) sen 3x dx =
b) x sen 2x2 dx =
R  R
R k ) cosec 2θ cotan 2θ dθ =
c) sec 2t tan 2t dt = R dx
l ) √5x+8 =
R
d ) (1 − cos t) sen t · dt =
m) √dx
R
=
e) 28 (7x − 2)−5 dx =
R
1−x
2 2 ·dx
n) ax2 +x
R
f ) x3 x4 − 1 dx =
R
3 =
R 9r2 dr R √
g) √1−r3 = ñ) x a2 − x2 dx =
R 4 2 3 R
y + 4y 2 + 1

h) y + 2y dy = o) cotan(x)dx =
R√ R dx
xsen x3/2 − 1 dx =

i) p) 1+sen(x) =

Respuestas.
(y 4 +4y 2 +1)3 3
a) I = − cos(3x)
3 +k h) I = 12 +k ñ) I = − 13 a2 − x2 2
+k
cos(2x2 ) 3
b) I = − 4 +k i) I = − 23 cos(x 2 − 1) + k o) I = ln|sen(x)| + k
1 −1

c) I = 2cos((2t) + k j) I = − 2x1
+ sen(2x +k p) I = −2 1 − x + k
4
1 2 1 q) I = 13 ln(a2 + x3 ) + k
d) I = 2 (1 − cos(t)) +k k) I = − 2 cosec(2t) + k
3
e) I = −(7x − 2)−4 l) I = 2 1
r ) I = − 31 (a2 − x2 ) 2
5 (5x + 8) + k.
2
1 4 √
f) I = 12 (x − 1)3 + k m) I = −2 1 − x + k s) I = ln(sen(x)) + k
1
1
g) I = −6(1 − x3 ) 2 +k n) I = 3 ln|a
2 + x3 | +k t) I = tan(x) + sec(x) + k

Método de integración por partes

Recordemos que, según las propiedades de las derivadas:

(u · v)0 = u0 v + uv 0

Entonces, sus integrales son iguales72 .


72
O difieren en una constante. Este hecho lo reflejaremos al finalizar la integral.

165
Z Z
(u · v)0 · dx = (u · v 0 + v · u0 ) · dx

Aplicamos las propiedades del los diferenciales y recordamos que la integral es la


aplicación inversa a la derivada, por lo tanto el primer miembro de la igualdad es igual a
u · v. Z Z
u · v = u · dv − v · du

Análogamente Z Z
u · dv = uv − v · du

Y esta última fórmula es la que usamos para integrar por el método de integración por
partes.
El objetivo es transformar una integral no inmediata en resta de un producto de
funciones y una integral que debe ser inmediata o por lo menos más sencilla que la
original. Se presentan distintos casos.

Problema 462. Calculen las siguientes integrales utilizando el método de integración por
partes:

sen2 (x) · dx =
R R
a) (x + 3) sen(x) dx = j)
R
b) xcos x dx = x5 ln(x) · dx =
R
k)
c)
R
t2 cos t dt = R √
R l) x 1 + x dx =
d) ln x dx = R
m) sen(ln x) dx =
x2 ex dx =
R
e) R 3√
n) x 1 − x2 dx =
p3 e−p dp =
R
f) R x
g)
R
arcsen(x) dx = ñ) e · sen(x) · dx =
R 3x+1
o) e · cos(x) · dx =
R
h) arctan(x) dx =
(x + 3) · ex dx =
R x
e · (x2 − 2x + 7)dx =
R
i) p)

Respuestas.

a) (x + 3)cos(x) + sen(x) + k j ) I = − 21 cos(x)sen(x) + k =


1 1
b) x · sen(x) + cos(x) + k 2 x − 4 sen(2x) + k

c) I = 2tcos(t) + (−2 + t2 )sen(t) + k k ) I = 16 x6 ln(x) − 36


1 6
x +k

d ) I = −x + xln(x) + k l ) I = 5 (1 + x) 1 + x − 23 (1 + x) + k
2 2

e) I = ex (2 − 2x + x2 ) + k m) I = − 21 xcos(ln(x)) + 12 xsen(ln(x)) + k
3 5
f ) I = e−p (−6 − 6p − 3p2 − p3 ) + k. n) I = − 31 (1 − x2 ) 2 + 51 (1 − x2 ) 2 + k

g) I = x · arcsen(x) + 1 − x2 + k ñ) I = − 21 ex cos(x) + 12 ex sen(x) + k
1 3x+1 3 3x+1
h) I = x · arctan(x) − 21 ln(1 + x2 ) + k o) I = 10 e sen(x) + 10 e cos(x) + k
i) I = (x + 2)ex + k p) I = ex (x2 − 4x + 11) + k

Integración por fracciones simples

Este método lo utilizamos para integrales del tipo:


Z
p(x)
I= dx
q(x)

Tales que p(x) y q(x) 6= 0 son polinomios y Gr(p) < Gr(q).

166
El método consiste en descomponer una expresión racional en suma de fracciones del tipo
Aj
x−rj , siendo Aj un número real cualquiera y rj una raíz de q(x). La suma tendrá tantas
fracciones como lo indique el grado de q. Primero efectuamos la descomposición y luego
integramos.
Primer caso: Las raíces de q(x) son reales y distintas.
Problema 463. Hallen los números A, B, r1 y r2 tales que
3x − 1 A B
= +
x2 − 5x + 6 x − r1 x − r2

Resolución. Observamos que las raíces de q son r1 = 2; r2 = 3. Como el grado de q es 2,


entonces planteamos:
3x − 1 A B
= +
x2 − 5x + 6 x−2 x−3
Extraemos denominador común en el segundo miembro de la igualdad.
3x − 1 A(x − 3) + B(x − 2)
=
x2 − 5x + 6 (x − 2)(x − 3)
Simplificamos los denominadores y obtenemos
3x − 1 = A(x − 3) + B(x − 2)
Para hallar los valores de A y B podemos reemplazar la variable x por cualquier número
real (ya que los valores obtenidos deben verificar la igualdad para cualquier x). Sin
embargo la ecuación resulta más sencilla si reemplazamos sucesivamente por cada una de
las raíces de q. Entonces, si x = 2:
3 · 2 − 1 = A(2 − 3) + B(2 − 2) ⇒ A = −5
Análogamente, si reemplazamos x = 3 obtenemos B = 8. Entonces
3x − 1 −5 8
= +
x2 − 5x + 6 x−2 x−3
Segundo caso: El polinomio q(x) tiene raíces reales múltiples.
En este caso, si una de las raíces r es de multiplicidad n entonces habrá n términos de la
A
forma (x−r) n−k con k = 1, 2, · · · n. El procedimiento es idéntico.

5x2 −36x+48
R
Problema 464. Calculen la integral x3 −8x2 +16x
dx.

Resolución.
Notamos que las raíces de q(x) son x1 = 0; x2 = x3 = 4. Entonces planteamos la igualdad
5x2 − 36x + 48 A B C
3 2
= + +
x − 8x + 16x x x − 4 (x − 4)2
Sumamos las fracciones del segundo miembro y obtenemos
5x2 − 36x + 48 A(x − 4)2 + Bx(x − 4) + Cx
3 2
=
x − 8x + 16x x(x − 4)2
Simplificamos los denominadores (que son idénticos) y obtenemos
5x2 − 36x + 48 = A(x − 4)2 + Bx(x − 4) + Cx
Si reemplazamos x → 0 obtenemos A = 3. Si reemplazamos x → 4 obtenemos C = −4.
Para obtener el valor de B podemos reemplazar los valores de A y C obtenidos, y x por
cualquier número real. Obtenemos B = 2. Luego
5x2 − 36x + 48 −4
Z Z Z Z
3 2
I= dx = dx + dx + dx
x3 − 4x2 + 4x x x−4 (x − 4)2

167
4
I = 3ln(x) + 2ln(x − 4) + +C
x−4
Tercer caso: Uno de los factores de q(x) es un polinomio de grado 2 sin raíces reales.
4x2 −8x+1
R
Problema 465. Calculen la integral (x+2)(x 2 −2x+3) dx

Resolución. Notamos que el denominador q(x) tiene una raíz real que es x = −2 y un
factor cuadrático sin raíces reales. Entonces descomponemos el argumento de la integral
como una suma de tres fracciones:
4x2 − 8x + 1 A B Cx
= + +
(x + 2)(x2 − 2x + 3) x + 2 x2 − 2x + 3 x2 − 2x + 3

Observamos que la tercera fracción sólo difiere de la segunda en que el numerador tiene
un factor igual a x.
Entonces sumamos las fracciones del segundo miembro de la igualdad

4x2 − 8x + 1 A(x2 − 2x + 3) + B(x + 2) + Cx(x + 2)


=
(x + 2)(x2 − 2x + 3) (x + 2)(x2 − 2x + 3)

Simplificamos los denominadores, que son iguales

4x2 − 8x + 1 = A(x2 − 2x + 3) + B(x + 2) + CX(x + 2)

Si reemplazamos x → −2 obtenemos A = 3. Para obtener el valor de B reemplazamos el


valor de A obtenido, y x → 0. Obtenemos B = −4. Para hallar el valor de C
reemplazamos los valores de A y B obtenidos, y x por cualquier número real. Obtenemos
C = 1. Entonces
4x2 − 8x + 1 −4
Z Z Z Z
3 x
2
dx = dx + 2
dx + 2
dx
(x + 2)(x − 2x + 3) x+2 x − 2x + 3 x − 2x + 3

La primera integral del miembro de la izquierda es inmediata:


Z
3
dx = 3ln(x + 2) + C
x+2
La segunda y tercera integral las reescribimos como sigue:
−4 x−1
Z Z Z Z
x 3
2
dx + 2
dx = 2
− 2
x − 2x + 3 x − 2x + 3 x − 2x + 3 x − 2x + 3
De las integrales resultantes, la primera la podemos resolver sustituyendo
t = x2 − 2x + 3; dt = 2(x − 1)dx. Luego:

x−1
Z Z
1 dt 1 1
2
= = ln(t) = ln(x2 − 2x + 3) + C
x − 2x + 3 2 t 2 2

La segunda requiere la sustitución z = x + 1; z 2 = x2 − 2x + 1; dz = dx. Luego:


√  
x−1
Z Z
3 dz 3 2
dx = 3 · = arctan √
x2 − 2x + 3 z2 + 2 4 2
Entonces
√  
1 2 3 2 x−1
I = 3ln(x + 2) + ln(x − 2x + 3) + arctan √ +C
2 4 2

Problema 466. Calculen las siguientes integrales.

168
1 x3 +x2 +x+2
R R
a) I = x2 −4
dx e) I = x2 +3x+2
dx
x+1 x2
R R
b) I = x3 +x2 −6x
dx f) I = 1−x4
dx
3x+5 3x2 −8x+6
R R
c) I = x3 −x2 −x+1
dx g) I = x2 −4x+4
dx
2x+7 x+7
R R
d) I = x−x2
dx h) I = x2 −8x+7
dx

Respuestas.

a) I = 41 ln(x − 2) − 14 ln(x + 2) + k e) I = 12 x2 − 2x + ln(x + 1) + 4ln(x + 2) + k


b) I = − 16 ln(x)+ 10
3 2
ln(x−2)− 15 ln(x+3)+k f ) I = 41 ln(x+1)− 41 ln(x−1)− 12 arctan(x)+k
2
c) I = − 12 ln(x − 1) − 4
+ 12 ln(x + 1) + k g) I = 3x + 4ln(x − 2) − x−2 +k
x−1
7 4
d ) I = −9ln(1 − x) + 7ln(x) + k h) I = 3 ln(x − 7) − 3 ln(x − 1) + k

Problema 467. Resuelvan las siguientes integrales aplicando el método conveniente.


Verifiquen el resultado obtenido.
dx
(2x − 3) · ex · dx =
R R
a) j) x2 −4
=
R
b) sen(x) · cos(x) · dx = k)
R
tan(x) · dx =
5x · ln(x2 ) · dx =
R
c)
(x − 1)2 cos(x3 − 3x2 + 3x − 1)dx =
R
l)
5)2 ex
R
d) (x − · · dx = R
R ln(x)
m) (3x − 7) · cos(x) · dx =
e) x · dx =
sen(x) · ex · dx =
R
√ n)
(x − 5)2 · (3 −
R
f) x) · dx =
2x−1
R
2 ñ) · dx =
xe−x · dx =
R
g) x2 −x+5
2
3x · ex · dx =
R 2x−3
R
h) x2 −3x+2
dx = o)
2x−5
R√
x · x1 + 5x − 2 · dx =
R 
i) x2 −3x+2
dx = p)

169
Respuestas.

a) I = ex (−3 + 2x) + k j ) I = 3cos(x) − 7sen(x) + 3xsen(x) + k


b) I = − 12 cos2 (x) + k k ) I = 12 ex (−cos(x) + sen(x)) + k
c) I = − 52 x2 + 25 x2 ln(x2 ) + k l ) I = −ln(x2 − x + 5) + k
d) I = ex (37 − 12x + x2 ) + k m) I = 32 ex + k
2

1 2
e) I = 2 ln (x) +k 1 3 5
3 5 7 n) I = 2x 2 − 43 x 2 + 2x 2 + k.
f) I = 75x− 50
3 x
2 2
−15x +4x +x2 3
− 27 x 2 +k 2
2 ñ) I = 23 ex + k
g) I = − 21 e−x +k
1 5 3
h) I = −ln(cos(x)) + k o) I = 2x 2 + 2x 2 − 43 x 2 + k
√ √ √
i) I = 31 sen(x3 − 3x2 + 3x − 1) + k p) I = 2 x5 − 43 x3 + 2 x + k

Problema 468. Hallen el error en el siguiente razonamiento:


Z
1
I= · dx
x
Calculamos la integral por partes:
1 1
u= −→ du = − 2 · dx
x x
dv = dx −→ v = x
Entonces Z  
1 1
I = ·x− x· − 2 · dx
x x
Simplificamos y obtenemos Z
1
I =1+ · dx
x
Equivalentemente
I =1+I
Restamos I en ambos miembros y obtenemos

0=1
Ra
Problema 469. Hallen el valor de a tal que 0 (2x + 7)dx = 32.
Respuestas. a = 3, 15

Problema 470. A partir de las hipótesis que enunciamos a continuación, prueben que la
t
carga q de un circuito es igual a q0 · e− R·C .

Ley de Kirchoff (de la conservación de la energía en un circuito): VR + VC = 0.


q
Ley de Ohm: i(t) · R + C = 0.
dq
i(t) = dt .

Resolución. Según la hipótesis


q 1
i(t) = − ·
C R
Reemplazamos i(t)
dq q
=−
dt R·C
Separamos las variables
dq 1
=− · dt
q R·C

170
Integramos ambos miembros de la igualdad
Z Z
dq 1
= − · dt
q R·C
t
ln(q) = − +k
R·C
Aplicamos la definición del logaritmo.
t
q = e− R·C +k

Consideramos q0 = ek y aplicamos las propiedades de las potencias. Entonces


t
q = q0 · e− R·C

Problema 471. Consideren la función f (x) = sen(x).

a) Hallen el polinomio de Taylor de grado cinco, asociado a la función f en torno a


x = 0.
sen(x)
b) Hallen el polinomio de Taylor de g(x) = x de grado cuatro utilizando el
resultado obtenido en el ítem anterior.
c) Calculen en forma aproximada la integral
Z 1
sen(x)
I= dx
0 x

Sugerencia: utilicen el polinomio hallado en el ítem anterior.


d ) Calculen
L = lı́m g(x)
x→0

Respuestas. a) P = x − 61 x3 + 1 5
120 x b) P = 1 − 16 x2 + 1 4
120 x c) I = 1703
1800 d) L = 1.
1
Problema 472. Consideren la función f (x) = x2
.

a) Calculen el área encerrada entre la gráfica de f , el semieje X + y las rectas de


ecuación x = 1 y x = 2.
b) Calculen el área encerrada entre la gráfica de f , el semieje X + y las rectas de
ecuación x = 1 y x = 10.
c) Calculen el área encerrada entre la gráfica de f , el semieje X + y las rectas de
ecuación x = 1 y x = 99999.
d ) ¿Está acotada la región delimitada por la curva f , el semieje X + y la recta x = 1?
e) ¿Es finita el área de la región mencionada en el ítem anterior?
1 9
Respuestas. a) 2 b) 10 c) 1 d) No está acotada. e) Es finita.
2
Problema 473. Calculen el área encerrada bajo la gráfica de la función f (x) = e−x entre
x = −1 y x = 1. Sugerencia: hallen el polinomio de Taylor de la función dada.
Respuestas. A = 5235

171
Autoevaluación: integración
3
1. Halle una función f (x) tal que f 0 (x) = 2x −5x+1
x .
2. Resuelva el siguiente problema de valores iniciales
(
dy
dx = 2x − sen(x)
y(0) = 4

3. Un cuerpo se mueve a lo largo de una recta con velocidad


√ √
3
v(t) = t + 2 t2

Determine el desplazamiento de dicho cuerpo en el intervalo de tiempo entre t = 1 y t = 8.


R5 R5 R2
4. Sabiendo que f (x)dx = 9 y que 25 f (x)dx = 2, halle 6f (x)dx.
2 3 3
5. Halle el área encerrada entre las gráficas de las funciones

(x) = x2 − 3x − 4 g(x) = 3x − 9

6. Calcule las siguientes integrales por el método conveniente.


R
a) (5x + 7) · sen(x) · dx =
3
b) (x2 + 2) · ex +6x−1 · dx =
R

Respuestas a la autoevaluación.
1. 32 x3 − 5x + ln(x) + k con k ∈ R.
2. y = x2 + cos(x) + 3.
3. ∆s ∼ = 51, 62.
R2
4. 6f (x)dx = −24.
3
5. El área es igual a 32
3 .
6a) I = 5sen(x) − (5x + 7)cos(x) + k con k ∈ R.
3
6b) I = ex +6x−1 + k con k ∈ R.

172
6.5. Tabla de integrales

Sugerencia: Agreguen todas las funciones (con sus primitivas) que consideren
interesantes, así como los ejemplos que les puedan ser útiles para entender las
propiedades enunciadas. También pueden consultar una tabla de integrales más completa.
Por ejemplo, en https://www.utnianos.com.ar/foro/attachment.php?aid=12205.

No Función Primitiva Ejemplo


R
1 a ax + k 8 · dx = 8x + k
xn+1 4
n x · dx = x4 + k
R 3
2 x (n 6= −1) n+1 + k
1
3 x ln(x) + k
4 sen(x) −cos(x) + k
5 cos(x) sen(x) + k
6 tan(x) ln(sec(x)) + k
7 cotan(x) ln(sen(x)) + k
8 ln(x) x · ln(x) − x + k
9 ex ex + k
1
10 logb (x) ln(b) · (x · ln(x) − x) + k b ∈ R+ − {1}
1
11 ax x
ln(a) · Ra + k a ∈ R+ − {1}
R
12 f (x) ± g(x) f (x) · Rdx ± g(x) · dx + k
13 a · f (x) a · f (x)R · dx + k a, k ∈ R
14 u · dv u · v − v · du

173
7. Bibliografía y links multimedia
Coll, Pablo & Otros. Problemas para Matemática de Economía y Administración.
(2014). Ed. Universidad de Moreno.
Píngaro, Gonzalo. Diccionario de Teoremas Tomos I y II. (2017) Ed. Imprenta
Argentina.
De Guzmán, Miguel & Otros. Matemática I & II. Ed. Anaya.
Francisco José González Gutiérrez; Apuntes de lógica matemática. Cádiz (2005).
Publicación on-line en https://www.cs.buap.mx/~fjrobles/LogPro.pdf
GeoGebra y trigonometría.En
http://www.xente.mundo-r.com/ilarrosa/GeoGebra/index_trigonometria.html
Resumen de los casos de factoreo. En
https://julioprofe.net/material-de-apoyo/algebra/Resumen-de-los
-principales-casos-de-factorizacion%2C-con%20teoria-y-ejemplos.pdf
Tabla de integrales en
https://www.utnianos.com.ar/foro/attachment.php?aid=12205
Las definiciones y propiedades de los cuerpos geométricos.
http://lya.fciencias.unam.mx/gfgf/ga20132/poliedros/arch8.pdf
Paenza, Adrián. Matemática para todos.
http://cms.dm.uba.ar/material/paenza/libro7/matematica_para_todos.pdf
Revilla, Fernando. Apuntes varios. http://fernandorevilla.es/blog/2014/02/
08/supremo-infimo-maximales-y-minimales/
Rodriguez S, Julio. Valor absoluto. http://www.mat.uson.mx/~jldiaz/Documents/
Desigualdades/valor_absolutov1.pdf
Apuntes, ejercicios y problemas de geometría. https://www.geoka.net.
Autores varios. Composición de funciones.
http://www.mate.unlp.edu.ar/practicas/52_7_26092016102641.pdf.
Mohigefer, R. Problemas de continuidad y derivabilidad.
http://www.e-matematicas.es/Ficheros/2BACH-CT/
Problemas%20resueltos%20de%20continuidad%20y%20derivabilidad.pdf.
Gonzalez Gutierrez, Francisco. Apuntes de lógica matemática.
https://www.cs.buap.mx/~fjrobles/LogPro.pdf.

Bibliografía complementaria para los estudiantes


• Integrales. Técnicas de integración. Aplicación de las integrales. Ed. El
Coloquio. Buenos Aires, 1974.
• Sadoski, M. & Guber, R. Elementos de cálculo diferencial e integral. Editorial
Alsina.
• Leithold, Louise. El cálculo. Ed. Oxford University Press.
• Pearson, Thomas. Cálculo en una variable.
• Larson, Roland & Hostetler, Robert. Cálculo y geometría analítica. Vol. I.
• Stewart, James. Cálculo; conceptos y contextos. Thomson Editores.
• Sagastume - Berra - Fernández. Álgebra y cálculo numérico. Editorial Kapeluz.

174

También podría gustarte